Sie sind auf Seite 1von 147

Anatomy -2015

Section

Part
(Q.1) Hypogastric sheath is a condensation of
Scarpa's fascia
(a)
Colles fascia
(b)
Pelvic fascia
(c)
Lumbar fascia
(d)
Your Response :
Correct Answer
C
:
Pelvic fascia
Exp:
Hypogastric sheath is a condensation of pelvic fascia
The hypo-gastric sheath is a pelvic fascia condensation.
It lies along the postero-lateral pelvic walls and carries the neuro-vascular bundles
towards the pelvic viscera.
It also provides pelvic viscera support.
As hypogastric sheaths extend towards the viscera 3 lamina are formed:
Anterior lateral ligament of urinary bladder.
It carries superior vesical arteries and veins
Posterior lateral ligaments of rectum
It carries middle rectal arteries and veins
Middle lamina
a. male recto-vesical septum (between urinary bladder & rectum)
b. female Cardinal ligament of uterus
- It carries uterine artery in its superior-most portion at the base of broad ligament
of uterus.
Pelvic fascia condensations are good supports of uterus and include ligaments
like:
Transverse cervical ligament of Mackenrodt (also called cardinal ligament of
uterus as mentioned above))

Pubo-cervical ligament
Sacro-cervical ligament
Supports of uterus:
Muscular (Dynamic supports) - provide excellent support
Pelvic diaphragm (levator ani muscle etc.)
Urogenital diaphragm (urethral sphincter & deep transverse perinei muscle)
Perineal body (common perineal tendon for attachment of numerous perineal
muscles)
Pelvic fascia condensations (passive supports) provide good support
Transverse cervical ligament of Mackenrodt (also called cardinal ligament)
Pubo-cervical ligament
Sacro-cervical ligament
Peritoneal folds - provide poor support
Broad ligament
Round ligament of uterus

Prolapse of uterus
May occur if the supports are weakened.
During parturition the muscular supports undergo lot of stretching and may give
up, leading to uterus being pushed inside vagina and come out into the perineum.

Surgical support
The cardinal ligaments have enough fibrous content to provide anchor for the
wide loops of sutures during several surgical procedures.
References:
1. Pg. 345-349; Keith L Moore (6th Ed.)
Vol-2

(Q.2) Which is a tract of posterior column?


Spino rubral
(a)
Tract of gracilis
(b)
Spino cerebellar
(c)
Posterior spinothalamic
(d)
Your Response :

2. Pg. 361; B D Chaurasia (4th Ed.);

Correct Answer
B
:
Tract of gracilis
Exp:
Posterior (dorsal) column is constituted by the fasciculus gracilis&cuneatus.
Dorsal column is a sensory tract and carries sensation like pressure, fine touch &
vibration (PTV). It also carries the tactile discrimination,
proprioception&stereognosis.
References: Pg.147; Snells Clinical Neuroanatomy (6th Ed.)

(Q.3) Which of the following statements about peritoneum and/or mesenteries is INCORRECT?
The gastrosplenic and splenorenal ligaments normally prevent fluids from entering
(a)
the omental bursa from the left paracolic gutter.
(b)

A mesentery transmits the neurovascular supply to the organ with which it is


related
The common bile duct is contained within the gastrocolic ligament.

(c)
The adrenal glands are situated posterior to parietal peritoneum.
(d)
Your Response :
Correct Answer
C
:
The common bile duct is contained within the gastrocolic ligament.
Exp:

The common bile duct is formed by the union of the common hepatic duct and the
cystic duct (from the gall bladder).
It is later joined by the pancreatic duct to form the ampulla of Vater. There, the
two ducts are surrounded by the muscular sphincter of Oddi.
Viscera/Fascia/Peritoneal
Specializations
Organ/Part of Organ
Location/Description Notes
liver
largest digestive organ the liver consists of 4
in the body, located anatomical lobes: left, right,
mostly in the right
quadrate and caudate; it lies
upper quadrant
inferior to the diaphragm and
rises as high as the 5th rib on
the right side; liver functions
include: synthesis, storage
and release of glycogen and
vitamins; synthesis of blood
proteins; phagocytosis of old
red blood cells; removal of
toxic substances
bare area of the liver
the part of the
bare area is between the
diaphragmatic surface anterior and posterior
of the liver that is not laminae of the coronary
covered by visceral
ligament and is in contact
peritoneum
with the diaphragm
porta hepatis
region of the visceral porta hepatis is equivalent to
surface of the liver
the hilum of the spleen or
where hepatic ducts, lung
hepatic aa. & portal v.
enter/leave

quadrate lobe

ligamentum venosum

gallbladder

part of liver between


the bed of the
gallbladder and the
round ligament of the
liver
cord-like ligament
that lies within the
attachment of the
hepatogastric
ligament to the liver,
between the caudate
lobe and the left lobe
diverticulum of the
biliary tract that hangs
below the liver
between the right and
quadrate lobes

gallbladder, body of

the main part of the


gallbladder

gallbladder, fundus of

expanded inferior part


of the gallbladder

gallbladder, neck of

constricted superior
part of the gallbladder

hepatic duct, common

duct formed by the


joining of the of right
and left hepatic ducts
duct that drains the
left lobe of the liver,
quadrate lobe and part
of the caudate lobe of
the liver
duct that drains the
right lobe of the liver
and part of the
caudate lobe of the
liver
part of the biliary tree
that is formed by the
union of the cystic
and common hepatic

hepatic duct, left

hepatic duct, right

bile duct, common

quadrate lobe has the


external appearance of being
part of the right lobe, but it is
functionally part of the left
lobe
ligamentum venosum is a
remnant of the obliterated
ductus venosus which
shunted oxygenated blood
from the placenta around the
liver to the inferior vena
cava during development
located in the right upper
quadrant; gallbladder is
connected to the common
bile duct via the cystic duct;
it both stores and
concentrates bile; pain from
the gallbladder may be
referred to the right upper
quadrant or to the right
shoulder
body of the gallbladder lies
in the bed of the gallbladder,
between the right and
quadrate lobes of the liver
fundus of the gallbladder is
located near the inner
surface of the anterior
abdominal wall at the
junction of the right 9th
costal cartilage and the right
semilunar line; it normally
hangs below the margin of
the liver
neck of the gallbladder is the
portion that connects the
body of the gallbladder to
the cystic duct; it contains a
spiral fold
common hepatic duct unites
with the cystic duct to form
the common bile duct
left hepatic duct joins with
right hepatic duct to form the
common hepatic duct

right hepatic duct joins with


the left hepatic duct to form
the common hepatic duct

common bile duct drains into


the hepatopancreatic ampulla
and greater duodenal papilla
in company with the main

ducts; located in the pancreatic duct


hepatoduodenal
ligament
cystic duct
duct of the gallbladder cystic duct joins with the
common hepatic duct to
form the common bile duct;
cystic duct both fills and
drains the gallbladder
falciform ligament
sickle-shaped fold of falciform ligament contains
peritoneum
the round ligament of the
connecting the liver to liver (ligament teres hepatis);
the umbilicus
it is a remnant of the ventral
mesogastrium of the embryo
coronary ligament
peritoneum
it has three parts which
connecting the liver to together form a complete
the inferior surface of circle: right triangular
the diaphragm
ligament, left triangular
ligament, falciform ligament;
area between the anterior
and posterior laminae of the
coronary ligament is called
the bare area of the liver
hepatoduodenal ligament part of the lesser
hepatoduodenal ligament
omentum connecting contains the common bile
the liver to the 1st
duct, proper hepatic a. and
part of duodenum
portal v.; it forms the
anterior wall of the
omental (epiploic) foramen
hepatogastric ligament
part of the lesser
hepatogastric ligament
omentum connecting forms the anterior wall of
the liver to the lesser the lesser peritoneal sac
curvature of the
stomach
round ligament of the liver cord-like ligament
round ligament of the liver
that lies within the
courses from the liver to the
free margin of the
umbilicus; it is the remnant
falciform ligament
of the umbilical v.; also
known as: ligamentum teres
hepatis
triangular ligament
extreme left or right triangular ligaments are
end of the coronary
formed by the joining of the
ligament of the liver anterior and posterior
laminae of the coronary
ligament; they delineate the
right and left extremes of the
bare area of the liver
ligamentum teres hepatis
cord-like ligament
ligamentum teres hepatis
that lies within the
courses from the liver to the
free margin of the
umbilicus; it is the remnant
falciform ligament
of the umbilical v.; also
known as: round ligament of
liver
peritoneum
serous membrane
visceral peritoneum lies on
lining the peritoneal the surfaces of the
cavity
abdominal and pelvic
organs; parietal peritoneum
lines the inner surfaces of the

peritoneum, parietal

peritoneum, visceral

walls of the abdominopelvic


cavity
serous membrane
peritoneum, pleura and
lining the inner
pericardium are all serous
surfaces of the walls membranes that formed from
of the abdominopelvic the same layer of tissue
cavity
(splanchnic mesoderm) in
the embryo
serous membrane
visceral peritoneum is
lying on the surfaces formed from the splanchnic
of the abdominal and mesoderm in the embryo
pelvic organs

(Q.4) Regarding the azygos venous system True is:


In congenital absence of IVC the azygos vein enlarges.
(a)
About 10% of the population have an azygos lung lobe.
(b)
The thoracic duct and aorta are to the right of the azygos vein.
(c)
The hemiazygous vein is a exact mirror image of entire azygous vein on left side
(d)
Your Response :
Correct Answer
A
:
In congenital absence of IVC the azygos vein enlarges.
Exp:
True in the azygous continuation of the IVC, the azygos is a large structure,but
otherwise the anatomy is unaltered. This may be confused with amediastinal mass
False in 1% of the population, the azygos vein traverses the lung beforeentering
the SVC resulting in the azygos fissure. The azygos lobe is not a truesegment.
False they are to its left.
False hemiazygos, accessory hemiazygos veins together may form a mirror
image of the azygous vein.

(Q.5) Nucleus pulposus of intervertebral disc is the remnant of which of the structures marked below?

(a)
(b)
(c)
(d)
Your Response :
Correct Answer
B
:

A
B
C
D

Exp:

B
B marked structure is notochord. It stimulates the surface ectoderm for
neurulation and it will give rise to nucleus pulposus and apical ligament of the
dens.

(Q.6) Facial colliculus located at


Upper Pons
(a)
Lower Pons
(b)
Medulla
(c)
Interpeduncular fossa
(d)
Your Response :
Correct Answer
B
:
Lower Pons
Exp:
Facial colliculus is a rounded elevation seen in the floor of 4th ventricle, at the
posterior aspect of lower pons (A).
It is raised due to the axons of facial nerve winding around the abducent nucleus.
There are 2 facial colliculi in the floor of fourth ventricle.
Mid brain (C) shows 4 rounded elevations on its posterior aspect: 2 superior and 2
inferior colliculi.
Interpeduncular fossa (D) shows 2 rounded elevations - mamillary bodies in it.
Medulla (B) has 2 rounded elevations on the anterior aspect, called as the olives.
References:
1. Pg. 342; B D Chaurasia (4th Ed.); Vol-2
The facial colliculus is not formed by the facial nerve nucleus, but by the fibres of
the facial nerve arching backwards around the abducens nerve (CN VI) nucleus
before turning forwards once more. The colliculus is an elevation on the floor of
the fourth ventricle.
A lesion involving the facial colliculus is therefore likely to result in
facialcolliculus syndrome by involving:
abducens nerve (CN VI) nucleus
facial nerve (CN VII) fibres at the genu
medial longitudinal fasciculus

(Q.7) A patient experiences visual difficulties. When a light is shined in her right eye, there is no
pupillary response in either eye. However, upon shining a light in her left eye, both ipsilateral and
contralateral pupillary responses are apparent. Her extraocular movements are intact. What is
the most likely location of her lesion?
Oculomotor nerve, left side
(a)

(b)

Oculomotor nerve, right side

Optic nerve, left side


(c)
Optic nerve, right side
(d)
Your Response :
Correct Answer
D
:
Optic nerve, right side
Exp:
This woman has a "Marcus-Gunn pupil" with a defect in the afferent pathway of
the optic nerve (in this case, on the right side). Recall that the afferent limb of the
papillary light reflex is the optic nerve (CN II); the efferent limb is the oculomotor
nerve (CN III; parasympathetic fibers).
When light is shined into her right eye, because her right optic nerve is not
functioning properly, the light signal is not transmitted to the central nervous
system (CNS), resulting in no papillary response.
As light is shined into her left eye, the left optic nerve transmits the signal to the
CNS, which then sends an outbound signal through both the right and left
oculomotor nerves to cause pupillary constriction in both eyes.
Anatomically, the optic nerve is a special sensory nerve that originates in the
retina of the eye, passes through the optic foramen of the sphenoid bone, and has a
destination in the diencephalon via the optic chiasm.
The oculomotor nerve) innervates all extraocular muscles except the lateral rectus
(innervated by the abducens nerve) and the superior oblique (innervated by the
trochlear nerve).
The oculomotor nerve also mediates papillary constriction (parasympathetic
fibers), eyelid opening (levator palpebrae), and innervates the ciliary muscle
(allowing accommodation).
A patient with a lesion of the left optic nerve would have no papillary responses in
either eye when shining a light in the left eye; pupillary responses would be
present in both eyes when shining a light in the right eye.

(Q.8) Association fibres are all exceptUncinate


(a)
Cingulum
(b)
Forceps major
(c)
Longitudinal fasciculus
(d)
Your Response :
Correct Answer
C
:
forceps major
Exp:
Association fibers
1. Cingulum

Projection fibers
1. Fimbria

2. Superior longitudinal 2. Fornix


fasciculus
3. Corona radiata
3. Inferior longitudinal
fasciculus
4. Internal capsule

Commissural fibers
1. Anterior
commissure
2. Hippocampal
commissure
3. Habenular

commissure
4. Uncinate fasciculus
4. Posterior
commissure
5. Corpus callosum

(Q.9) The derivative of the second pharyngeal arch is:


Eustachian tube
(a)
External auditory meatus
(b)
Tensor tympani
(c)
Stylohyoid muscle
(d)
Your Response :
Correct Answer
D
:
Stylohyoid muscle
Exp:
Pharyngeal Arch Artery
Arch
1
Terminal
Branch of
maxillary
artery

Cranial Nerve

Skeletal elements

Maxillary and
mandibular
division of
trigemenial (V)

Derived from arch


cartilages
(originating from
neural crest):

Muscles

Muscles of
mastication
(temporalis,
masseter, and
pterygoids),
mylohyoid,
From
maxillary cartilages: anterior belly of
digastric, tensor
tympani, tensor
Alispenoid,
veli palatini
incus
(originate from
cranial
From
somitomere 4)
mandibular:
Mackels
cartilage, malleus
Upper portion of
external ear (auricle)
is derived from
dorsal aspect of 1st
pharyngeal arch.
Derived by direct
ossification from
arch dermal
mesenchyme:

Maxilla,
zygomatic,
squamous portion of
temporal bone,
mandible
Stapedius
Facial nerve (VII) Stapes, styloid
artery
process, stylohyoid
(embryologic)
ligament, lesser
and
horns and upper rim

Muscles of
facial
expression
(orbicularis

cortiotympanic
artery (adult)

of hyoid (derived
from the second arch
cartilage; originate
from neural crest).

oculi,
orbicularis oris,
auricularis,
platysma,
frontoooccipitalis,
Lower portion of
external ear (auricle) buccinator),
is derived from 2nd posterior belly
of digastric,
pharyngeal arch.
stylohyoid,
stapedius
(originate from
cranial
somitomere 6)
Common
Glossopharyngeal Lower rim and
Sytlopharyngeus
carotid artery, (IX)
greater horn of hyoid (originate from
most of
(derived from the
cranial
internal
third arch cartilage; somitomere 7)
carotid
originate from neural
crest cells)
Left: Arch of Superior
Laryngeal cartilages Constrictors of
aorta;
laryngeal branch (Derived from the 4th pharynx,
of vagus (X)
arch cartilage,
cricothyroid,
originate
from
lateral
levator veli
Right: Right
plate
mesoderm)
palatine
subclavian
(originate from
artery;
occipital
somites 2-4)
Original
sprouts of
pulmonary
arteries
Ductus
Recurrent
Laryngeal cartilages Intrinsic
arteriosus;
laryngeal branch (derived from the 6th- muscles of
roots of
of vagus (X)
arch cartilage;
larynx (originate
definitive
originate from lateral from occipital
pulmonary
plate mesoderm)
somites 1 and 2)
arteries

Origins of craniofacial muscles:


Muscles
Mesodermal Origin
Somitomeres 1, 2
Superior, medial and ventral recti
Somitomere 3
Superior oblique
Somitomere 4
Jaw-closing muscles
Somitomere 5
Lateral rectus
Somitomere 6
Jaw-opening and other 2nd arch
muscles
Somitomere 7
Stylopharyngeus
Somites 1, 2
Somites 2-5

Intrinsic laryngeals
Tongue muscles

Innervation
Oculomotor (III)
Trochlear (IV)
Trigeminal (V)
Abducens (VI)
Facial (VII)
Glossopharyngeal
(IX)
Vagus (X)
Hypoglossal (XII)

Derivates of pharyngeal pouches:


Auditory tube, which comes in contact with epithelial
First pouch
line of first pharyngeal cleft, where future external
acoustic meatus will form.

Distal portion will form tympanic cavity (lining will


become eardrum)

Second pouch
Third pouch
Fourth pouch
Fifth pouch

Proximal portion will become auditory tube


Forms buds that penetrate surrounding mesenchyme,
which together form the palatine tonsils
Forms thymus and inferior parathyroid glands
Forms superior parathyroid glands
Forms utlimobranchial body

Derivatives of pharyngeal clefts/grooves:


Initially, four clefts exist; however, only one gives rise to a definite structure in
adults.
1st pharyngeal cleft Penetrates underlying mesenchyme and forms EAM. The
bottom of EAM forms lateral aspect of tympanic cavity.
2nd pharyngeal cleft
Undergoes active proliferation and overlaps remaining
clefts. It merges with ectoderm of lower neck such that the
remaining clefts lose contact with outside. Temporarily, the
clefts form an ectodermally lined cavity, the cervical sinus,
but this disappears during development.

(Q.10) Damage to the parasagittal region and falx cerebri will most likely result in which of the
following neurologic deficits?
Altered taste
(a)
Leg paralysis
(b)
Loss of facial sensation
(c)
Ptosis
(d)
Your Response :
Correct Answer
B
:
Leg paralysis
Exp:
A meningioma of the parasagittal region and the falx cerebri would be located at
the top of the brain, near the midline. In this position, it could compress the
sensory or motor cortex supplying the lower extremities. The falx cerebri is a
fold of dura mater that projects between the cerebral hemispheres in the
longitudinal tissues. Its interior portions attach anteriorly to the crista galli and
posteriorly to the internal occipital crest.
Taste is supplied by cranial nerves VII, IX, and X. These nerves arise from the
brainstem.
Facial sensation is supplied by cranial nerve V, the nuclei of which are in the
brainstem. Furthermore, the area of the sensory cortex that subserves the face is
on the lateral aspect of the cortex and would not be affected by a tumor in the
parasagittal region.
Ptosis can be caused by a deficit in cranial nerve III, which arises from the
brainstem.
Unilateral deafness suggests damage to cranial nerve VIII, which arises from the
brainstem.

(Q.11) Corneal stroma develop from


Mesoderm
(a)
Endoderm
(b)
Ectoderm
(c)
Neural crest cells
(d)
Your Response :
Correct Answer
D
:
. Neural crest cells
Exp:
Corneal stroma is derived from the mesenchyme of neural crest cells (NCCs).
NCCs form the mesenchyme of the eyeball and contribute to
Sclera and choroid of the eyeball
Stroma of the cornea, iris and ciliary body
Ciliaris muscle of the ciliary body
In the past, most of these eyeball components were described under the
derivatives of mesoderm.
Now, it is established that Neural crest cells (NCCs) form the mesenchyme of the
head and neck and contribute to most of the eyeball.
Cornea:
Components
Surface epithelium
Stroma
Inner epithelium
(Endothelium)

Source
Surface ectoderm
NCCs
NCCs

Endoderm contributes nothing for the eye.


Ectoderm derivatives:
Surface ectoderm
Surface epithelium of cornea
Eye lens

Neural plate ectoderm


Retina (& optic nerve)
Epithelium of ciliary body and the iris
Sphincter & dilator pupillae muscles of the iris

Neural crest cell derivatives:


Head and Neck
Thorax
Abdomen
Most of the skull A- P septum of heart
Chromaffin cells of Adrenal
bones
medulla
Teeth (Odontoblast) Tunica media of great Auerbach&Meissner ganglia in the
vessels
gut tube
Parafollicular C
cells of thyroid
gland
References:

Pg. 702; Gray's Anatomy (40th Ed.)


Pg. 429; The developing human Moore &Persaud (8th Ed.)

(Q.12) A sharp instrument passing through the superior orbital fissure would most likely sever which
of the following structures?
Abducent nerve
(a)
Facial nerve
(b)
Mandibular nerve
(c)
Middle meningeal artery
(d)
Your Response :
Correct Answer
A
:
Abducent nerve
Exp:
A good way to remember what passes through the superior oribital fissure is that
everything that innervates the eye,other than the optic nerve, passes through this
fissure. This incudes the oculomotor nerve (CN III), the trochlear nerve (CN IV),
the ophthalmic nerve (V1), and the abducens nerve (CN VI).
The facial nerve (CN VII; ) passes through the internal auditory meatus.
The mandibular nerve (V3; ) passes through the foramen ovale.
The maxillary nerve (V2; ) passes through the foramen rotundum.
The middle meningeal artery passes through the foramen spinosum.
Openings Into the Orbital Cavity
OPENING
ANATOMICAL DETAILS
Lies anteriorly About one-sixth of the eye is
Orbital opening
exposed; the remainder is protected by the walls
of the orbit.
The supraorbital notch is situated on the superior
Supraorbital notch
orbital margin. It transmits the supraorbital nerve
/Foramen
and blood vessels.
Situated on the floor of the orbit in the orbital
Infraorbital groove and
plate of the maxilla they transmit the infraorbital
canal
nerve (a continuation of the maxillary nerve) and
blood vessels.
Located anteriorly on the medial wall; it
Nasolacrimal canal
communicates with the inferior meatus of the
nose. It transmits the nasolacrimal duct.
Located posteriorly between the maxilla and the
Inferior orbital fissure
greater wing of the sphenoid; it communicates
with the pterygopalatine fossa. It transmits the
maxillary nerve and its zygomatic branch, the
inferior ophthalmic vein, and sympathetic nerves.
Located posteriorly between the greater and lesser
Superior orbital fissure
wings of the sphenoid it communicates with the
middle cranial fossa. It transmits the lacrimal
nerve, the frontal nerve, the trochlear nerve, the
oculomotor nerve (upper and lower divisions), the

Optic canal

abducent nerve, the nasociliary nerve, and the


superior ophthalmic vein.
Located posteriorly in the lesser wing of the
sphenoid it communicates with the middle cranial
fossa. It transmits the optic nerve and the
ophthalmic artery.

(Q.13) A knife wound to the neck damages the posterior cord of the brachial plexus. Which of the
following muscles would be most likely be paralyzed?
Deltoid
(a)
Flexor carpi ulnaris
(b)
Flexor digitorum superficialis
(c)
Flexor pollicis brevis
(d)
Your Response :
Correct Answer
A
:
. Deltoid
Exp:
The posterior cord supplies the axillary and radial nerves. Of the muscles listed,
only the deltoid is supplied by one of these two nerves, specifically the axillary
nerve. The deltoid originates from the clavicle and scapula. It inserts into the
deltoid tuberosity of the humerus. This muscle is responsible for abduction of the
arm.
The flexor carpi ulnaris is supplied by the ulnar nerve.
The flexor digitorum superficialis, the flexor pollicis brevis, and the palmaris
longus are supplied by the median nerve.

(Q.14) During the process of meiosis, a single homologous chromosome pair fails to separate during the
first meiotic division. This failure would be most likely to produce which of the following
conditions if fertilization occurs and an embryo later develops?
Balanced translocation
(a)
Triploidy
(b)
Trisomy
(c)
Unbalanced translocation
(d)
Your Response :
Correct Answer
C
:
Trisomy
Exp:
Meiosis is cell division that produces gametes with half of the normal somatic
chromosome complement. The process described is nondisjunction, which will
cause one daughter cell to have 24 chromosomes, while the other will have 22
chromosomes. When a gamete with the normal 23 chromosomes combines at
fertilization with a gamete with 22 or 24 chromosomes, the embryo will have 47
chromosomes (trisomy) or 45 chromosomes (monosomy). Nondisjunction can
occur in either the first or second meiotic division.
Balanced translocation occurs when non-homologous chromosomes exchange
genetic material in such a way that no critical genetic material is lost.
Triploidy is the term used when a cell has 69 chromosomes (3N or 3 sets), and

can occur in tumors or when an egg is fertilized by two sperm.


An unbalanced translocation occurs when non-homologous chromosomes
exchange genetic material with a net loss or gain of critical genetic material.
Uniploidy is the state of having 23 chromosomes, seen normally in sperm and
eggs.

(Q.15) The superior ophthalmic vein directly communicates with which of the following dural venous
sinuses?
Cavernous sinus
(a)
Occipital sinus
(b)
Sigmoid sinus
(c)
Superior petrosal sinus
(d)
Your Response :
Correct Answer
A
:
Cavernous sinus
Exp:
The anterior continuation of the cavernous sinus, the superior ophthalmic vein,
passes through the superior orbital fissure to enter the orbit. Veins of the face
communicate with the superior ophthalmic vein. Because of the absence of
valves in emissary veins, venous flow may occur in either direction. Cutaneous
infections may be carried into the cavernous sinus and result in a cavernous sinus
infection, which may lead to an infected cavernous sinus thrombosis. The
cavernous sinus is lateral to the pituitary gland and contains portions of cranial
nerves III, IV, V1, V2, and VI, and the internal carotid artery.
The occipital sinus is at the base of the falx cerebelli in the posterior cranial
fossa. It drains into the confluence of sinuses.
The sigmoid sinus is the anterior continuation of the transverse sinus in the
middle cranial fossa. The sigmoid sinus passes through the jugular foramen and
drains into the internal jugular vein.
The superior petrosal sinus is at the apex of the petrous portion of the temporal
bone and is a posterior continuation of the cavernous sinus. The superior petrosal
sinus connects the cavernous sinus with the sigmoid sinus.
The straight sinus is at the intersection of the falx cerebri and the falx cerebelli in
the posterior cranial fossa. The straight sinus connects the inferior sagittal sinus
with the confluence of sinuses.

(Q.16) In preparation for a procedure to remove the fingernail on an index finger, the physician would
most likely anesthetize a branch of the
Anterior interosseus nerve
(a)
Median nerve
(b)
Musculocutaneous nerve
(c)
Radial nerve
(d)
Your Response :
Correct Answer
B
:
Median nerve
Exp:

The median nerve supplies the surface of the lateral palm, the palmar surface of
the first three digits, and the distaldorsal surface of the index and middle fingers
(including the nail beds).
Therefore, prior to performing surgery in this area, it is essential to anesthetize a
branch of this nerve (possibly a proper digital branch) to eliminate pain sensation
around the nail bed of the index finger.
The median nerve of the branchial plexus distributes to the flexor muscles on the
forearm (flexor carpi radialis and palmaris longus), the pronators (p. quadratus
and p. teres), digital flexors, and skin over the lateral surface of the hand.
Neither the anterior interosseus nor the musculocutaneous nerves supplies the
hand. The anterior interosseous nerve supplies the flexor pollicis longus, the
lateral half of flexor digitorum profundus, and pronator quadratus.
The musculocutaneous nerve supplies the coracobrachialis, biceps, and most of
the brachialis muscle, then becomes the lateral cutaneous nerve of the forearm.
The radial nerve supplies skin on the radial side of the dorsal surface of the hand,
but not the fingertips.
The ulnar nerve supplies the palmar and dorsal surfaces of the medial hand,
including the palmar and dorsal surfaces of the fourth and fifth digits.

(Q.17) Filumterminale is chiefly composed of


Piamater
(a)
Duramater
(b)
Fibrous tissue
(c)
Neural tissue
(d)
Your Response :
Correct Answer
C
:
Fibrous tissue
Exp:
The filumterminale is a delicate filament, about 20 cm. in length, prolonged
downward from the apex of the conus medullaris.
It consists of two parts, an upper and a lower.
The upper part, or filumterminaleinternum, measures about 15 cm. in length and
reaches as far as the lower border of the second sacral vertebra. It is contained
within the tubular sheath of dura mater, and is surrounded by the nerves forming
the cauda equina, from which it can be readily recognized by its bluish-white
color.
The lower part, or filumterminaleexternum, is closely invested by, and is
adherent to, the dura mater; it extends downward from the apex of the tubular
sheath and is attached to the back of the first segment of the coccyx.
The filumterminaleconsists mainly of fibrous tissue, continuous above with that
of the pia mater.

Adhering to its outer surface, however, are a few strands of nerve fibers which
probably represent rudimentary second and third coccygeal nerves; further, the
central canal of the medulla spinalis extends downward into it for 5 or 6 cm.
References:
1. Pg. 749; Gray's Anatomy (40th Ed.)

(Q.18) In a post polio case, ilio-tibial tract contracture is likely to result in:
Extension at the hip and knee
(a)
Extension at the hip
(b)
Flexion at the hip and the knee
(c)
Extension at the knee
(d)
Your Response :
Correct Answer
C
:
Flexion at the hip and the knee
Exp:
Flexion at the hip and the knee
Explanation:
-Contracture of iliotibial tract in polio leads to flexion at the hip & the knee joint.
-It also leads to abduction and lateral rotation at the hip joint and the knee joint.
Iliotibialband
Attachments
Superiorly : Anterior prong over TFL, middle prong over Gluteus Medius,
posterior prong over Gluteus Maximus
Distally contributes to lateral patellar retinaculum and attaches to lateral tibial
plateau and fibular head
Plane : Anterolateral to hip joint; anterolateral to knee joint (while in
extension)
Deformities from ITB contracture
Flexion and abduction of hip (+ ER from maintenance of malposture)
Valgus deformity of knee
Knee flexion from spasm and myostatic contracture of short head of biceps,
through attachments to lateral intermuscular septum and fascia lata
ER tibia, if left unchecked may lead to posterolateral subluxation of knee
Positional pes varus from an ill-fitting orthosis

Pelvic obliquity due to weight bearing on abducted hip, subluxation of


contralateral hip
Scoliosis
Treatment of ITB contracture
Nonoperative : Passive exercises to maintain ROM, stretching to correct minimal
contracture
Operative release of ITB : Excise ~ 7 cm of ITB, fascia lata over vastus lateralis
and intermuscular septum (Younts procedure)
Lateral rotatory subluxation of knee : Fractional lengthening of biceps femoris
with Z-lengthening of the lateral collateral ligament

(Q.19) If a patient has a drooping right eyelid and a dilated right pupil, which of the following neural
structures is most likely affected?
Cervical sympathetic chain
(a)
Facial nerve
(b)
Oculomotor nerve
(c)
Superior cervical ganglion
(d)
Your Response :
Correct Answer
C
:
Oculomotor nerve
Exp:
The oculomotor nerve innervates the levator palpebrae superioris, which elevates
the eyelid. This nerve also innervates the inferior oblique muscles, as well as the
superior, inferior, and medial rectus muscles.
The oculomotor nerve also contains preganglionic parasympathetic fibers that
synapse, in the ciliary ganglion, on postganglionic parasympathetic nerve fibers
that innervate the sphincter pupillae muscle, which constricts the pupil.
A lesion of the oculomotor nerve may therefore result in both drooping of the
eyelid (ptosis) and dilation of the pupil (mydriasis).
The cervical sympathetic chain contains preganglionic sympathetic nerve fibers,
arising from the upper thoracic spinal cord, which ascend to the cervical
sympathetic ganglia.
A lesion of these nerves may result in Horner's syndrome, which includes a
ptosis and miosis (pupillary constriction) and, often, anhidrosis (lack of
sweating).
The facial nerve innervates the muscles of facial expression, including the
orbicularis oculi muscle. A lesion of this nerve may therefore result in the
inability to close the eye.
The superior cervical ganglion contains the cell bodies of postganglionic
sympathetic nerves that innervate structures in the head. A lesion of this structure
will cause Horner's syndrome.

The trigeminal nerve provides sensory innervation to much of the head. A lesion
of this nerve may interfere with the corneal blink reflex.

(Q.20) An injection to anesthetize pain from a fracture of the seventh rib should be given in what
area?

(a)
(b)
(c)
(d)

Seventh intercostal space immediately below the seventh rib in the


midclavicular line
Seventh intercostal space immediately below the seventh rib just lateral to the
angle of the rib
Seventh intercostal space immediately below the seventh rib just medial to the
angle of the rib
Sixth intercostal space immediately above the seventh rib in the midclavicular
line

Your Response :
Correct Answer
B
:
Seventh intercostal space immediately below the seventh rib just lateral to
Exp:
the angle of the rib
The seventh intercostal nerve (the anterior ramus of the seventh thoracic spinal
nerve) innervates the seventh rib. After passing through the intervertebral
foramen between the seventh and eighth thoracic vertebrae, the nerve lies in the
seventh intercostal space.
After passing the angle of the rib, it occupies a position along the lower border of
the rib, in the costal groove. Use of a local anesthetic at this point will
anesthetize the rib.
By the time the intercostal nerve has reached the midclavicular line, it has
already innervated most of the rib. Use of an anesthetic at this point would not be
effective.
While the intercostal nerve is in the intercostal space medial to the angle of the
rib , it is not along the lower border of the rib.
The intercostal nerve does not lie along the upper border of the rib. Thus,
injection at these sites would not be effective.

(Q.21) The nucleus that lies immediately medial (and deep) to the uncus is the:
Amygdala
(a)
Caudate nucleus
(b)
Claustrum
(c)
Hippocampus
(d)
Your Response :
Correct Answer
A
:
The uncus, which is the medial protrusion of the parahippocampal gyrus, is an
Exp:
external structure seen on the ventralsurface of the temporal lobe. The amygdala
is a collection of nuclei that lies directly beneath the uncus.
The caudate nucleus is a deep nuclear structure that lies lateral to the lateral
ventricles.

The claustrum is a thin and elongated nucleus that lies just medial to the insular
cortex.
The hippocampus is a nuclear structure that lies in the interior of the
parahippocampal gyrus.
The putamen is a nuclear structure that resides lateral to the caudate and medial
to the claustrum.

(Q.22) The smooth part of the right atrium derives from which of the following embryonic structures?
Bulbus cordis
(a)
Primitive atrium
(b)
Primitive ventricle
(c)
Sinus venosus
(d)
Your Response :
Correct Answer
:
Ans. D. Amygdala
Exp:
The smooth part of the right atrium (the sinus venarum) is derived from the sinus
venosus. The coronary sinus and the oblique vein of the left atrium also derive
from the sinus venosus.
The bulbus cordis gives rise to the smooth part of the right ventricle (conus
arteriosus) and the smooth part of the left ventricle (aortic vestibule).
The primitive atrium gives rise to the trabeculated part of the right and left atria.
The primitive ventricle gives rise to the trabeculated part of the right and left
ventricles.
The truncus arteriosus gives rise to the proximal part of the aorta and the
proximal part of the pulmonary artery. COMMON ANOMALIES OF
CARDIAC DEVELOPMENT
Embryological precursor Abnormality
Sinus venosus and
tributaries
Interartrial septum and
atria

Anomalous systemic venous drainage


Persistent foramen ovale
Ostium secundum defect
Sinus venous defect
Ostium primum defect

Endocardial cushions

Ostium primum defect


Endocardial atrioventricular defect
Tricuspid atresia/stenosis
Ebstein's anomaly
Cor triatrium

Interventricular septum and


ventricles

Ventricular septal defect (membranous,


muscular)
Common ventricle
Hypoplastic left ventricle

Uhl's dysplastic right ventricle


Bulbus cordis and
ventricular outflow

Pulmonary stenosis (valve/infundibular)


Aortic stenosis (valve/subaortic)
Tetralogy of Fallot
Ventriculo-arterial discordance (uncorrected
and corrected transposition)

Truncus arteriosus

Common arterial trunk, persistent truncus


ateriosus
Aortopulmonary window
Uncorrected transposition (discordance
ventricles and great arteries)
Congenitally corrected transposition of great
arteries
Pulmonary arterial trunk atresia

Branchial (aortic) arches

Double aortic arch: aortic rings


Right aortic arch either isolated or with
congenital heart disease
Aberrant right subclavian artery
Interruption or absence of right or left
pulmonary artery
Patent arterial duct (ductus arteriosus)
Supravalvular aortic stenosis
Coarctation of the aorta

(Q.23) Muscular component of dorsal aorta develops from


Paraxial mesoderm
(a)
Intermediate mesoderm
(b)
neural crest
(c)
Lateral plate mesoderm
(d)
Your Response :
Correct Answer
D
:
Lateral plate mesoderm
Exp:
Smooth muscles of all blood vessels, GIT and respiratory tract are derived from
lateral plate mesoderm.

(Q.24) Following a surgical procedure on the right side of the neck, a patient can no longer raise his
right arm above the horizontal position. The patient also cannot shrug his right shoulder. Which
of the following nerves was injured?
Axillary nerve
(a)
Great auricular nerve
(b)
Greater occipital nerve
(c)
Spinal accessory nerve
(d)
Your Response :
Correct Answer
D
:
Spinal accessory nerve
Exp:

The spinal accessory nerve crosses the posterior triangle of the neck immediately
deep to the investing fascia of the neck. This nerve innervates the trapezius
muscle, which is responsible for upward rotation and elevation of the scapula.
A lesion of this nerve in the posterior triangle leads to paralysis of the trapezius.
Without the ability to upwardly rotate the scapula, abduction and flexion of the
arm above the horizontal plane is not possible. Also, shrugging of the shoulder is
impaired with paralysis of the trapezius muscle.
The axillary nerve does not pass through the neck. It is a branch of the brachial
plexus, and it leaves the axilla to innervate the deltoid and teres minor muscles.
The great auricular nerve and transverse cervical nerve are branches of the
cervical plexus, which provide cutaneous innervation to the skin of the neck. No
muscles are innervated by these nerves.
The greater occipital nerve is the dorsal ramus of the second cervical spinal
nerve. It provides cutaneous innervation to the skin of the back of the head. No
muscles are innervated by this nerve.

(Q.25) A woman has a fracture of the left tenth and eleventh ribs. The organ most likely to get injured
by these fractured ribs is:
Descending colon
(a)
Jejunum
(b)
Left adrenal gland
(c)
Spleen
(d)
Your Response :
Correct Answer
D
:
Spleen
Exp:
The spleen is a soft, friable organ with a thin capsule and is subject to injury
upon trauma to the left side of the abdomen. It is located in the upper left
quadrant of the abdomen, deep to the left ninth, tenth, and eleventh ribs. It is the
most commonly injured organ in the abdomen.
The adult spleen contains the largest number of lymphoid tissues in the human
body.
The descending colon lies in a retroperitoneal position on the left side of the
posterior abdominal wall. The descending colon begins at the splenic flexure
immediately inferior to the spleen.
The jejunum is a peritoneal structure suspended by a long mesentery. It is located
primarily in the upper left quadrant of the abdomen. Its long mesentery allows
the jejunum to be highly mobile and thus is not likely to be injured by trauma to
the body wall.
The left adrenal gland is a retroperitoneal structure that lies near the upper pole
of the left kidney. It is embedded within fat and is thus well protected from
injury.
The left kidney is a retroperitoneal structure that is well protected by fat.

(Q.26) Azygous vein drains


Into superior vena cava
(a)
Anterior to costochondral junction
(b)
Oesophagus
(c)
Directly into right atrium
(d)
Your Response :
Correct Answer
A
:
Into superior vena cava
Exp:
The azygous vein is formed by the confluence of the Lumbar azygous vein,
Right ascending lumbar vein and right subcostal vein.
It passes superiorly through the aortic aperture, posterior to the right
diaphragmatic crus and anterior to the twelfth vertebral body.
The azygos vein transports deoxygenated blood from the posterior walls of
thethoraxandabdomeninto thesuperior vena cavavein. The anatomy of thisblood
vesselcan be quite variable. In some rare variations for example, it also drains
thoracic veins, bronchial veins and even gonadal veins. The vein is so named
because it has no symmetrically equivalentveinon the left side of the body.
Ref. Pg. 213; B D Chaurasia (4th Ed.); Vol-1

(Q.27) All are true regarding the pudendal nerve, except:


Sensory and motor
(a)
Derived from S2, 3, 4
(b)
Comes out through the lesser sciatic foramen
(c)
Main nerve supply of pelvic organs
(d)
Your Response :
Correct Answer
D
:
Main nerve supply of pelvic organs
Exp:
Main nerve supply of pelvic organs
Pudendal nerve is a somatic nerve and is not the main supply of pelvic viscera
(D).
It is the nerve of perineum carrying root value S-2, 3 & 4 (B) and is a mixed
nerve with both the sensory and motor (A) components.
Pelvic viscera are mainly supplied by the autonomic nervous system:
sympathetic (T-10, 11, 12; L-1, 2) and parasympathetic (S-2, 3, 4).
Pudendal nerve passes through both the sciatic notches and the information given
in other choices is perfectly fine, in anatomical position.

(Q.28) Choose the correct option for following anomaly?

Persistent right anterior cardinal & right common cardinal vein


(a)
Persistent left anterior cardinal& left common cardinal vein
(b)
Persistent left subcardinal vein
(c)
Persistent right umblical vein
(d)
Your Response :
Correct Answer
B
:
Persistent left anterior cardinal & left common cardinal vein
Exp:
Persistent left anterior cardinal vein gives rise to aberrant left Superior vena Cava
which opens in coronary sinus as can be appreciated in picture.

(Q.29) Attempts to straighten out a flexed thigh cause great pain in a patient with appendicitis. This is
due to the position of the appendix near which muscle?
Adductor magnus
(a)
Biceps femoris
(b)
Gluteus maximus
(c)
Psoas major
(d)
Your Response :
Correct Answer
D
:
Psoas major
Exp:
The path of the psoas major lies in the retroperitoneum and comes close to the
appendix. Acute appendicitis can cause either infection or a sympathetic
inflammation of the psoas. This produces clinically a "positive psoas sign," in
which attempts to straighten the patient's flexed (to relieve pain) hip produce
sometimes marked exacerbation of the pain. None of the other muscles listed
pass near the appendix.
SIGNS IN APPENDICITIS :
Named after the Danish surgeonNiels Thorkild Rovsing,is a
Rovsings sign
sign ofappendicitis. If palpation of theleft lower quadrantof
a person'sabdomenincreases thepainfelt in the right lower
quadrant, the patient is said to have a positive Rovsing's

sign and may have appendicitis. In acute appendicitis,


palpation in the leftiliac fossamay produce pain in the right
iliac fossa.
Psoas sign or "Obraztsova's sign" is right lower-quadrant
Psoas sign
pain that is produced with either the passive extension of
the patient's right hip (patient lying on left side, with knee in
flexion) or by the patient's active flexion of the right hip
while supine. The pain elicited is due to inflammation of the
peritoneum overlying the iliopsoas muscles and
inflammation of the psoas muscles themselves.
Straightening out the leg causes pain because it stretches
these muscles, while flexing the hip activates the iliopsoas
and therefore also causes pain.
If an inflamed appendix is in contact with theobturator
Obturator sign
internus, spasm of the muscle (called theobturator sign) can
be demonstrated by flexing and internal rotation of the hip.
This maneuver will cause pain in thehypogastrium.
This sign, developed in and popular in southwest England,
Massouh sign
describes a firm swish of the examiners index and middle
finger across the patients abdomen fromxiphoid process of
the sternumto first the left and then the right iliac fossa. A
positive Massouh sign is a grimace of the patient upon a
right sided (and not left) sweep, because initial stage
appendicitis usually causes localised irritation of the wellinnervated peritoneum.
From the history given, the appearance of pain in the
Kochers sign
epigastric region or around the stomach at the beginning of
disease with a subsequent shift to the right iliac region.
Increased pain in the right upper quadrant with coughing.
Dunphys sign
BartomierIncreased pain on palpation at the right iliac region as
Michelson's sign
patient lies on his/her left side compared to when patient
was on supine position.
Aure-Rozanova
Increased pain on palpation with finger in rightPetit
sign
triangle.
Sitkovskiy
Increased pain in the right iliac region as patient lies on
(Rosenstein)'s sign his/her left side.
Blumberg's sign
is a nonspecific sign that is elicited during physical
examination inmedicine. It is indicative ofperitonitis.
Theabdominal wallis compressed slowly and then rapidly
released. A positive sign is indicated by presence of pain
upon removal of pressure on the abdominal wall. It is very
similar torebound tendernessand might be regarded by some
authors as the same thing, or at least a particular application
of it.
The name comes from German surgeonJacob Moritz
Blumberg.
Positive Blumberg sign or rebound tenderness is indicative
of peritonitis. This method is specially useful in diagnosing
appendicitis requiring urgent management

(Q.30) Which of the following structures does the fetal allantoic duct become in the adult?
Cloaca
(a)
Medial umbilical ligament
(b)
Urachus
(c)
Ureter

(d)
Ans C

Your Response :
Correct Answer
:
Urachus
Exp:
Theurachusis a fibrous remnant of theallantois, a canal that drains theurinary
bladderof the fetus that joins and runs within theumbilical cord.The fibrous
remnant lies in thespace of Retzius, between the transversalis fascia anteriorly
and theperitoneumposteriorly.
The vesico-urethral portion of theurogenital sinusabsorbs the ends of
theWolffian ductsand the associated ends of therenal diverticula, and these give
rise to thetrigone of the bladderand part of theprostatic urethra. The remainder of
the vesico-urethral portion forms the body of the bladder and part of the prostatic
urethra; its apex is prolonged to theumbilicusas a narrow canal, which later is
obliterated as the urachus goes on to definitively form themedian umbilical
ligament(urachus).
Failure for the lumen of the urachus to be filled in leaves a patent (open) urachus.
The telltale sign is leakage of urine through the umbilicus. A patent urachus
needs to be surgically removed. There are four anatomical causes:
Urachal cyst: there is no connection between the bladder and the umbilicus
Urachal fistula: there is free communication between the bladder and umbilicus
Urachal diverticulum[Vesicourachal diverticulum]: the bladder exhibits
outpouching[2]
Urachal sinus: the pouch opens toward the umbilicus[3]
The urachus is also subject to neoplasia.Urachal adenocarcinomais histologically
similar to adenocarcinoma of the bowel.

(Q.31) Pain of ovarian pathology is referred to


Back of thigh
(a)
Anterior thigh
(b)
Medial thigh
(c)
Gluteal region
(d)
Your Response :
Correct Answer
C.
:
Medial thigh
Exp:
Pain of ovarian pathology is referred to the periumblical (T10-T11) area (due to
sensory fibres accompanying sympathetic nerves to aortic plexus) along
cutaneous distribution of obturator nerve on inner aspect of thigh (because
obturator neurovascular bundle lie lateral to ovary in ovarian fossa).

(Q.32) A surgeon inadvertently sections the recurrent laryngeal nerve during a procedure. Which of
the following muscles would retain its innervation subsequent to this injury?
Cricothyroid
(a)

(b)
(c)
(d)

Lateral cricoarytenoid
Posterior cricoarytenoid
Thyroarytenoid

Your Response :
Correct Answer
A
:
Cricothyroid
Exp:
The recurrent laryngeal nerve is a branch of the vagus nerve, which innervates all
of the intrinsic laryngeal muscles, except for the cricothyroid muscle. The
cricothyroid is attached to the cricoid cartilage and the thyroid cartilage;
contraction of this muscle tends to stretch and adduct the vocal ligament. The
cricothyroid is innervated by the external laryngeal nerve.
The lateral cricoarytenoid muscle is innervated by the recurrent laryngeal nerve
and is attached to the cricoid cartilage and the arytenoid cartilage. Its contraction
causes adduction of the vocal ligament.
The posterior cricoarytenoid muscle is innervated by the recurrent laryngeal
nerve and is attached to the cricoid cartilage and the arytenoid cartilage. Its
contraction causes abduction of the vocal ligament.
The thyroarytenoid muscle is innervated by the recurrent laryngeal nerve and is
attached to the thyroid cartilage and the arytenoid cartilage. Its contraction
causes slackening of the vocal ligament.
The vocalismuscle is the most medial part of the thyroarytenoid muscle. It
attaches to either the thyroid cartilage and the vocal ligament, or the arytenoid
cartilage and the vocal ligament. It is innervated by the recurrent laryngeal nerve.
Its contraction causes tension on segments of the vocal ligament.

(Q.33) As a result of a viral infection, a patient has swelling of the left facial nerve within the facial
canal. His face appears asymmetrical, and he C/O that saliva drips from his mouth while he is
chewing. Paralysis of which of the following muscles accounts for these symptoms?
Buccinator
(a)
Masseter
(b)
Palatoglossus
(c)
Palatopharyngeus
(d)
Your Response :
Correct Answer
A
:
Buccinator
Exp:
Compression of the facial nerve within the facial canal may result in facial palsy
(Bell's palsy).
Because the muscles on one side of the face are paralyzed, the face appears
asymmetrical. The buccinator muscle, which is located within the cheek and is
innervated by the facial nerve, functions to hold food against the teeth while it is
being chewed. Paralysis of this muscle can result in food and saliva
accumulating between the teeth and the cheek. The buccinator originates from
the alveolar processes of the maxilla and mandible. It inserts into the fibers of the
orbicularis oris.
The masseter and temporalis muscles are innervated by the mandibular division
of the trigeminal nerve.

These muscles of mastication function to elevate the mandible.


The palatoglossus and palatopharyngeus muscles are innervated by the vagus
nerve. The palatoglossus, with its mucosal covering, forms the palatoglossal fold
(anterior pillar of the fauces), immediately anterior to the palatine tonsil.
This muscle functions to draw the tongue and soft palate closer together, as
occurs during swallowing. The palatopharyngeus, with its mucosal covering,
forms the palatopharyngeal fold (posterior pillar of the fauces), which is
immediately posterior to the palatine tonsil. This muscle causes elevation of the
pharynx, as occurs during swallowing.

(Q.34) Biopsy of the lower esophagus in a patient with chronic reflux demonstrates epithelial
metaplasi(a.) Which of the following cell types was most likely observed in the involved areas?
Ciliated columnar epithelium
(a)
Cuboidal epithelium
(b)
Keratinizing squamous epithelium
(c)
Nonciliated columnar epithelium
(d)
Your Response :
Correct Answer
D
:
Nonciliated columnar epithelium
Exp:
The medical condition is Barrett's esophagus, in which the normally
nonkeratinizing squamous epithelium of the esophagus undergoes metaplasia to
gastric or intestinal-like epithelium composed of nonciliated columnar epithelial
cells. Barrett's esophagus typically develops in the setting of chronic
gastroesophageal reflux and significantly increases the risk of later development
of adenocarcinoma of the distal esophagus.
Ciliated columnar epithelium is found in the respiratory tract.
Cuboidal epithelium is found in the kidney, peritoneal lining, and pleural lining.
Keratinizing squamous epithelium is found in skin.
Nonkeratinizing squamous epithelium, in addition to being the normal
epithelium of the esophagus, is found in mouth, nose, and vagina.

(Q.35) Despite blockage of the celiac trunk, the organs receiving their blood supply from the trunk
continue to operate normally. This is due to anastomoses between which vessels?
Left gastroepiploic artery and right gastroepiploic artery
(a)
Left gastroepiploic artery and right hepatic artery
(b)
Proper hepatic artery and gastroduodenal artery
(c)
Superior pancreaticoduodenal artery and inferior pancreaticoduodenal artery
(d)
Your Response :
Correct Answer
D.
:
Superior pancreaticoduodenal artery and inferior pancreaticoduodenal
Exp:
artery
The superior pancreaticoduodenal artery is a branch of the gastroduodenal artery,

which is a branch of the common hepatic artery, itself a branch of the celiac
trunk.
The inferior pancreaticoduodenal artery is a branch of the superior mesenteric
artery. Occlusion of the celiac trunk would allow blood from the superior
mesenteric artery to reach the branches of the celiac trunk via the connections
between the superior and inferior pancreaticoduodenal arteries.
The left gastric and right gastric arteries both receive their blood from the celiac
trunk. The left gastric artery is a direct branch of the celiac trunk. The right
gastric artery is usually a branch of the proper hepatic artery, which is a branch
of the common hepatic artery (a branch of the celiac trunk).
The left and right gastroepiploic arteries both receive their blood supply from the
celiac trunk. The left gastroepiploic artery is a branch of the splenic artery, which
is a branch of the celiac trunk. The right gastroepiploic artery is a branch of the
gastroduodenal artery, which is a branch of the common hepatic artery (a branch
of the celiac trunk).
The proper hepatic and gastroduodenal arteries are branches of the common
hepatic artery, which is a branch of the celiac trunk.
The right colic and middle colic arteries are both branches of the superior
mesenteric artery.

(Q.36) Which structure traverses through the arrow marked foramen in the given picture?

Maxillary nerve
(a)
Mandibular nerve
(b)
Ophthalmic
artery
(c)
Ophthalmic nerve
(d)
Your Response :
Correct Answer
A
:
Maxillary nerve
Exp:

As shown in the picture above the arrow corresponds to Foramen rotundum. The
structure passing through foramen rotundum is Maxillary Nerve (A)
Summary of the More Important Openings in the Base of the Skull and the
Structures That Pass Through Them
Opening in
Bone of Skull
Skull
Anterior Cranial Fossa
Perforations in Ethmoid
cribriform plate
Middle Cranial Fossa
Optic canal
Lesser wing of
sphenoid
Superior orbital Between lesser and
fissure
greater wings
of sphenoid
Foramen
Greater wing of
rotundum
sphenoid
Foramen ovale Greater wing of
sphenoid
Foramen
Greater wing of
spinosum
sphenoid
Foramen
Between petrous part
lacerum
of temporal and
sphenoid
Posterior Cranial Fossa
Foramen
Occipital
magnum
Hypoglossal
Occipital
canal
Jugular foramen Between petrous part
of temporal and
condylar part of
occipital
Internal acoustic Petrous part of

Structures Transmitted

Olfactory nerves

Optic nerve, ophthalmic artery


Lacrimal, frontal, trochlear,
oculomotor, nasociliary, and
abducent nerves; superior ophthalmic
vein
Maxillary division of the trigeminal
nerve
Mandibular division of the trigeminal
nerve, lesser petrosal nerve
Middle meningeal artery
Internal carotid artery

Medulla oblongata, spinal part of


accessory nerve, and right and left
vertebral arteries
Hypoglossal nerve
Glossopharyngeal, vagus, and
accessory nerves; sigmoid sinus
becomes internal jugular vein
Vestibulocochlear and facial nerves

meatus

temporal

(Q.37) Which of the following locations in the embryo later forms the dorsal horn of the spinal cord?
Alar plate
(a)
Basal plate
(b)
Neural crest
(c)
Rostral end of neural tube
(d)
Your Response :
Correct Answer
A
:
Alar plate
Exp:
The spinal cord arises from the caudal end of the neural tube. During
development, an alar and a basal plate is formed, separated by a longitudinal
groove called the sulcus limitans .
The alar plate forms the dorsal (posterior) part of the spinal cord and becomes
the sensory or afferent portion of the cord.
The basal plate (choiceB) is the ventral (anterior) part of the cord and becomes
the motor, or efferent, portion of the spinal cord and therefore would contain
anterior horn cells.
The neural crest develops into multipolar ganglion cells of autonomic ganglia,
pseudounipolar cells of spinal and cranial nerve ganglia, leptomeningeal cells,
Schwann cells, melanocytes, chromaffin cells of the adrenal medulla, and
odontoblasts.
The brain forms from the rostral end of the neural tube .

(Q.38) Umbilical arteries arise from which fetal vessels?


Aorta
(a)
Carotid arteries
(b)
Ductus arteriosus
(c)
Iliac arteries
(d)
Your Response :
Correct Answer
D
:
Iliac arteries
Exp:
The paired umbilical arteries arise from the iliac arteries.
They supply unoxygenated fetal blood to the placenta. The single umbilical vein
takes the newly oxygenated fetal blood from the placenta to the liver and then to
the inferior vena cava via the ductus venosus.
Near the level of vertebra L4, the terminal segment of the abdominal aorta
divides to form the right and left common iliac arteries. These arteries carry
blood to the pelvis and lower limbs.
As these arteries travel along the inner surface, they descend behind the cecum
and sigmoid colon, where each divides to form the internal iliac artery and

external iliac artery.

(Q.39) All developed up to the adult size at birth except


Mastoid Antrum
(a)
Tympanic cavity
(b)
Orbital cavity
(c)
Ear ossicle
(d)
Your Response :
Correct Answer
C
:
Orbital cavity
Exp:
Tympanic cavity, mastoid (tympanic) antrum, ear ossicles and the Internal ear
are of adult size in the foetal skull.
At birth the orbits appear relatively large. Growth of the orbits is complete by
7th year.
Mastoid process is absent at birth - appears during the later part of 2nd year and
the mastoid air cells appear during 6th year.
Size of mastoid antrum is of adult size at birth but the thickness of the wall
(temporal bone) over it keeps increasing with age adding one millimeter each
year.
At birth, Mastoid antrum lies 3 mm deep from the surface, whereas, it is 15 mm
deep at the age of 12 years.
Paranasal sinuses are rudimentary at birth. The maxillary sinus is the 1st PNS to
develop and is identifiable at birth.
No diploe (& diploic veins) in the skull at birth they appear by 4th year of age.
References:

1. Pg. 416; Gray's Anatomy (40th Ed.)

2. Pg. 29,30; B D Chaurasia (4th Ed.); Vol-3


(11th Ed.)

3. Pg. 35; Lasts Anatomy

(Q.40) Which of the following sites contains striated muscle that is not under voluntary control?
Bladder
(a)
Colon
(b)
Esophagus
(c)
Gallbladder
(d)
Your Response :
Correct Answer
C
:
Esophagus
Exp:
Striated (skeletal) muscle not under voluntary control is an unusual feature of the
upper and middle thirds of the esophagus. The middle third of the esophagus
contains roughly half striated and half smooth muscle; the lower third contains
only smooth muscle. All the other structures listed in the answer choices contain
smooth muscle.

As a side note, cardiac muscle cells do not rely on nerve activity to start a
contraction. Instead, specialized pacemaker cells establish a regular rate of
contraction. Because these pacemaker cells regulate the heart and the central
nervous system does not, cardiac muscle is also considered striated, involuntary
muscle.

(Q.41) Structure injured while resecting the free edge of lesser omentumCystic duct
(a)
Portal vein
(b)
Left gastric artery
(c)
Right gastroepiploic artery
(d)
Your Response :
Correct Answer
B
:
Portal vein
Exp:
Lesser omentum: - It extends from liver to stomach (lesser curvature) and
duodenum (first 2 cm). The part of lesser omentum from liver to stomach is
called hepatogastric ligament, and part from liver to duodenum is called
hepatoduodenal ligament. Hepatogastric ligament forms anterior free wall of
lesser sac (omental bursa) and contains right and left gastric vessels, branches of
gastric nerves and lymph vessels. Hepatoduodenal ligament forms right free
margin, contains portal vein, hepatic artery proper, bile duct, lymphatics and
hepatic plexus of nerves. Lesser omentum develops from ventral mesogastrium
(dorsal part).

(Q.42) Which structure is NOT in the posterior wall of the omental bursa?
Body of the pancreas.
(a)
Celiac artery (trunk).
(b)
Upper pole of the left kidney.
(c)
Transverse colon and its mesocolon
(d)
Your Response :
Correct Answer
D
:
Transverse colon and its mesocolon
Exp:
Omental bursa is demarcated anteriorly by the caudate lobe of the liver, the
stomach and lesser omentum. Posteriorly it is marked by the pancreas. Its left
lateral margin is made by the left kidney and adrenal gland. Its boundary on the
right is made by the epiploic foramen and lesser omentum. If these structures
rupture they may leak into the lesser sac. For the stomach, which lies anterior to
the omental bursa, the rupture must be on the posterior side; if it were anteriorly
located, the leak would collect in the greater sac.
The lesser sac is embryologically formed from an infolding of the greater
omentum. The open end of the infolding, known as the epiploic foramen, is
usually proximal to the stomach

(Q.43) Which of the following respiratory system components is derived from neural crest?
Endothelial cells
(a)
Epithelium of primary bronchi
(b)
Laryngeal cartilage
(c)

Tracheal glands
(d)
Your Response :
Correct Answer
C
:
Laryngeal cartilage
Exp:
Laryngeal cartilages are derived from neural crest.
The larynx is composed of three cartilages that form the "body of the larynx": the
thyroid cartilage, the cricoid cartilage, and the epiglottis.
The larynx also contains three pairs of smaller hyaline cartilages: the arytenoid,
corniculate, and cuneiform cartilages.
The endothelial cells , in the simple squamous epithelium that lines the
pulmonary capillaries, are derived from visceral mesoderm.
The epithelial lining of primary bronchi is derived from endoderm.
Tracheal glands and epithelium both derive from endoderm.
Type I pneumocytes are derived from endoderm.

(Q.44) The extraocular muscles are derived from which of the following structures?
Branchial arches
(a)
Optic cup ectoderm
(b)
Somites
(c)
Somitomeres
(d)
Your Response :
Correct Answer
D
:
Somitomeres
Exp:
The somitomeres are specialized masses of mesoderm found in the head region
that give rise to the muscles of the head.
The extraocular muscles are derived from somitomeres 1, 2, 3, and 5.
The major development of the eye takes place between week 3 and week 10 and
involves ectoderm, neural crest cells, and mesenchyme.
The neural tube ectoderm gives rise to the retina, the iris and ciliary body
epithelia, the optic nerve, the smooth muscles of the iris, and some of the
vitreous humor. Surface ectoderm gives rise to the lens, the conjunctival and
corneal epithelia, the eyelids, and the lacrimal apparatus.
The remaining ocular structures form from mesenchyme. On or about day 22,
two small grooves develop on each side of the developing forebrain in the neural
folds. They are called optic grooves, or optic sulci. As the neural tube closes,
these grooves become outpocketings and are now called optic vesicles.
The optic vesicles extend from the forebrain toward the surface ectoderm
through the adjacent mesenchyme. As the optic vesicles grow toward the

ectoderm, their connections to the forebrain become attenuated to form optic


stalks, which will eventually become the optic nerves.
The portion of each optic vesicle that interacts with the surface ectoderm induces
that area of the ectoderm to form a thickening called the lens placode (a
precursor of the lens).
The lens placode invaginates to become a lens pit, which soon forms a complete
circle that pinches off from the surface ectoderm to become the lens vesicle. At
the same time the lens vesicle is forming, the optic vesicle also invaginates to
form a double-layered structure called the optic cup.
So at this point we see a goblet-shaped optic cup with the lens vesicle floating in
its open end The developing optic vesicle and stalk have a groove on their
inferior surfaces called the optic, or choroidal, fissure, through which blood
vessels gain access to the optic cup as well as the lens vesicle.
The blood vessels are the hyaloid artery, a branch of the ophthalmic artery, and
its accompanying vein. The choroidal fissure will eventually fuse, completing the
eye wall inferiorly and enclosing the vessels in a canal in the optic stalk. When
the lens matures later on in fetal life, the distal end of the hyaloid artery will
disintegrate and its proximal end will persist as the central retinal artery.
Development of the Retina
The two layers of the optic cup will further differentiate into the retina of the
mature eye. The two layers are unequal in size - the outer one is thinner than the
inner one.
The optic cup can be divided into two portions, the anterior 1/5 (rim) and the
posterior 4/5.
The rim area will ultimately form the iris and ciliary body, and the posterior 4/5
will form the retina (Figure 1).
The outer layer of the posterior 4/5 will become the pigment layer of the retina,
and the inner one will become the neural retina. These two layers are separated
by the intraretinal space
The development of the retinas pigment layer is very straightforward, with the
appearance of melanin granules in the cells of this layer at around 4 1/2 weeks.
Slightly later, at about 6 weeks, the cells in the posterior aspect of the inner layer
of the optic cup begin a more complicated process. The cells immediately
adjacent to the intraretinal space begin to differentiate into
the photoreceptors (rods and cones).
The next layer of cells will become the Muller supporting cells and the bipolar
neurons, and the innermost superficial layer will develop into the axons of
the ganglion cells, the ones that will make up the optic nerve.
This means light actually passes through the neuronal layers before reaching the
rods and cones. The ganglion cell fibers gradually fill in the lumen of the optic
stalk as it becomes the optic nerve.
By eight months, all the layers of the retina that you will see in your Histology
course are recognizable. But maturation of the photoreceptors continues after

birth, which in part explains why a babys visual acuity improves as he or she
grows.
Development of the Lens
At about the same time as the pigmented layer of the retina is developing, the
cells of the posterior part of the lens vesicle transform into elongated,
slender primary lens fibers. These new cells fill in the previously hollow
structure. About four weeks later, more lens fibers develop, this time from the
anterior wall of the lens vesicle (secondary lens fibers).
Development of the Choroid, Sclera and Cornea
During the sixth and seventh weeks the mesenchyme that surrounds the external
surface of the optic cup condenses into two layers, an inner, pigmented, vascular
layer known as the choroid and an outer, fibrous layer called the sclera. The
mesenchyme that is anterior to the developing lens splits into two layers that
surround the newly formed anterior chamber of the eye.The inner layer is
continuous with the choroid and is called the iridopupillary membrane and the
outer layer is continuous with the sclera. The outer layer will form the substantia
propria, or stroma of the cornea. The cornea has three layers, epithelium, stroma,
and endothelium. The external corneal epithelium develops from surface
ectoderm and the endothelium forms from neural crest cells that migrate from the
rim of the optic cup. As mentioned above, the stroma is derived from the
surrounding mesenchyme. The iridopupillary membrane eventually disappears
completely, which allows communication between the anterior and posterior eye
chambers.
Development of the Iris and Ciliary Body
The anterior rim of the optic cup gives rise to the epithelium of the iris and the
ciliary body. Remember that the inner layer of the posterior 4/5 of the optic cup
forms the neural retina of the eye. The anterior part of this inner layer forms the
non-pigmented layer of the iris and the ciliary process epithelium. The outer
layer of the optic cup in this region contributes the pigmented epithelial layer. A
few folds form in the anterior aspect of the optic cup and this forms the ciliary
processes.The stroma of the iris and the ciliary body develop from neural crest
cells that migrate into the area. Within the stroma of the iris, the sphincter
pupillae and dilator pupillae muscles develop from optic cup neuroectoderm. In
contrast, the ciliary muscle, which is responsible for changing the shape of the
lens, is derived from overlying mesenchyme. The color of the eye is determined
by the amount of melanin distributed in the stroma of the iris. Eyes of all colors
have melanin in the epthelium on the posterior aspect of the iris.
Vitreous Body
The vitreous body forms in the center of the optic cup posterior to the lens. It is
comprised of a gel-like substance called vitreous humor derived from
mesenchymal cells of neural crest origin. Later on more vitreous humor is added
which is believed to come from the neuroectoderm of the optic cup.
Eyelid and Conjunctiva
The eyelids begin to form in the sixth week from neural crest cells as well as
surface ectoderm just anterior to the cornea. They begin as two folds of skin that
meet over the cornea and they are attached to one another until about the
27thweek when they separate. While they are adherent to one another there is a

conjunctival sac between the eyelids and the cornea.The orbicularis oculi, which
is found within the eyelids, forms from the second branchial arch along with the
other muscles of facial expression and will be innervated with SVE fibers.
Extraocular Muscles
The extraocular muscles develop from three preotic somites. These are the
somites founds anterior to the developing ear of the embryo. Each preotic somite
is supplied by its own cranial nerve. Remember three different cranial nerves
(III, IV, and VI) supply the extraocular muscles. So the somite which is supplied
by the III cranial nerve forms 5 of the 7 extraocular muscles while the remaining
two each give rise to one muscle each.
Regulation of Eye Development
Normal development of the eye requires a rather complex interplay between
different tissues of the eye and involves several reciprocal inductive events (Fig.
2).
The PAX6 gene product, a transcription factor, is a key player in the process.
Development of the eye begins with the designation of a single eye field in the
neural plate before neurulation begins. The separation of this single eyefield into
two eyefields is dependent upon secretion of sonic hedgehog (shh) from the
prechordal plate.
It has been suggested that the sonic hedgehog protein suppresses the expression
of the PAX6 gene and upregulates the PAX2 gene in the anterior neural ridge,
which causes the field to divide in two. A defect in the sonic hedgehog protein or
its expression, therefore, results in cyclopia.
In the third week when the optic vesicle buds from the neuroectoderm, it induces
the overlying surface ectoderm to form the lens placode by secreting the growth
factor BMP4.
The ability of the surface ectoderm to respond to BMP4 is dependent on the
expression of the PAX6 gene in the surface ectoderm. The lens placode in turn
becomes the inducer and secretes growth factors (FGF among them) that induce
the optic vesicle to differentiate into the optic cup.
Then as the lens vesicle forms from the lens placode it secretes factors that
induce the formation of the neural retina in the wall of the optic cup. In addition,
the lens vesicle also induces the overlying ectoderm to begin forming the
cornea. Now the neural retina becomes the inducer and secretes factors that
cause the cells at the inner aspect of the lens vesicle to elongate and become lens
fibers.
As the inner neural retinal layer is forming, the mesenchyme surrounding the
optic cup secretes transforming growth factor (TGF), which induces the
formation of the pigmented retinal layer as well as the choroid and the sclera.
DERIVATIVES OF VARIOUS LAYERS IN THE EYE
Pigmented epithelia of retina ( 1 layer), ciliary body
Neuroectoderm
(1 layer), and iris (2 layers).
Sensory retina, and innermost (nonpigmented) layer
of ciliary body

Optic nerve
Iris sphincter and dilator muscles

Surface ectoderm

Vitreous (part)
1.
Lens
2.

Corneal epithelium

3.

Conjunctiva and caruncle

4.

Eyelid skin

5.
Lacrimal apparatus (glands and drainage
system)
Head mesenchyme (neural 1. Blood vessels
crest and/or mesoderm)
2. Corneal stroma and endothelium
3.

Stroma of choroid, ciliary body, and iris

4.

Ciliary muscle

5.

Sclera

6.

Optic nerve sheath (meninges)

7.

Extraocular muscles and fasciae

8. Remainder of the eyelids (orbicularis oculi


muscle, tarsus, orbital septum, etc)
9.

Vitreous (part)

(Q.45) Primary curvatures of vertebral column are


Cervical & lumbar
(a)
Thoracic & sacral
(b)
Cervical & thoracic
(c)
Thoracic & Lumbar
(d)
Your Response :
Correct Answer
B
:
Thoracic & sacral
Exp:
In a newborn the vertebral column is concave anteriorly throughout its extentAttitude of Flexion.
When the baby crosses the age of 2 months, neck holding has got initiated and
leads to development of a secondary curvature in the cervical spine, which will
be convex anteriorly.
As the individual crosses the age of 10 months there comes another secondary

curvature, this time in the lumbar region. This anterior convexity in the lumbar
region occurs due to the attainment of standing posture by the age of 10 months.
The secondary curvatures are the cervical & lumbar and will be termed convex
anteriorly as per convention of anatomical position looking at the patient from
the anterior side.
The thoracic & sacral curvatures are concave anterior, as they were at birth, so
are called as primary curvatures.
References: 1. Pg. 196; B D Chaurasia (4th Ed.); Vol-1.

(Q.46) A patient is unable to close her right eye. Physical examination reveals weakness of the right
orbicularis oculi. Which of the following additional symptoms would likely also be present?
Blurred vision
(a)
Hyperacusis
(b)
Inability to chew
(c)
Inability to feel the face
(d)
Your Response :
Correct Answer
B
:
Hyperacusis
Exp:
The facial nerve innervates the muscles of facial expression. The visceral motor
portion innervates the lacrimal gland and nasal mucous glands via the
sphenopalatine ganglion.
The submandibular and sublingual salivary glands are innervated via the
submandiublar ganglion.
This patient has a lesion of the facial nerve (VII), which leads to an inability to
close the ipsilateral eye because of damaged motor fibers to the orbicularis oculi.
This patient would also lose her corneal reflex on that side because of an
inability to blink and would have ipsilateral paralysis of the muscles of facial
expression distal to the lesion.
If the lesion affected the facial nerve more proximally, additional findings would
be hyperacusis (increased sensitivity to sound because of stapedius muscle
paralysis), lack of taste sensation in the anterior two-thirds of the tongue, and
disturbed lacrimation and salivation.
Blurred vision could occur with lesions of the oculomotor (CN III), abducens
(CN VI), or trochlear (CN IV) nerves, which innervate the extraocular muscles.
CN III innervates the medial rectus, inferior rectus, superior rectus, and inferior
oblique muscles. CN VI innervates the lateral rectus and CN IV innervates the
superior oblique.
An inability to chew would probably be the result of a lesion of the trigeminal
nerve (CN V). Motor fibers of CN V innervate the muscles of mastication
(temporalis, masseter, and medial and lateral pterygoid muscles), and a lesion of
these fibers may cause the jaw to deviate to the side of the weak muscles.
An inability to feel the face would also be the result of a CN V lesion. This
lesion could result in the psilateral loss of general sensation of the face and also

of the mucous membranes of the oral and nasal cavities.


A lesion of the accessory nerve (CN XI) would cause paralysis of the trapezius
muscle, which results in a sagging of the shoulder and a weakness in attempting
to shrug the shoulder.

(Q.47) A patient received a severe blow to the lateral side of the head, resulting in an epidural
hematom(a.) Which of the following blood vessels was most likely torn?
Anterior cerebral artery
(a)
Middle cerebral artery
(b)
Middle meningeal artery
(c)
Superficial temporal artery
(d)
Your Response :
Correct Answer
C
:
Middle meningeal artery
Exp:
The middle meningeal artery is in the interior of the lateral portion of the cranial
cavity, embedded in the periosteal (outer) layer of the dura. A tear of this artery
results in blood entering the potential space between the outer dural layer and the
skull (epidural space), causing an epidural hematoma.
The anterior cerebral artery and middle cerebral artery (choices A and B) lie on
the surface of the brain. The anterior cerebral arteries supply the medial surface
of the cerebral hemispheres, and the middle cerebral arteries supply the lateral
surface of the cerebral hemispheres. A tear of either of these arteries would result
in blood entering the subarachnoid space (subarachnoid hemorrhage).
The superficial temporal artery is a branch of the external carotid artery and is
external to the skull. This artery supplies the skin and other tissue of the temple
region.
The superior cerebral veins drain the cerebral hemisphere and enter the superior
sagittal sinus. A tear of these veins results in blood entering the potential space
between the dura and arachnoid (subdural space), causing a subdural hematoma.

(Q.48) X-ray of a child's arm after a fall appears to show a fracture near, but not at, distal end of the
uln(a.) Before diagnosing a fracture, you should also consider the possibility that this is actually
which of the following?
Articular cartilage
(a)
Epiphyseal plate
(b)
Perichondrium
(c)
Primary ossification center
(d)
Your Response :
Correct Answer
B
:
Epiphyseal plate
Exp:
The epiphyseal plate of the bone contains cartilage that is radiolucent. The plate
in a bone that is not yet fully ossified can produce a "line" crossing the bone near
the end. This may be easily mistaken for a fracture by the inexperienced.
Anatomically, the epiphyseal plate separates the epiphysis from the diaphysis.

Articular cartilage is radiolucent, but occurs at the very tip of the long bones.
Perichondrium is usually difficult to see on x-ray.
Primary and secondary ossification centers are radiopaque.

(Q.49) Which of the following embryonic structures gives rise to the adrenal cortex?
Ectoderm
(a)
Endoderm
(b)
Mesoderm
(c)
Mesonephros
(d)
Your Response :
Correct Answer
C
:
Mesoderm
Exp:
The mesoderm gives rise to the adrenal cortex. In addition, it also gives rise to
connective tissue, cartilage, bone, muscle, blood and lymph vessels, kidneys,
gonads, serous membranes lining body cavities, and the spleen.
The ectoderm gives rise to the central nervous system, peripheral nervous
system, epidermis and its appendages, mammary glands, pituitary gland, tooth
enamel, and the neural crest.
The endoderm gives rise to the parenchyma of the tonsils, thyroid and
parathyroid glands, thymus, liver, pancreas, the epithelial lining of the
gastrointestinal and respiratory tracts, urinary bladder, urethra, and auditory tube.
The mesonephros functions as an interim kidney in the embryo.
The neural crest cells give rise to cells of the spinal and cranial nerves,
autonomic ganglia, melanocytes, leptomeninges, connective tissue and bone of
branchial arch origin, and the adrenal medulla.

(Q.50) Most common aberration in renal vessel development


Supernumerary arteries
(a)
Supernumerary renal vein
(b)
Double renal vein
(c)
Double renal artery
(d)
Your Response :
Correct Answer
A
:
Supernumerary arteries
Exp:
The most common aberration in renal vessel development is supernumerary
(accessory) renal arteries (a).
Approximately 25 % of adult kidneys present with 2 - 4 renal arteries (more
common on left side).
Supernumerary arteries arise from the aorta and follow the main renal artery to

the hilum of kidney. Usually they are given superior to the main renal artery.
The arteries may also enter the kidney directly to reach the poles
(superior/inferior) instead of hilum.
An accessory renal artery to the inferior pole of kidney usually cross the ureter
anteriorly and obstructs it, thus leading to hydronephrosis.
The role of crossing supernumerary renal arteries in ureteropelvic junction
obstruction (UPJO) and hydronephrosis is controversial. Though UPJO is
believed to be secondary to abnormal muscular relaxation, a crossing renal artery
has been observed in 1139% of cases.
A supernumerary renal artery is a common arterial variation, which warrants
considerations in a variety of urologic and retroperitoneal operations.
Supernumerary renal arteries can be associated with other uro-vascular variations
and anomalies such as duplicated renal veins and ureters, aberrant origin of the
gonadal arteries, persistence of fetal renal lobulation, and kidney malrotation.
Supernumerary arteries are end arteries andif they are damaged or ligated, the
sector supplied by the artery becomes ischaemic.
Supernumerary arteries are twice as common as supernumerary renal veins
(b).
References: 1. Pg. 250; The Developing Human - Keith L Moore (8th Ed.)

(Q.51) Gluteus medius is supplied by the nerve


Superior gluteal
(a)
Inferior gluteal
(b)
Femoral
(c)
Sciatic
(d)
Your Response :
Correct Answer
A
:
Superior gluteal
Exp:
Gluteus medius is supplied by superior gluteal nerve (a).
Superior gluteal nervesupplies three muscles:
- Gluteus medius

- Gluteus minimus

- Tensor fascia lata

These three muscles lead to abduction and medial rotation at hip joint.
They also prevent downward tilt of unsupported pelvis when person is standing
on one foot.
If superior gluteal nerve is damaged, the three muscles get paralysed and the
unsupported pelvis tilts down Trendelenburg test positive.
Inferior gluteal nerve (b) supplies only one muscle Gluteus maximus. This
muscle is the chief muscle of hip extension.

Femoral nerve (c) supplies the anterior thigh muscles like quadriceps femoris.
Sciatic nerve (d) supplies the posterior thigh muscles and the muscles of the leg
and sole.
References: 1. Pg. 73,75; B D Chaurasia (4th Ed.); Vol-2

(Q.52) In cases of dysphagia where the esophagus is compressed, which structure would be most likely
to cause the compression?
Left atrium
(a)
Left ventricle
(b)
Pulmonary trunk
(c)
Right atrium
(d)
Your Response :
Correct Answer
A
:
Left atrium
Exp:
The left atrium forms most of the posterior wall of the heart. The esophagus
passes immediately posterior to the heart. Enlargement of the left atrium may
compress the esophagus and cause dysphagia. Anatomically, the esophagus
begins posterior to the cricoid cartilage, at the level of vertebrum C6.
From this point, it descends toward the thoracic cavity posterior to the trachea,
passes inferiorly along the dorsal wall of the mediastinum, and enters the
abdominopelvic cavity through an opening in the diaphragm, the diaphragmatic
hiatus.
The left ventricle forms most of the left border of the heart and most of the
diaphragmatic surface of the heart.
The left ventricle is not related to the esophagus.
The pulmonary trunk emerges from the right ventricle on the anterior surface of
the heart. The pulmonary trunk is not related to the esophagus.
The right atrium forms the right border of the heart. It is not related to the
esophagus.
The right ventricle forms most of the anterior wall of the heart and a small
portion of the diaphragmatic surface of the heart. It is not related to the
esophagus.

(Q.53) Which of the tissues normally has the highest percentage of mucus-secreting cells is:
Esophageal mucosa
(a)
Oral mucosa
(b)
Parotid gland
(c)
Sublingual gland
(d)
Your Response :
Correct Answer
D
:

Exp:

Sublingual gland
The sublingual salivary glands are located beneath the mucous membrane of the
floor of the mouth. Numerous sublingual ducts open along either side of the
lingual frenulum. Salivary glands can contain predominantly serous cells,
predominantly mucous cells, or both in their acini. As you proceed from the
midline laterally, acini in the sublingual gland are almost pure mucous cells,
whereas acini in the submandibular gland contain a mixture of serous and
mucous cells. Acini in the parotid gland are mostly pure serous cells.
The esophageal mucosa and the oral mucosa are squamous epithelia that do not
form acini.

(Q.54) The primary arterial supply to the nasal mucosa is a direct branch of which of the following
arteries?
Facial artery
(a)
Maxillary artery
(b)
Superficial temporal artery
(c)
Superior labial artery
(d)
Your Response :
Correct Answer
B
:
Maxillary artery
Exp:
The major source of blood supply to the nasal mucosa is the sphenopalatine
artery, which is the terminal branch of the maxillary artery. The sphenopalatine
artery enters the nasal cavity from the pterygopalatine fossa by passing through
the sphenopalatine foramen in the lateral wall of the nasal cavity.
The facial artery is a direct branch of the external carotid artery. It provides most
of the blood supply to the superficial face.
The superficial temporal artery is a terminal branch of the external carotid artery.
The other terminal branch is the maxillary artery. The superficial temporal artery
provides blood supply to the temporal region and the lateral portion of the scalp.
The superior labial artery is a branch of the facial artery. The superior labial
artery provides blood supply to the upper lip. It has a septal branch that provides
some of the blood supply to the anterior portion of the septal mucosa of the nasal
cavity. However, this is not the major blood supply to the nasal cavity.
The transverse facial artery is a branch of the superficial temporal artery. It
provides blood supply to the parotid gland, the parotid duct, and the skin of the
lateral face.

(Q.55) All of the following are characteristic of a bronchopulmonary segment except?


it is surgically resectable
(a)
it is named according to the segmental bronchus supplying it
(b)
it is drained by intrasegmental branch of pulmonary vein
(c)
Each is supplied by 2 arteries.
(d)
Your Response :
Correct Answer
C
:

Exp:

It is drained by intrasegmental branch of pulmonary vein


Each of the tertiary bronchi serves a specific bronchopulmonary segment. These
segments each have their own artery. Thus, each bronchopulmonary segment is
supplied by a bronchus, and two arteries, a pulmonary artery and a bronchial
artery which run together through the center of the segment. Veins and
lymphatics drain along the edges.
There are 10 bronchopulmonary segments in the right lung (3 in superior lobe, 2
in middle lobe, 5 in inferior lobe) and 8-10 segments on the left (4-5 in upper
lobe, 4-5 in lower lobe). Each segment is separated from the others by a layer of
connective tissue.
This means that each bronchopulmonary segment is a discrete anatomical and
functional unit, and this separation means that a bronchopulmonary segment can
be surgically removed without affecting the function of the other segments
Each broncho-pulmonary segment is drained by a intersegmental vein and not
INTRA segmental vein.

(Q.56) A mass is noted at the back of a young man's tongue. A biopsy's pathology report comes back
with a diagnosis of normal thyroid tissue. This finding is related to the embryonic origin of the
thyroid near which of the following structures?
First pharyngeal pouch
(a)
Foramen cecum
(b)
Nasolacrimal duct
(c)
Second
pharyngeal arch
(d)
Your Response :
Correct Answer
B
:
Foramen cecum
Exp:
The thyroid gland originates as a mass of endodermal tissue near the foramen
cecum, which is near the tuberculum impar (which becomes the central part of
the tongue). During development, the thyroid descends in front of the pharynx,
maintaining a connection to the tongue via the thyroglossal duct. Usually, the
thyroglossal duct disappears.
Uncommonly, residual ectopic thyroid tissue can be left anywhere along the
path, including at the back of the tongue. (In rare cases, all of the thyroid tissue
remains at this site, forming a mass that should not be excised, for obvious
reasons!)
Anatomically, the thyroid gland curves across the anterior surface of the trachea
just below the thyroid cartilage that forms the majority of the anterior surface of
the larynx. The two lobes of the thyroid gland are united by a slender connection,
the isthmus.
The first pharyngeal pouch develops into the middle ear and eustachian tube.
The nasolacrimal ducts connect the eyes to the oropharynx.
The second pharyngeal arch develops into many muscles of the face and the
styloid process of the temporal bone.

The third pharyngeal pouch develops into the thymus and inferior parathyroid
glands.

(Q.57) If the nerve that accompanies the superior laryngeal artery is damaged, which of the following
functional losses will ensue?
Loss of sensation in the laryngeal mucosa above the vocal folds
(a)
Loss of sensation in the laryngeal mucosa below the vocal folds
(b)
Loss of sensation in the pharyngeal mucosa
(c)
Paralysis of the cricothyroid muscle
(d)
Your Response :
Correct Answer
A
:
Loss of sensation in the laryngeal mucosa above the vocal folds
Exp:
The superior laryngeal artery is a branch of the superior thyroid artery. It enters
the larynx by passing through the cricothyroid membrane. In this region, it is
accompanied by the internal branch of the superior laryngeal nerve. This nerve
provides sensory innervation to the laryngeal mucosa above the vocal folds.
The laryngeal mucosa below the vocal folds receives its sensory innervation
from the recurrent laryngeal nerve.
The pharyngeal mucosa receives its sensory innervation from the
glossopharyngeal nerve.
The cricothyroid muscle receives its motor innervation from the external branch
of the superior laryngeal nerve.
The lateral cricoarytenoid muscle receives its motor innervation from the
recurrent laryngeal nerve.

(Q.58) An atrial septal defect results from failure of the


Ostium primum to form within the septum primum
(a)
Ostium secundum to form within the septum primum
(b)
Septum primum to fuse with the endocardial cushions
(c)
Septum primum to fuse with the septum secundum
(d)
Your Response :
Correct Answer
C
:
Septum primum to fuse with the endocardial cushions
Exp:
The septum primum (first interatrial septum) develops by growing from the
cranial end of the embryonic atrium toward the endocardial cushions. The gap
that exists between the two atria during this period is the ostium primum.
As the septum primum continues its growth, the ostium primum gets smaller
until it is closed when the septum primum completes its growth and completely
fuses with the endocardial cushions. Failure of the septum primum to fuse
completely with the endocardial cushions leaves a persistent ostium primum,
known as a primum-type atrial septal defect.
The ostium primum does not form within the septum primum . The ostium

primum is the communication between the two atria that exists during the
formation of the septum primum.
That is, the ostium primum is the space within the developing atrium not yet
occupied by the septum primum.
The ostium secundum normally forms within the septum primum before the
ostium primum closes by fusion of the septum primum with the endocardial
cushions. Failure of the ostium secundum to form would result in embryonic
death because there would be no pathway for blood to pass from the right atrium
to the left atrium when the ostium primum closes, thus depriving the embryo of
oxygenated blood.
Most of the septum primum normally disappears. The part that remains forms the
valve of the foramen ovale. This part of the septum primum normally does not
fuse with the septum secundum during prenatal life. After birth, the valve of the
foramen ovale is pushed against the septum secundum as a result of the increased
pressure in the left atrium.
This achieves functional closure of the foramen ovale. Fusion does not normally
occur at this time; it usually occurs later in life in most people. In some people,
however, complete fusion never occurs (probe patency). The septum secundum
normally does not fuse with the endocardial cushions.

(Q.59) A CT scan of the head demonstrates a mass in the olfactory groove are(a.) The axons likely
compressed by this mass project to which of the following structures?
Insula
(a)
Nucleus ambiguus
(b)
Postcentral gyrus
(c)
Pyriform cortex
(d)
Your Response :
Correct Answer
D
:
Pyriform cortex
Exp:
The olfactory nerves can be damaged by head injury, severe infection, and
tumors (such as meningiomas) that maycompress the olfactory bulbs.
Unilateral damage is often not noticed by the patient, but bilateral damage
(whichsurprisingly can also be missed if it develops slowly) may lead to
occasional bizarre errors in odor identification.
The olfactory nerves are technically the neurons on the mucosal side of the
cribriform plate that send processes through the plate to synapse in the olfactory
bulb, which in turn sends axons to the pyriform cortex (the primary olfactory
cortex).
The insula is thought to contain the primary gustatory (taste) cortex.
The nucleus ambiguus provides the special visceral efferent fibers carried by the
glossopharyngeal and vagus nerves.
The postcentral gyrus contains the primary sensory cortex.
The precentral gyrus contains the primary motor cortex.

(Q.60) A biopsy specimen demonstrates a ciliated columnar epithelium. From which of the following
locations in the female genital tract was the biopsy obtained?
Cervix
(a)
Endometrium
(b)
Fallopian tube
(c)
Ovary
(d)
Your Response :
Correct Answer
C
:
Fallopian tube
Exp:
The fallopian tube is the only structure in the female genital tract with a ciliated
columnar epithelium; the beating of the cilia helps move the egg into the uterus.
This fact is also sometimes clinically helpful because dilated and deformed
fallopian tubes can be microscopically distinguished from cystic ovarian tumors
by the presence of the cilia. Each uterine tube is a hollow, muscular tube
measuring approximately 13 cm in length.
Each uterine tube is divided into three regions: the infundibulum, the ampulla,
and the isthmus.
The cervix and vagina are lined by squamous epithelium.
The endometrium is lined by columnar epithelium (although a few ciliated cells
may be present).
The covering of the ovary is cuboidal epithelium, and cysts within the ovary can
be lined by cuboidal or nonciliated columnar epithelium.

(Q.61) A person sits with his legs straight together and then opens his legs against lateral resistance.
Which nerve controls this type of movement?
Deep peroneal nerve
(a)
Femoral nerve
(b)
Obturator nerve
(c)
Superior gluteal nerve
(d)
Your Response :
Correct Answer
D
:
Superior gluteal nerve
Exp:
The motion described is hip abduction. The superior gluteal nerve from roots L4S1 is responsible for hip abduction.
The deep peroneal nerve controls dorsiflexion of the foot. If you see a patient
with "foot drop," think of damage to the deep peroneal nerve.
The femoral nerve is responsible for knee extension.
The obturator nerve is responsible for hip adduction. The hip adduction machine
would provide resistance to the medial aspects of the thighs, and the individual
would push his/her legs together against the machine's resistance.

The sciatic nerve controls knee flexion.

(Q.62)

All structure are attached to stomach EXCEPT


Lesser omentum
(a)
Greater omentum
(b)
Hepatogastric ligament
(c)
Gastrorenal ligament
(d)
Your Response :
Correct Answer
D
:
Gastrorenal ligament
Exp:
The lesser omentum is derived from the ventral mesentery and is attached to the
lesser curvature of the stomach. It has two components: the hepatogastric
ligament and the hepatoduodenal ligament. The greater omentum is derived from
the dorsal mesentery and is attached to the greater curvature of the stomach. The
gastrosplenic ligament also is derived from the dorsal mesentery and connects
the greater curvature of the stomach to the spleen. There is no gastrorenal
ligament.

(Q.63) Damage to the upper fibers of the trapezius muscle would most likely impair which
movements?
Depression of the medial end of the scapula
(a)
Elevation of the acromion
(b)
Flexion of the neck
(c)
Raising of the medial border of the scapula
(d)
Your Response :
Correct Answer
B
:
Elevation of the acromion
Exp:
The trapezius is a large muscle arising in the posterior midline from the superior
nuchal line of the skull, ligamentum nuchae, and spinous processes of all of the
thoracic vertebrae. The upper fibers, which are potentially damaged by wounds
to the posterior and lateral neck, insert on the crest of the spine of the scapula,
the medial border of the acromion, and the lateral third of the clavicle. The upper
fibers serve to elevate the acromion.
The lower fibers of the trapezius depress the medial end of the scapula .
The sternocleidomastoid are important in extension of the head at the atlantooccipital joint and flexion of the cervical vertebral column .
The rhomboids and levator scapulae are important in raising the medial border of
the scapula .
Contraction of a single sternocleidomastoid muscle is important in turning of the
face sideways in the contralateral direction.

(Q.64) Following radiograph represents which rotational anomaly of midgut loop?

Reverse rotation
(a)
Non rotation
(b)
Mixed rotation
(c)
Omphalocele
(d)
Your Response :
Correct Answer
B
:
Non rotation
Exp:
Anomalies of the Midgut
Congenital abnormalities of the intestine are common; most of them are
anomalies of gut rotation-nonrotation or malrotation of the gut-that result from
incomplete rotation and/or fixation of the intestines.
Nonrotation occurs when the intestine does not rotate as it reenters the
abdomen. As a result, the caudal limb of the midgut loop returns to the
abdomen first and the small intestines lie on the right side of the abdomen
and the entire large intestine is on the left.
The usual 270-degree counterclockwise rotation is not completed, and the cecum
lies just inferior to the pylorus of the stomach. The cecum is fixed to the
posterolateral abdominal wall by peritoneal bands that pass over the duodenum.
These bands and the volvulus (twisting) of the intestines cause duodenal
obstruction.
This type of malrotation results from failure of the midgut loop to complete the
final 90 degrees of rotation. Only two parts of the intestine are attached to the
posterior abdominal wall: the duodenum and proximal colon.
This improperly positioned and incompletely fixed intestine may lead to a
catastrophic twisting of the midgut-midgut volvulus. The small intestine hangs
by a narrow stalk that contains the superior mesenteric artery and vein.
When midgut volvulus occurs, the superior mesenteric artery may be obstructed,
resulting in infarction and gangrene of the intestine supplied by it.
Infants with intestinal malrotation are prone to volvulus and present with bilious
emesis (vomiting bile). A simple contrast study can determine the presence of
rotational abnormalities.
Reversed Rotation

In very unusual cases, the midgut loop rotates in a clockwise rather than a
counterclockwise direction. As a result, the duodenum lies anterior to the
superior mesenteric artery rather than posterior to it, and the transverse colon lies
posterior instead of anterior to it. In these infants, the transverse colon may be
obstructed by pressure from the superior mesenteric artery. In more unusual
cases, the small intestine lies on the left side of the abdomen and the large
intestine lies on the right side, with the cecum in the center. This unusual
situation results from malrotation of the midgut followed by failure of fixation of
the intestines.

(Q.65) Which of the following cell types is derived from neuroepithelial cells?
Astrocytes
(a)
Enterochromaffin cells
(b)
Melanocytes
(c)
Odontoblasts
(d)
Your Response :
Correct Answer
A
:
Astrocytes
Exp:
Astrocytes and oligodendrocytes are both derived from glioblasts, which, in turn,
are derived from neuroepithelial cells. Other neuroepithelial cell derivatives
include neuroblasts and ependymal cells. The astrocytes are the largest and most
numerous glial cells.
These cells are responsible for maintaining the blood-brain barrier, creating a
three-dimensional framework for the central nervous system, performing repairs
in damaged neural tissues, and controlling theinterstitial environment.
All the other choices are derived from neural crest cells. Other neural crest
derivatives include the neurons of the parasympathetic and sympathetic ganglia
(including the adrenal medulla), the dorsal root ganglia of the peripheral nervous
system, the sensory ganglia of cranial nerves V, VII, IX, and X, and the
leptomeninges (pia and arachnoid).

(Q.66) Following a fracture of the humerus, which of the following is responsible for producing the
majority of the new bone that will reunite the two fragments?
Cancellous bone
(a)
Cartilage
(b)
Compact bone
(c)
Periosteum
(d)
Your Response :
Correct Answer
D
:
Periosteum
Exp:
When the periosteum is torn during a fracture, it supplies cells that develop into
osteoblasts and are the major producers of the new bone that reunites the two
ends.
Heterotopic ossification (bone formed outside the regular bone) can occur as a
complication of fracture if some of the osteoblastic cells are misdirected into

adjacent tissues.
The periosteum assists in the attachment of the osteoblasts to surrounding tissues
and to associated tendons and ligaments. This cellular layer functions in bone
growth and participates in repair after an injury.
Pre-existing cancellous bone and compact bone are not the major source of
osteoblasts that form the new bone.
Cartilage and marrow do not contribute to new bone formation after fracture.

(Q.67) Which membrane is in the way if you try to reach the lesser peritoneal sac and head of the
pancreas after penetrating the greater peritoneal sac?
Falciform ligament
(a)
Gastrohepatic ligament
(b)
Gastrosplenic ligament
(c)
Hepatoduodenal ligament
(d)
Your Response :
Correct Answer
B
:
Gastrohepatic ligament
Exp:
The gastrohepatic ligament is the part of the lesser omentum that separates the
greater peritoneal sac from the right portion of the lesser peritoneal sac. This
portion of the lesser omentum has no significant blood vessels within it and may
be incised for surgical access.
The falciform ligament is a mesenteric membrane between the liver and the
anterior abdominal wall. This ligament is within the greater peritoneal sac and
does not separate it from the lesser peritoneal sac.
The gastrosplenic ligament passes from the greater curvature of the stomach to
the spleen. It separates the greater peritoneal sac from the left portion of the
lesser peritoneal sac. Incision of this structure would be the appropriate surgical
approach to gain access to the left side of the lesser peritoneal sac.
The hepatoduodenal ligament is part of the lesser omentum and separates the
greater peritoneal sac from the right portion of the lesser peritoneal sac. It forms
the anterior border of the epiploic foramen. However, the hepatoduodenal
ligament contains the common bile duct, the proper hepatic artery, and the portal
vein, and therefore may not be incised for surgical access.
The splenorenalligament passes from the spleen to the parietal peritoneum on the
anterior surface of the left kidney. It separates the greater peritoneal sac from the
left portion of the lesser peritoneal sac. This ligament contains the splenic artery,
splenic vein, and the tail of the pancreas, and therefore may not be incised to
gain access to the lesser peritoneal sac.

(Q.68) Which of the following fibers provide the only output from the cerebellar cortex?
Climbing
(a)
Golgi cell
(b)
Mossy
(c)
Purkinje
(d)
Your Response :

Correct Answer
D
:
Purkinje
Exp:
Two basic things about cerebellar circuitry that are well-worth knowing are that
Purkinje cells of the cerebellar cortex project to the deep cerebellar nuclei and
that these nuclei project out of the cerebellum. Purkinje cells are located in the
second (Purkinje) cell layer of the cerebellar cortex and form inhibitory synapses
on the deep cerebellar nuclei.
Climbing fibers are afferents to the cerebellum. Specifically, they originate from
the medullary olivary nuclear complex, enter the cerebellum through the inferior
cerebellar peduncle, and project to the deep cerebellar nuclei and the lateral
cerebellar hemispheres.
Golgi cell bodies reside in the granule cell layer of the cerebellar cortex and
project predominantly to the granule cells, where they form inhibitory synapses.
Therefore, they reside in and project to the cerebellar cortex.
Granule cells reside in the granule cell layer of the cerebellar cortex and send
their axons to the moleculary layer, where they bifurcate into parallel fibers.
Granule cells, therefore, both reside in and project to the cerebellar cortex.
Mossy fibers consist of all of the afferents (except the climbing fibers) to the
cerebellum. These fibers project to deep cerebellar nuclei and glomeruli in the
granular layer of the cerebellar cortex.

(Q.69) Structure passing through the marked opening in the diaphragm is

Oesophagus
(a)
Left gastric nerve
(b)
Right phrenic nerve
(c)
Hemiazygous vein
(d)
Your Response :
Correct Answer
C
:
Right phrenic nerve
Exp:
Marked opening is caval opening in the central tendon of diaphragm.
Level Contents
Description
Caval opening
T8
The caval opening passes through the central tendon

Esophageal hiatus T10

Aortic hiatus

T12

of the diaphragm. It contains the inferior vena


cavaand the right phrenic nerve.
The esophageal hiatus is situated in the posterior
part of the diaphragm, located slightly left of the
central tendon through the muscular sling of the
right crus of the diaphragm.It contains
the esophagus, and anterior and posterior vagal
trunks
The aortic hiatus is in the posterior part of the
diaphragm, between the left and right crus.It
contains the aorta, the azygos vein, the hemiazygos
vein, and the thoracic duct.

(Q.70) If cirrhosis causes obstruction of the portal circulation within the liver, portal blood could still
be conveyed to the caval system via which of the following?
Azygos and hemiazygos veins
(a)
Gonadal veins
(b)
Internal iliac veins
(c)
Splenic vein
(d)
Your Response :
Correct Answer
A
:
Azygos and hemiazygos veins
Exp:
The esophageal venous plexus, which drains into the azygos and hemiazygos
veins within the thorax, has anastomoses with branches of the left gastric vein.
Thus, following blockage of the portal vein, portal blood may enter the superior
vena cava via the azygos system. Other important portacaval connections include
the superior rectal vein with the middle and inferior rectal veins; paraumbilical
veins and epigastric veins (engorgement of these vessels results in caput
medusae); and the colic and splenic veins with renal veins and veins of the
posterior body wall.
The gonadal veins exclusively drain the gonads (although in the female, the
ovarian vein communicates with the uterovaginal plexus). These vessels have no
anastomoses with portal veins.
The internal iliac veins , which drain most of the pelvis and much of the inferior
extremities, have no demonstrated portal anastomoses.
The splenic vein is incorrect because it's in itself a component of the portal
venous system.
The vesical venous plexus , which is situated well within the pelvis and drains
the bladder and the prostate (or uterus and vagina) has no association with portal
vessels.

(Q.71) Secretory basket cells are located


inside secretory acini
(a)
surrounding the intercalated duct
(b)
between secretory cells and the basal lamina
(c)
surrounding the striated duct
(d)

Your Response :
Correct Answer
C
:
between secretory cells and the basal lamina
Exp:
Secretory basket cells are found in salivary gland acini and are also called
myoepithelial cells. The name implies that they are epithelial cells with musclelike contractile properties.
They are found between secretory cells and the basal lamina and exert pressure
on the secretory cells to release their secretions. Secretory acini themselves are
composed of mucous or serous secreting cells. Intercalated duct cells are
nonspecialized cuboidal epithelial cells.
Striated duct cells are columnar epithelial cells with striations (stripes) of rows of
mitochondria used for ATP generation for active transport.

(Q.72) Which muscle is the chief mover of the mandible TOWARD the left?
Left medial pterygoid
(a)
Left lateral pterygoid
(b)
Right medial pterygoid
(c)
Right lateral pterygoid
(d)
Your Response :
Correct Answer
D
:
Right lateral pterygoid
Exp:
There are two methods for answering this question. The formula method states
that the right lateral pterygoid moves the mandible left, whereas the left lateral
pterygoid moves the mandible right.
The understanding method begins with the idea that the medial pterygoid is a
closer (elevator).
The lateral pterygoid connects to the condyle and is a protruder. If the right
lateral pterygoid pulls the right condyle out while the left lateral pterygoid and
condyle remain stable, the left side will act as a stationary pivot point while the
right side protrudes outward and also toward the left (medially).

(Q.73) Which of the following cells is the germ cell closest to the basal lamina in the seminiferous
tubule?
Primary spermatocyte
(a)
Secondary spermatocyte
(b)
Spermatid
(c)
Spermatogonia
(d)
Your Response :
Correct Answer
D
:
Spermatogonia
Exp:
Maturation of germ cells (spermatogenesis) within the seminiferous tubules
occurs in a concentric pattern, with the less mature spermatogonia near the basal

lamina and the mature forms near the tubule center.


Along this route the developing sperm are nurtured by sertoli cells.
Spermatogonia are 2N cells and mature into larger primary spermatocytes (4N) .
These mature into secondary spermatocytes (2N) , and finally into spermatids
(1N) . Spermatids undergo spermiogenesis to become mature spermatozoa .
Acrosomes form from the Golgi apparatus, and a flagellum forms from
microtubules. Unneeded organelles are shed.
The seminiferous tubules of a reproductive-age male should exhibit all stages of
maturation, with mature flagellated sperm in their centers, no longer associated
with sertoli cells.

(Q.74) All the true about inguinal canal except:


Conjoint tendon forms part of the posterior wall
(a)
Superficial ring is found in ext oblique aponeurosis
(b)
Deep ring is an opening in transversalis abdominis
(c)
Internal oblique forms both anterior and posterior wall
(d)
Your Response :
Correct Answer
C
:
Deep ring is an opening in transversalis abdominis
Exp:
Deep inguinal ring is an opening in the transversalis fascia and not the muscle
(C).
Conjoint tendon is formed by the merging tendons of internal oblique and the
transversus abdominis and form the posterior wall (A) of inguinal canal at its
medial end.
Superficial ring is a triangular aperture found in external oblique aponeurosis
(B).
Internal oblique muscle lies anteriorly (D) at the lateral end of the inguinal canal.
It arches over the canal to form the roof and then runs medially and posteriorly to
meet with the transversus abdominis to form the conjoint tendon.

(Q.75)

All of the following are contents of deep perineal pouch except


Membranous part of urethra
(a)
Prostatic part of urethra
(b)
Transversus perinei profundus
(c)
Cowper glands
(d)
Your Response :
Correct Answer
B
:
Prostatic part of urethra
Exp:
The deep perineal pouch contains:
Membranous portion of the urethra (males) / proximal portion of urethra
(females)

External urethral sphincter (and membranous urethra) & Deep transverse


perineal muscle (both muscles constitute urogenital diaphragm)
Bulbourethral gland of Cowper (males)
Neuro-vascular bundle
References: 1. Pg. 335; B D Chaurasia (4th Ed.); Vol-2.

(Q.76) If a person has normal musculature, but has difficulty swallowing, which nerves should be
tested for function?
Hypoglossal and phrenic
(a)
Hypoglossal and splanchnic
(b)
Glossopharyngeal and vagus
(c)
Splanchnic and vagus
(d)
Your Response :
Correct Answer
D
:
Splanchnic and vagus
Exp:
The upper 2/3 of the esophagus contains striated muscle. It is derived from the
pharyngeal arches and innervated by the vagus nerve (CN X). The lower 1/3
contains smooth muscle from splanchnic mesoderm and is innervated by the
splanchnic plexus.
The vagus nerve has mixed sensory and motor functions. The sensory fibers
innervate the sensory nuclei and autonomic centers of the medulla oblongata.
The motor fibers innervate muscles of the palate, pharynx, and respiratory and
cardiovascular systems.
The hypoglossal nerve (choices A and B), or CN XII, moves the tongue.
The phrenic nerve (choices A and D), derived from C3, C4, and C5, innervates
the muscle of the diaphragm.
The glossopharyngeal nerve , or CN IX, functions in taste, swallowing, and
salivation, as well as monitoring the activity of the carotid body.

(Q.77) The left adrenal vein drains directly into which of the following veins?
Hemiazygos vein
(a)
Inferior vena cava
(b)
Left renal vein
(c)
Splenic vein
(d)
Your Response :
Correct Answer
C
:
Left renal vein
Exp:
The left adrenal vein and the left gonadal vein (either testicular or ovarian) drain
into the left renal vein. The left renal vein then drains into the inferior vena cava.

In contrast, the right adrenal vein and right gonadal vein drain directly into the
inferior vena cava.
The hemiazygos vein receives the venous drainage from the body wall on the left
side of the thorax and abdomen.
No visceral organs drain directly to the azygos or hemiazygos veins.
The inferior vena cava receives the direct venous drainage from the right adrenal
vein, but not the left adrenal vein. Remember, the inferior vena cava is on the
right side of the abdomen.
The splenic vein receives the venous drainage from the spleen and part of the
pancreas and stomach. The splenic vein is part of the portal venous system.
The superior mesenteric vein receives venous drainage from much of the
intestinal tract. It is part of the portal venous system and joins with the splenic
vein to form the portal vein.

(Q.78) Which of the following is present in males but not in females?


Bulbospongiosus muscle
(a)
Bulbourethral gland
(b)
Corpus cavernosum
(c)
Membranous urethra
(d)
Your Response :
Correct Answer
B
:
Bulbourethral gland
Exp:
The bulbourethral glands are paired structures located within the deep perineal
pouch, embedded within the sphincter urethrae. Their ducts pass to the spongy
urethra.
The homologous female structures are the greater vestibular (Bartholin's) glands,
which are located in the superficial perineal pouch.
The bulbospongiosus muscles lie superficial to the bulb of the penis in males and
to the bulbs of the vestibule infemales.
The corpora cavernosa are paired structures, consisting of cavernous erectile
tissue that form a large portion of the penile shaft in males and of the body of the
clitoris in females.
The membranous urethra is the portion of the urethra that passes through the
urogenital diaphragm in both males and females.
The perineal body is the centrally located tendinous structure that provides
attachment for perineal musculature in both males and females. It separates the
urogenital area from the anal area and is an important obstetric landmark.

(Q.79) Luminal narrowing of which of the following vessels would compromise blood flow through the
renal arteries?

(a)

Abdominal aorta

Celiac trunk
(b)
Common iliac artery
(c)
Inferior mesenteric artery
(d)
Your Response :
Correct Answer
A
:
Abdominal aorta
Exp:
The renal arteries emerge from the abdominal aorta at about the level of the
L1/L2 intervertebral disk and travel at nearly right angles to it (on the right,
passing posterior to the inferior vena cava) to enter the hilum of the kidney.
The celiac trunk gives off the common hepatic, splenic, and left gastric arteries.
The common iliac artery gives off the internal and external iliac arteries. In
addition, an unascended pelvic kidney may be supplied by the common iliac
artery.
The inferior mesenteric artery gives off the superior rectal, sigmoid, and left
colic arteries.
The superior mesenteric artery gives off the inferior pancreaticoduodenal,
intestinal (ileal and jejunal), right colic, middle colic, and ileocolic arteries.

(Q.80) Which muscle has fibers that enter the articular disc and capsule?
Medial pterygoid
(a)
Buccinator
(b)
Temporalis
(c)
Lateral pterygoid
(d)
Your Response :
Correct Answer
D
:
Lateral pterygoid
Exp:
Only the lateral pterygoid enters the temporomandibular joint (TMJ). Its fibers
join the articular disc and capsule.
These fibers are from the superior head of the muscle. Fibers from the inferior
head enter the bony head of the mandibular condyle.
The lateral pterygoid protrudes the mandible, depresses (opens) it, and moves it
left and right. None of the other muscles listed enter the TMJ.
Medial pterygoid and masseter primarily elevate (close) the mandible, whereas
temporalis elevates (anterior fibers) and retrudes (posterior fibers). Buccinator
forms the substance of the cheek and is also a muscle of facial expression. It is
not a muscle of mastication.

(Q.81) While lying supine in bed eating, a child aspirates a peanut. Which of the following
bronchopulmonary segments would this foreign object most likely enter?
Apical segment of the left upper lobe
(a)

(b)

Apical segment of the right upper lobe

Medial segment of the right middle lobe


(c)
Superior segment of the right lower lobe
(d)
Your Response :
Correct Answer
D
:
Superior segment of the right lower lobe
Exp:
Because the right main bronchus is wider and more vertical than the left, foreign
objects are more likely to be aspirated into the right main bronchus.
The superior segmental bronchus of the lower lobar bronchus is the only
segmental bronchus that exits from the posterior wall of the lobar bronchi.
Therefore, if a patient is supine at the time of aspiration, the object is most likely
to enter the superior segmental bronchus of the lower lobe.
None of the segmental bronchi of the left lung (choice A ) are likely to receive
the object because the object is less likely to enter the left main bronchus.
The apical segment of the right upper lobe is not likely to receive the foreign
object because of the sharp angle that the upper lobar bronchus makes with the
right main bronchus and the sharp angle that the apical segmental bronchus
makes with the lobar bronchus.
The medial segmental bronchus of the right middle lobe arises from the anterior
wall of the right middle lobar bronchus. Therefore, when the patient is supine,
the effect of gravity will tend to prevent the object from entering this segmental
bronchus.

(Q.82) A physician notes weakness of a patient's thumb in extension, although rotation, flexion,
abduction, adduction, and opposition are normal. Which of the following nerves is most likely
involved?
Median and radial
(a)
Median and ulnar
(b)
Median only
(c)
Radial only
(d)
Your Response :
Correct Answer
D
:
Radial only
Exp:
All three of the nerves listed innervate muscles that supply the thumb. Extension
is provided by the extensors pollicis longus and brevis, which are innervated by
the radial nerve.
The median nerve (choices A, B, and C) supplies the thenar group, which allows
the thumb to oppose, flex, abduct, and rotate.
The ulnar nerve (choices E and B) supplies the adductor pollicis, which adducts
the thumb.

(Q.83) From which of the following arteries does the sphenopalatine artery arise?
External carotid
(a)

Facial
(b)
Maxillary
(c)
Ophthalmic
(d)
Your Response :
Correct Answer
C
:
Maxillary
Exp:
The sphenopalatine artery is the terminal branch of the maxillary artery. The
maxillary artery arises from the external carotid artery and then passes through
the infratemporal fossa, giving off branches to structures in this region.
The artery then passes through the pterygomaxillary fissure to enter the
pterygopalatine fossa.
The terminal branch then passes through the sphenopalatine foramen to enter the
nasal cavity and supply much of the nasal mucosa, particularly in the posterior
region of the nasal cavity.
The external carotid artery arises from the common carotid artery at the carotid
bifurcation. The external carotid artery has eight branches that supply the head
and neck region: the superior thyroid, ascending pharyngeal, occipital, lingual,
facial, posterior auricular, maxillary, and superficial temporal arteries.
The facial artery arises from the external carotid artery. It provides blood supply
to much of the facial region. The nasal cavity (particularly the anterior portion)
receives some of its blood supply from branches of the facial artery.
The ophthalmic artery arises from the internal carotid artery immediately after
the internal carotid artery emerges from the cavernous sinus. The ophthalmic
artery passes through the optic canal to supply structures in the orbit.
The transverse facial artery is a branch of the superficial temporal artery. It
supplies blood to structures in the upper portion of the lateral face.

(Q.84) Which of the following helps to anchor an epithelial cell to the basement membrane?
Adherent junction
(a)
Connexon
(b)
Gap junction
(c)
Hemidesmosome
(d)
Your Response :
Correct Answer
D
:
Hemidesmosome
Exp:
Desmosomes are specializations of the lateral surface of the cell formed from the
juxtaposition of two disc-shaped structures in adjacent cells, acting much like
spot welds between the two cells. Hemidesmosomes are basically desmosomes
between a single cell and an extracellular matrix structure, such as a basement
membrane.
Adherent junctions , or zonulaadherens are bandlike junctions that help attach
adjacent epithelial cells to each other.

Gap junctions consist of a hexagonal lattice of tubular proteins called connexons


, which form channels allowing communication between cells.
Tight junctions , or zonulaoccludens, are formed by the fusion of the outer
leaflets of apposed cell membranes on the lateral cell surfaces, just beneath the
apical poles. They form a barrier to permeability or a seal around the cell.

(Q.85) Largest bursa of the body is?


Ilio-psoas burs(a.)
(a)
Suprapatellar burs(a.)
(b)
Subcoracoid burs(a.)
(c)
Olecranon bursa
(d)
Your Response :
Correct Answer
A
:
Ilio-psoas bursa.
Exp:
The iliopsaos bursa is the largest bursa of the human body and is bilaterally
present in 98% of adults.
This bursa is bounded by the musculotendinous junction of the M. iliopsoas
(anteriorly) and by the fibrous capsule of the hip (posteriorly). It extends from
the inguinal ligament superiorly to the lesser trochanter inferiorly and is flanked
by the femoral vessels (medially) and the femoral nerve (laterally).
The bursa is always collapsed, when in healthy condition. In some situations the
bursa is enlarged, and contains fluid. When enlarged, it may cause a variety of
symptoms such as pain, and immobility.
The -est list in Anatomy :
Largest and deepest cranial fossa
Thickest/strongest orbital wall
Thinnest/weakest
Farthest and deepest quadrant of tympanic
membrane
Smallest and most numerous lingual papillae
Largest diploic vein
Largest terminal branch of ophthalmic nerve
Smallest terminal branch of ophthalmic nerve
Largest of the sympathetic cardiac branches
Largest scalenous muscle
Smallest scalenous muscle
Thickest intervertebral disc
Thinnest intervertebral disc
Largest papillary muscle of rt. Ventricle
Rib with the largest obliquity
Shortest,broadest and most curved rib
Largest rib
Thickest and toughest of the meninges
Largest branch of vertebral artery
Largest branch of ICA
Largest horn of lateral ventricle

Posterior
Lateral
Medial
Antero-inferior
Filiform
Occipital diploic vein
Frontal nerve
Lacrimal nerve
Middle cervical cardiac branch
S.medius
S.posterior
At lumbar region
At superior thoracic region
Anterior papillary muscle
9th rib
1st rib
7th rib
Duramater
PICA
MCA
Inferior horn

Largest commissure of brain


Thickest part of corpus callosum
Thickest cutaneous nerve
Largest organnele in eukaryotes
Male urethra : Shortest part
Male urethra : Longest part
Male urethra : widest and most dilatable part
Male urethra : Narrowest and least dilatable
part
Strongest ligament of the body
Largest peripheral parasympathetic ganglion
Largest bursa of the body
Largest joint
Longest muscle
Largest branch of coeliac trunk
Smallest bone and muscle
Largest sesamoid bone
Longest vein

Corpus callosum
Splenium
Greater occipital nerve
Endoplasmic reticulum
Membranous
Penile
Prostatic
Urethral orifice
Ilio-femoral/Bigelow ligament
Sphenopalatine ganglion
Ilio-psoas bursa
Knee
Sartorius
Splenic artery
Stapes and stapedius
Patella
GSV

(Q.8 Which of the following muscle is not supplied by Lateral plantar nerve?
6)

(a)
(b)
(c)
(d)
Your Response :
Correct Answer
B
:
Ex 2
p:

1
2
3
4

(Q.87) Collecting part of kidney of derived from


Metanephros
(a)
Mesonephros
(b)
Ureteric bud
(c)
Wolffian duct
(d)
Your Response :
Correct Answer
C
:
Ureteric bud
Exp:
During the fifth week of gestation, the mesonephric duct develops an
outpouching, the ureteric bud, near its attachment to the cloaca.
As the cranial end of the ureteric bud extends into the intermediate mesoderm
(metanephros), it undergoes a series of branchings to form the collecting duct
system of the kidney. It also forms the major and minor calyces and the renal
pelvis.
The portion of the intermediate mesoderm in contact with the tips of the
branching ureteric bud is known as the metanephrogenic blastema. Signals
released from the ureteric bud induce the differentiation of the metanephrogenic
blastema into the renal tubules (nephrons of kidney).
As the renal tubules grow, they come into contact and join with connecting

tubules of the collecting duct system, forming a continuous passage for flow
from the renal tubule to the collecting duct.

(Q.88) Right fourth arch artery gives rise to


Right subclavian artery
(a)
Common carotid artery
(b)
Internal carotid artery
(c)
External carotid artery
(d)
Your Response :
Correct Answer
A
:
Right subclavian artery
Exp:
The third aortic arch constitutes the commencement of the internal carotid
artery, and is therefore named the carotid arch. External carotid artery is a new
bud from the region of internal carotid & belongs to third arch only.
The fourth right arch forms the right subclavian as far as the origin of its
internal mammary branch; while the fourth left arch constitutes the arch of the
aorta between the origin of the left carotid artery and the termination of the
ductus arteriosus
The fifth arch disappears on both sides
The sixth right arch disappears; the sixth left arch gives off the pulmonary
arteries and forms the ductus arteriosus; this duct remains pervious during the
whole of fetal life, but is obliterated a few days after birth
In the early embryo the right and left arches each gives a branch to the lungs, but
that later both pulmonary arteries take origin from the left arch

(Q.89) The Eustachian tube is made up of ?


Hyaline cartilage.
(a)
Fibrocartilage
(b)
Elastic cartilage
(c)
Fibro-elastic cartilage
(d)
Your Response :
Correct Answer
C
:
Elastic cartilage
Exp:
Cartilage is a form of connective tissue in which the cells and fibers are
embedded in a gel-like matrix, the latter being responsible for its firmness and
resilience. Except on the exposed surfaces in joints, a fibrous membrane called
the perichondrium covers the cartilage. There are three types of cartilage:
Hyaline cartilage has a high proportion of amorphous matrix that has the same
refractive index as the fibers embedded in it. Throughout childhood and
adolescence, it plays an important part in the growth in length of long bones
(epiphyseal plates are composed of hyaline cartilage). It has a great resistance to
wear and covers the articular surfaces of nearly all synovial joints. Hyaline
cartilage is incapable of repair when fractured; the defect is filled with fibrous
tissue.

Fibrocartilage has many collagen fibers embedded in a small amount of matrix


and is found in the discs within joints (e.g., the temporomandibular joint,
sternoclavicular joint, and knee joint) and on the articular surfaces of the clavicle
and mandible. Fibrocartilage, if damaged, repairs itself slowly in a manner
similar to fibrous tissue elsewhere. Joint discs have a poor blood supply and
therefore do not repair themselves when damaged.
Elastic cartilage possesses large numbers of elastic fibers embedded in matrix.
As would be expected, it is flexible and is found in the auricle of the ear, the
external auditory meatus, the auditory tube, and the epiglottis. Elastic cartilage, if
damaged, repairs itself with fibrous tissue.
Hyaline cartilage and fibrocartilage tend to calcify or even ossify in later life.

(Q.90) Not Mesodermal in origin is


Respiratory Tract Lining
(a)
Kidney
(b)
Heart
(c)
Occipital bone
(d)
Your Response :
Correct Answer
A
:
Respiratory Tract Lining
Exp:
Respiratory system* develops from forget and is endodermal
The mesoderm forms: Musculo-Skeletal muscle(MS), Dermis of skin,
Connective Tissue, Urogenital system(UGS) including kidney*, Cardio-vascular
system(CVS) including Heart*, Blood (lymph cells), and Spleen

(Q.91) Following have no lymphatics except


Brain
(a)
Internal Ear
(b)
Dermis
(c)
Eye
(d)
Your Response :
Correct Answer
C
:
Dermis
Exp:
Lymphatic capillaries are absent in the central nervous system, the meninges,
the eyeball (except the conjunctiva), the orbit, the internal ear, within striated
muscle, the bone marrow, the liver lobule, the spleen pulp and kidney
parenchyma. They are entirely absent in cartilage.
The lymphatic fluid is transported to progressively larger lymphatic vessels
culminating in the right lymphatic duct (for lymph from the right upper body)
and the thoracic duct (for the rest of the body); these ducts drain into the
circulatory system at the right and left subclavian veins.
The bone marrow & thymus are the primary lymph organs

Spleen, lymph nodes, Peyers patches, tonsils, vermiform appendix, and red bone
marrow are accessory/secondary lymphoid tissues.

(Q.92) All of the following are categorized as secondary lymphoid organs except
Lymph nodes
(a)
Spleen
(b)
Thymus
(c)
Sub epithelial collections of lymphocytes
(d)
Your Response :
Correct Answer
C
:
Thymus
Exp:
The lymphoid tissue of the body considered collectively; it can be divided into
primary (or central) lymphoid tissues, the thymus and bone marrow where
lymphocytes differentiate from stem cells, and secondary (or peripheral) tissues,
the lymph nodes, spleen, and gut-associated lymphoid tissue (tonsils, Peyer's
patches) where lymphocytes take part in immune responses.

(Q.93) Untrue about articular cartilage is


Covers the articular surfaces of all synovial joints
(a)
Is of hyaline variety in all synovial joints
(b)
No ossification with age
(c)
Is devoid of nerves, vessels and perichondrium
(d)
Your Response :
Correct Answer
B
:
Is of hyaline variety in all synovial joints
Exp:
In synovial joints (like temporomandibular and sterno-clavicular) the articular
surfaces are lined by fibro-cartilage instead of hyaline cartilage.
Hyaline cartilage loses the tendency to ossify with age as it takes the role of
articular cartilage
Hyaline cartilage also loses the perichondrium, as it becomes articular cartilage

(Q.94) Following statements describe the epithelial lining of various body tubes. Choose the incorrect
statement
Trachea: Pseudostratified ciliated columnar
(a)
Ileum: Columnar epithelium
(b)
Mucosa below pectinate line: Keratinized stratified squamous
(c)
Proximal prostatic urethra: Transitional epithelium
(d)
Your Response :
Correct Answer
C
:
Mucosa below pectinate line: Keratinised stratified squamous
Exp:
Mucosa below pectinate line is wet and cannot have keratin. Keratin is present in
the lowermost anal canal, which is lined by true skin.

Stratified squamous, non-keratinised is a special wet* epithelium found at


following sites: exposure of the body tubes to the exterior, especially if there is
friction., e.g., cornea, conjunctiva, nasal openings, oral cavity till oesophagus
(including tongue, tonsils, palate, part of epiglottis), terminal anal canal (pecten),
terminal urethra, vagina & labia minora.
The epithelium of the urethra starts off as transitional cells as it exits the bladder.
Further along the urethra there are stratified columnar* cells, then stratified
squamous cells near the external meatus (exit hole).
EPITHELIAL LINING AT VARIOUS SITES IN THE BODY :
SITE
EPITHELIUM TYPE
Pleura,Pericardium, Peritoneum,
Simple squamous epithelium
Endocardium
Thyroglossal cyst
Pseudostratified columnar and
Squamous epithelium
Alveoli in premature/preterms
Low cuboidal epithelium
Ethmoidal nasal polyp
Early stage : Ciliated columnar
epithelium
Late stage : Transitional and squamous
epithelium(Metaplastic)
Paranasal sinuses
Ciliated columnar epithelium
Nasolabial cysts
Pseudostratified columnar epithelium
Bartholins duct
Multilayered columnar epithelium
Bartholins gland acini
Low columnar / Cuboidal single layer
Endocervix / cervical canal
Ciliated High columnar epithelium
Ectocervix
Squamous epithelium
Endometrium
Columnar epithelium
Adult vagina
Stratified squamous epithelium
Newborn vagina
Transitional epithelium
Endosalpinx
Tall ciliated columnar epithelium
Chocolate cyst of ovary in
Columnar cells with tendency to form
endometriosis
papillae
Vocal cord and upper part of vestibule Stratified squamous epithelium
of larynx
Rest of the larynx
Cilitaed columnar epithelium
Bowmans capsule
Simple squamous epithelium
PCT
Simple cuboidal epithelium with brush
border
DCT
Simple cuboidal epithelium without
brush border
Thin Loop of Henle
Low cuboidal epithelium or squamous
epithelium
Thick LH
Cuboidal epithelium
Collecting duct
Simple cuboidal or columnar epithelium
Proximal urethra
Transitional epithelium
Mid urethra
Pseudostratifid columnar epithelium
Most distal part of urethra
Stratified squamous epithelium
Mouth , oral cavity and esophagus
Stratified squamous nonkeratinized
epithelium
Stomach
Simple columnar epithelium
Small intestine
Columnar epithelium with brush border

Large intestine
Transition zone of anal canal
Anal canal /anal verge
Thyroid follicles
Ovarian germinal epithelium
Gall bladder
Secondary ovarian follicles
Ducts of salivary glands
Sensory epithelium of olfactory area
Trachea and large bronchi

Microvilli
Simple columnar epithelium
Stratified columnar epithelium
Stratified squamous epithelium
Cuboidal cells
Cuboidal cells
Columnar cells with brush border
Stratified cuboidal epithelium
Stratified columnar epithelium
Pseudostratified columnar epithelium
Pseudostratified columnar epithelium

Part of Auditory tube


Ductus deferens

(Q.95) Following statements describe the intercellular junctional complexes. Choose the incorrect
statement
Zona occludens is also known as tight junctions
(a)
Zona adherens are the spot-welds 'Desmosomes'
(b)
Hemidesmosomes use integrin as cell adhesion molecule
(c)
Connexons are the integral component of gap-junction
(d)
Your Response :
Correct Answer
B
:
Zona adherens are the spot-welds 'Desmosomes'
Exp:
It is the macula adherens, which are known asspot-welds 'Desmosomes'
Tight junctions, or zonulaoccludens, are the closely associated areas of two
cells whose membranes join together forming a virtual impermeable barrier to
fluid.
Desmosome, also known as macula adherens (adhering spot), is a cell structure
specialized for cell-to-cell adhesion.
The cell adhesion proteins of the desmosome are members of the cadherin family
of cell adhesion molecules.
Hemidesmosomes
When visualized by electron microscopy, hemidesmosomes are similar in
appearance to desmosomes. Rather than linking two cells, hemidesmosomes
attach one cell to the extracellular matrix.
Rather than using cadherins, hemidesmosomes use integrin cell adhesion
proteins.

(Q.96) Choose the incorrect statement about a developing long bone


Growth occurs maximum at metaphysis
(a)
Metaphysis is the epiphyseal end of diaphysis
(b)
Osteoprogenitor cells divide to form osteoblasts which become osteocyte later
(c)
Haversian system &Volkman canals are develop in diaphysis
(d)

Your Response :
Correct Answer
A
:
Growth occurs maximum at metaphysis
Exp:
Growth occurs maximum at epiphyseal (growth) plate
The metaphysis is the body of cartilage that separates the epiphyses and the
diaphysis of long bones during growth.
It is this part of the bone that grows during childhood; as it grows, it ossifies near
the diaphysis and the epiphyses.
Osteons (also called Haversian system) are predominant structures found in
some lamellar or compact bone.
In the center of the osteon is a central canal, called the Haversian canal.
Osteocytes are found between concentric lamellae and connected to each other
and the central canal by cytoplasmic processes through the canals called
canaliculi. This network permits the exchange of nutrients and metabolic waste.
Osteons are connected to each other and the periosteum by oblique channels
called Volkmann's canals.

(Q.97) Following statements describe characteristics features of various epiphyses. Choose the incorrect
statement

(a)

Os trigonum is an extended tubercle at the back of talus sometimes occurring as a


separate bone
Mastoid process is a type of traction epiphysis

(b)
Head of 1st metacarpal is an aberrant epiphysis
(c)
Pressure epiphysis ossifies later than traction epiphysis
(d)
Your Response :
Correct Answer
D
:
Pressure epiphysis ossifies later than traction epiphysis
Exp:

Atavistic epiphysis: This kind of epiphysis is a part of the bone which exists as a
separate bone in lower animals; but in humans it has fused with the main bone
(since it has lost its individual function). E.g., os trigonum, coracoid process.
Pressure epiphysis: The part of bone found at the articular end, which is
supposed to be involved in transmission of weight and is within the joint
intracapsular.
Traction epiphysis: The part of bone which is produced due to traction by
some muscles on the cartilage after birth. It is almost always extra-capsular
(since, muscles usually do not go unto capsule !) and ossifies later than the
pressure epiphysis
Aberrant epiphysis: The epiphysis which may not be developing in some
fraction of population. E.g., head of 1st metatarsal (Thumb)

(Q.98) The second costochondral joint is a

Fibrous joint
(a)
Synovial joint
(b)
Synchondrosis
(c)
Syndesmosis
(d)
Your Response :
Correct Answer
C
:
Synchondrosis
Exp:
The costochondral joints are the articulations between the ribs and costal
cartilage. All of them are hyaline cartilaginous (synchondrosis) joints.
Each rib has a depression shaped like a cup that the costal cartilage articulates
with. There is normally very minimal movement at these joints.
The chondrosternal articulations of the first rib is primary cartilagenous
(synchondrosis)
Articulations of the cartilages of the ribs 2-7 with the sternum are plane synovial
joints

(Q.99) True statement regarding branchial anomalies


Most commonly second arch is involved
(a)
Cyst is less common than sinus
(b)
No 1stth arch anomalies are seen
(c)
Internal draining sinuses are most common
(d)
Your Response :
Correct Answer
A
:
Most commonly second arch is involved
Exp:
Branchial cleft anomaliescomprise of a spectrum of congenital defects that occur
in the head and neck. The anomalies result frombranchial apparati(6 arches)
which are the embryologic precursors of the earand the muscles, blood vessels,
bones, cartilage, and mucosallining of the face, neck, and pharynx1.
The 6 branchial arches give rise to 5 clefts.
The range of anomalies can include either a
cyst:no internal or external communication
fistula:communicates both internally and externally
sinus: incomplete tract
Cysts are considered to be commoner than the other two entities by a ratio of
~2:16
Among fistulae and sinuses the order prevalence is thought to be: external
draining sinus > complete fistula > internal draining sinus6; although some
anomalies can occur in combination.
The complete list therefore includes:
first branchial cleft anomalies (5-8%)1-8
first branchial cleft cyst

first branchial cleft fistula


first branchial cleft sinus
second branchial cleft anomalies (commonest by far: 90-95%)8
second branchial cleft cyst
second branchial cleft fistula
second branchial cleft sinus
third branchial cleft anomalies (rare)
third branchial cleft cyst
third branchial cleft fistula
third branchial cleft sinus
fourth branchial cleft anomalies (rare)
fourth branchial cleft cyst
fourth branchial cleft fistula
fourth branchial cleft sinus
the fifth cleft does not give rise to thecervical sinus of Hiswhich is part of the
reason that there are no fifth branchial cleft anomalies
The 3rdand 4thbranchial arches tend to be very close and therefore the distinction
between these two cleft anomalies can be difficult on imaging

(Q.100) Choose the incorrect statement regarding structures constituting the anatomical angina
pectoris

(a)
(b)
(c)
(d)

The pain fibres from heart run in the middle and inferior cervical cardiac
branches of sympathetic trunk
The superficial cardiac plexus receives left superior cervical cardiac branch
from vagus nerve
Sympathetic fibres come from cervical and upper thoracic ganglia of
sympathetic trunk to carry pain fibres
Intercostobrachial nerve is the lateral cutaneous branch of anterior primary rami
of T2

Your Response
:
Correct Answer
B
:
The superficial cardiac plexus receives left superior cervical cardiac
Exp:
branch from vagus nerve
The superficial cardiac plexus receives left superior cervical cardiac branch
from sympathetic trunk, whereas, inferior branch is from vagus.

(Q.101) Following are the structure passing through foramen magnum except
Cerebellar tonsils
(a)
Spinal cord
(b)
Spinal accessory nerve
(c)
Vertebral artery
(d)
Your Response
:
Correct Answer
B
:
Spinal cord
Exp:
Structure crossing foramen magnum is lower medulla.
The Spinal cord begins at the upper border of atlas vertebra.

(Q.102) Following muscles are the abductors of eye except


Superior oblique
(a)
Superior rectus
(b)
Inferior oblique
(c)
Lateral rectus
(d)
Your Response
:
Correct Answer
B
:
Superior rectus
Exp:
The superior rectus muscle is one of the extraocular muscles.
It is innervated by the superior division of the oculomotor nerve.
In the primary position (looking straight ahead), the superior rectus muscle's
primary function is elevation, although it also contributes to intorsion and
adduction.
Superior oblique: The primary action of the superior oblique muscle is
intorsion; the secondary action is depression (primarily in the adducted
position); the tertiary action is abduction (DOI).
Both the obliques are abductors

(Q.103) X marked aperture is for the passage of

(a)
(b)
(c)
(d)
Your Response
:
Correct Answer
:
Exp:

Anterior interosseous nerve


Anterior interosseous artery
Posterior interosseous nerve
Posterior interosseous artery

B
. Anterior interosseous artery
Interosseous membrane
The interosseous membrane is a broad, thin, collagenous sheet. Its fibres slant
distomedially between the radial and ulnar interosseous borders, and its distal
part is attached to the posterior division of the radial border. Two or three
posterior bands occasionally descend distolaterally across the other fibres. The
membrane is deficient proximally, starting 2 or 3 cm distal to the radial
tuberosity, and broader at midlevel. An oval aperture near its distal margin
conducts the anterior interosseous vessels to the back of the forearm, and
the posterior interosseous vessels pass through a gap between its proximal
border and the oblique cord.

(Q.104) Subdural haemorrhage has occurred due to venous bleeds. Which veins are involved
Bridging
(a)
Middle meningeal
(b)
Subarachnoid
(c)
Pterygoid
(d)
Your Response
:
Correct Answer
A
:
Bridging
Exp:
Subdural haemorrhages result from rupture of cortical bridging veins. These
connect the venous system of the brain to the large intradural venous sinuses
and lie relatively unprotected in the subdural space.

The bridging veins drain the superior cerebral region and are also called as
superior cerebral veins

(Q.105) Obstruction to the flow of CSF at the foramen of Monro will most likely lead to the
enlargement of?

(a)
(b)
(c)
(d)
Your Response
:
Correct Answer
:
Exp:

Only lateral ventricle


Only third ventricle
Both lateral and third ventricles
Fourth ventricle

A
Only lateral ventricle
CSF formation & circulation: CSF is produced predominantly by the choroid
plexus of the lateral*, followed by third and fourth ventricles.
Flow is caudal through the ventricular system Lateral ventricles to aqueduct
of Sylvius via foramina of Monro. Then it reaches fourth ventricle and move
into the subarachnoid space through the foramina of Luschka and Magendie
(on the roof of fourth ventricle).
Hydrocephalus is a congenital or acquired condition characterised by marked
dilatation of the cerebral ventricles, accompanied by the accumulation of
cerebrospinal fluid within the skull.
More commonly it results from impaired absorption of CSF than from
excessive secretion.
Aetiology: atresia, stenosis, forking, or gliosis of the aqueduct of Sylvius
both lateral and third ventricles are dilated*
Atresia of foramina of Magendie or Luschka - the Dandy-Walker syndrome
Obstruction of 4th ventricular outflow - Arnold Chiari malformation
If the obstruction is only at foramen of Monro then the lateral ventricle of the
same side will show dilatation.

(Q.106) Arachnoid villi are finger like projections from


Subarachnoid space into a space between two layers of duramater
(a)
Subarachnoid space into a space exterior to duramater
(b)
Subarachnoid space into a space interior to duramater
(c)
A space between two layers of duramater into subarachnoid space
(d)
Your Response
:
Correct Answer
A
:
Subarachnoid space into a space between two layers of duramater
Exp:

Arachnoid villi drain the CSF from the subarachnoid space into the dural
venous sinuses.
Dural venous sinus is a venous space between the two layers of duramater,
namely, endosteal & meningeal layers.

(Q.107) The taste pathway from palate goes through


Chorda tympani branch of facial nerve
(a)
Greater petrosal nerve branch of facial nerve
(b)
Palatine branch of Vagus nerve
(c)
Palatine branch of Glossopharyngeal nerve
(d)
Your Response
:
Correct Answer
B
:
Greater petrosal nerve branch of facial nerve
Exp:
Palatine taste is carried by the greater petrosal branch of facial nerve towards
the geniculate ganglion
Chorda tympani carry the taste fibres from anterior 2/3 of tongue towards the
same Geniculate ganglion.

(Q.108) Choose the correct statement about facial nerve and its branches
Greater petrosal nerve arises distal to the nerve to stapedius
(a)
Chorda tympani contains general somatic efferent fibres
(b)
Posterior belly of digastric is supplied by general visceral efferent component
(c)
(d)

Chorda tympani is related topographically with mandibular division of


trigeminal nerve

Your Response
:
Correct Answer
D
:
Chorda tympani is related topographically with mandibular division of
Exp:
trigeminal nerve
Facial nerve gives three branches in the facial canal in a proximo-distal
sequence: greater petrosal, nerve to stapedius & chorda tympani
Greater petrosal (GP) & chorda tympani (CT) nerves carry dual fibre
(GVE&SVA), whereas, nerve to stapedius carries the SVE fibres.
SVE fibres carry motor supply to the branchial arch musculature e.g., stapedius,
posterior belly of digastric*
GVE deals with the autonomic nervous system*, for e.g., GP nerve supplies
preganglionic parasympathetic fibres to lacrimal gland & CT nerve supplies
sub-mandibular salivary gland
SVA deals with sensations like taste, for e.g., palatal taste is carried by the GP
nerve & anterior tongue taste by the CT nerve

The chorda tympani is a nerve that branches from the facial nerve (cranial
nerve VII) inside the facial canal, just before the facial nerve exits the skull via
the stylomastoid foramen. The chorda tympani carries two types of nerve fibers
from their origin with the facial nerve to the lingual nerve that carries them to
their destinations:
Special sensory fibers providing taste sensation from the anterior two-third of
the tongue.
Presynaptic parasympathetic fibers to the submandibular ganglion, providing
secretomotor innervation to two salivary glands: the submandibular gland and
sublingual gland.
The CT nerve continues through the petrotympanic fissure, after which it
emerges from the skull into the infratemporal fossa. It soon combines with the
larger lingual nerve, a branch of the mandibular nerve (cranial nerve V3)
topographically (location wise) related*.

(Q.109) A patient arrives in the emergency room after having suffered severe head trauma in a
motorcycle accident. Radiographic studies of the head reveal a basilar skull fracture in the
region of the foramen ovale. Which of the following functional losses would most likely be
related to this injury
Loss of abduction of the eye
(a)

(b)
(c)
(d)
Your Response
:
Correct Answer
:
Exp:

Loss of sensation over the forehead


Loss of sensation over the zygoma
Paralysis of anterior belly of digastric

D
Paralysis of anterior belly of digastric
Mandibular nerve passes through the foramen ovale & supplies the muscles of
mandible like anterior belly of digastric.
Foramen ovale is present in the greater wing of sphenoid, which is present in
the base of skull

(Q.110) Petits ligament is located at


Neck
(a)
Upper limb
(b)
Lower limb
(c)
Pelvis
(d)
Your Response
:
Correct Answer
D
:
Pelvis
Exp:
SOME SPECIFIC NAMED LIGAMENTS IN THE BODY :
Weitbrechts ligament The oblique cord connecting the ulna and the radius
Pouparts ligament
Inguinal ligament

Petits ligament

Gimbernats ligament
Mackenrodts ligament
Lisfrancs ligament
Henles ligament
Lockwoods ligament

Thickened portion of Pelvic fascia between the cervix


and vagina. Passes posteriorly in the recto-Uterine
fold to attach to the Ant. surface of sacrum
Lacunar ligament which forms the medial portion of
the femoral ring
Transverse cervical ligament-important support of the
uterus
Ligament joining the cuneiform bone of ankle to the
2nd metatarsal.
The conjoined tendon of the transverses abdominis
muscle
Suspensory ligament of the eyeball

(Q.111) Which of the following is true regarding gastrulation?


Establishes all the three germ layers
(a)
Occurs at the caudal end of the embryo prior to its cephalic end
(b)
Involves the hypoblastic cells of inner cell mass
(c)
Usually occurs at 4 weeks
(d)
Your Response
:
Correct Answer
A
:
Establishes all the three germ layers
Exp:
(Ref. Human Histology by IB Singh, 7th ed., pg41; Langmans embryology
9th/pg. 65)
Gastrulation
It is the most characteristic event occurring during the third week of gestation
Gastrulation is process that establishes all three germ layers (ectoderm,
mesoderm, and endoderm) in the
embryo.
It begins with formation of the primitive streak on the surface of the epiblast.
Initially, the streak is vaguely defined, but in a 15- to 16-day embryo, it is
clearly visible as a narrow groove with slightly bulging regions on either side.
The cephalic end of the streak, the primitive node, consists of a slightly elevated
area surrounding the small primitive pit. Cells of the epiblast migrate toward
the primitive streak. Upon arrival in the region of the streak, they become flaskshaped, detach from the epiblast, and slip beneath it. This inward movement is
known as invagination.
Once the cells have invaginated, some displace the hypoblast, creating the
embryonic endoderm, and others come to lie between the epiblast and newly
created endoderm to form mesoderm. Cells remaining in the epiblast then form
ectoderm.
Thus, the epiblast, through the process of gastrulation, is the source of all of the
germ layers, and cells in these layers will give rise to all of the tissues and
organs in the embryo.

(Q.112) All the following features are seen in neurons from dorsal root ganglia, except:
They have centrally located nuclei
(a)
They are derived from neural crest cells
(b)
They are multipolar
(c)
They contain lipofuscin granules
(d)
Your Response
:
Correct Answer
C
:
They are multipolar
Exp:
(Ref. IB Singh embryology 7th/pg. 312; IBsingh histology 4th/pg. 164)
DRG or spinal ganglia are unipolar or pseudounipolar.
Sympathetic or autonomic ganglia are multipolar.
DRG (sensory or spinal
Autonomic sympathetic ganglion
ganglion)
1. Consist of pseudounipolar Consist of multipolar neurons
neurons
2. Has cell bodies of afferent Has cell bodies of efferent neurons.
neurons
3. Neurons arranged in groups Neurons and nerve fibers are scattered.
separated by nerve fibers in
groups
4. Large rounded cell body
Cell body is smaller and irregular with eccentric
with central nucleus and
nucleus and prominent nucleolus.
prominent nucleolus
Neurons of sensory spinal nerve root ganglion, sympathetic (autonomic)
ganglia and sensory ganglia of 5th to 10th cranial nerves except 6th are derived
from neural crest.

(Q.113) The weight of the upper limb is transmitted to the axial skeleton by:
Coracoclavicular ligament
(a)
Coracoacromial ligament
(b)
Costoclavicular ligament
(c)
Coracohumeral ligament
(d)
Your Response
:
Correct Answer
A
:
Coracoclavicular ligament
Exp:
(Ref. Keith Moore clinical anatomy 4th/pg. 787; BDC vol-1. 3rd/pg. 132)
The weight of the upper limb is transmitted to the axial skeleton by
Coracoclavicular ligament.
Coracoclavicular If this ligament tore, the shoulder would fall away from the
clavicle.
Acromioclavicular The AC joint is weak and easily injured by a direct blow.
Coracoacromial
The coracoacromial arch is so strong that the humeral body

Costoclavicular

or clavicle would fracture first.


This is located between the first rib and the clavicle, not
near the acromion.

(Q.114) The superficial external pudendal artery is a branch of:


Femoral artery
(a)
External iliac artery
(b)
Internal iliac artery
(c)
Aorta
(d)
Your Response
:
Correct Answer
A
:
Femoral artery.
Exp:
The superficial external pudendal artery is a branch of Femoral artery.
Also know:
Branches of internal pudendal artery, including transverse labial perineal artery,
artery to vestibular bulb and deep and dorsal artery to clitoris, chiefly supply
vulva.

(Q.115) Diaphragmatic hernia can occur through all the following, except:
Esophageal opening
(a)
Costovertebral triangle
(b)
Costal and sternal attachment of diaphragm
(c)
Inferior vena cava opening
(d)
Your Response
:
Correct Answer
D
:
Inferior vena cava opening
Exp:
(Ref. Nelson Paediatrics 18th/pg. 746)
Diaphragmatic hernia
A diaphragmatic hernia is defined as a communication between the abdominal
and thoracic cavities with or without abdominal contents in chest.
The defect may be at the esophageal hiatus (hiatal), paraesophageal (adjacent to
hiatus), retrosternal (maorgagni) or at the posterolateral portion (Bochdalek) of
the diaphragm.

(Q.116) Sites of Anatomical Ureteric constrictions are following except:


Ureteropelvic junction
(a)
Ureterovesical junction
(b)
Crossing of iliac artery
(c)
Ischial spine
(d)
Your Response

:
Correct Answer
D
:
Ischial spine
Exp:
(Ref. Bailey and Love 24th/ Fig. 64.2; pg. 13218)
The potential sites of stone impaction in ureter include:
UPJ
Ureter crossing the pelvic brim (iliac crossing)
Juxtaposition of vas deferens or broad ligament.
UVJ
Ureteric orifice.

(Q.117) All the following are true regarding blood supply to the kidney, except:
Stellate veins drain superficial zone
(a)
It is site of portosystemic anastomosis
(b)
The renal artery divides into five segmental arteries before entering the hilum
(c)
Its segmental arteries are end-arteries
(d)
Your Response
:
Correct Answer
B
:
It is site of portosystemic anastomosis
Exp:
(Ref. Snells anatomy pg.122/;Keith Moore clinical anatomy 4th/pg. 286)
PortalSystemic Anastomoses
Under normal conditions, the portal venous blood traverses the liver and drains
into the inferior vena cava of the systemic venous circulation by way of the
hepatic veins. This is the direct route. However, other, smaller communications
exist between the portal and systemic systems, and they become important
when the direct route be- comes blocked.
These communications are as follows:
At the lower third of the esophagus, the esophageal branches of the left gastric
vein (portal tributary) anastomose with the esophageal veins draining the
middle third of the esophagus into the azygos veins (systemic tributary).
Halfway down the anal canal, the superior rectal veins (portal tributary)
draining the upper half of the anal canal anastomose with the middle and
inferior rectal veins (systemic tributaries), which are tributaries of the internal
iliac and internal pudendal veins, respectively.
The paraumbilical veins connect the left branch of the portal vein with the
superficial veins of the anterior abdominal wall (systemic tributaries). The

paraumbilical veins travel in the falciform ligament and accompany the


ligamentum teres.
The veins of the ascending colon, descending colon, duodenum, pancreas, and
liver (portal tributary) anastomose with the renal, lumbar, and phrenic veins
(systemic tributaries).

(Q.118) A patient with external hemorrhoids develops pain while passing stools. The nerve mediating
this pain is:

(a)
(b)
(c)
(d)
Your Response
:
Correct Answer
:
Exp:

Hypogastric nerve
Pudendal nerve
Splanchnic visceral nerve
Sympathetic plexus

B
Pudendal nerve
(Ref. BDC 4th/pg. 383; Keith Moore clinical anatomy 4th/pg. 347)
Belo pectinate line, where external hemorrhoids occur the nerve supply is by
inferior rectal nerve, a branch of pudendal nerve (S2,3,4).
Above pectinate line, where internal hemorrhoids occur, pelvic splanchnic
nerves (S2, S3, S4) provide parasympathetic supply while inferior hypogastric
plexus (L1,L2) provide sympathetic supply.
External anal sphincter is supplied by inferior rectal nerve, while the internal
sphincter contract by sympathetic and relax by parasympathetic nerves.

(Q.119) Which of the following muscles is supplied by mandibular nerve:


Masseter
(a)
Buccinator
(b)
Tensor veli palati
(c)
Posterior belly of digastric
(d)
Your Response
:
Correct Answer
A
:
Masseter.
Exp:
(Ref. Anatomy by BD Chaurasia, Vol. III, 3 rd ed., pg 115)
This question was wrongly asked at one of the prestigious institutes and carries
two answers, hence we need to choose any of the two.
Mandibular nerve supplies the masseter muscle as well as Tensor veli palati.
Masseter has been taken as the best answer because it is more frequently
discussed muscle as compared with the other.

The anterior trunk gives off:


A sensory branch, the buccal nerve, which supplies part of the skin of the cheek
and the mucous membrane on its inner aspect; and
Motor branches to the masseter, temporalis and lateral pterygoid muscles.
The posterior trunk, which is principally sensory, divides into three
branches:
The auriculotemporal nerve, which conveys sensory bres to the skin of the
temple and auricle and secretomotor bres from the otic ganglion to the parotid
gland;
The lingual nerve, which passes downwards under cover of the ramus of
the mandible to the side of the tongue where it supplies the
mucous membrane of the oor of the mouth, the anterior twothirds of the tongue (including the taste buds by way of bres which join it
from the chorda tympani), and the sublingual and submandibular salivary
glands;
The inferior alveolar (dental) nerve, which passes down into the mandibular
canal and supplies branches to the teeth of the lower jaw. It then emerges from
the mental foramen to supply the skin of the chin and lower lip. This branch
also conveys the only motor component of the posterior trunk: the nerve to the
mylohyoid, supplying the muscle of that name and the anterior belly of the
digastric.

(Q.120) The sensory supply of the palate is through all of the following, except:
Facial nerve
(a)
Hypoglossal nerve
(b)
Glossopharyngeal nerve
(c)
Maxillary division of trigeminal nerve
(d)
Your Response
:
Correct Answer
B
:
Hypoglossal nerve
Exp:
(Ref. BDC 4thVol 3/pg. 211)
Nerve supply of soft palate:
Palate is supplied by maxillary and mandibular (branches of trigeminal nerve),
facial and glossopharyngeal nerves.
General sensory: maxillary nerve through pterygopalatine ganglion
Special sensory (gustatory): Facial nerve (lesser palatine greater petrosal
geniculate ganglion nucleus of tractus solitorius )
Secretomotor: Lesser palatine Greater petrosal nerve Superior salivatory
nucleus.

- All muscles of soft palate except tensor veli palatini (mandibular nerve) are
supplied by cranial part of accessory nerve (pharyngeal plexus).

(Q.121) All of the following are features of large intestine, except:


Large intestine secretes acidic mucus, which helps in formation of stools
(a)
It is a site of mucocutaneous junction
(b)
Its epithelium contains goblet cells in large numbers
(c)
Absorbs salt and water
(d)
Your Response
:
Correct Answer
A
:
Large intestine secretes acidic mucus, which helps in formation of stools
Exp:
(Ref. BDC 4thVol 3/pg. 211)
Large intestine:
Secretions of large bowel are alkaline (not acidic).
Large bowel extends from IC junction to the anus (site for mucocutaneous
junction).
Villi are absent, but its epithelium is absorptive.
Payers patches are also absent.
Adequate lubrication for passage of the contents is provided by the mucin
secreting goblet cells.
Following are present in large intestine:
Sacculations,
Appendices epiploicae and
Teniae coli
Appendices epiploicae are small peritoneal outpouchings filled with fat found
all over large bowel except at:
Caecum
Appendix
Rectum
Taenia coli are three ribbon-like bands of longitudinal muscles coat of large
bowel. Proximally starts at base of appendix and distally spread upto terminal
sigmoid colon to become continuous with longitudinal muscle coat of rectum.

(Q.122) In flexion and abduction of shoulder all of the following structures are compressed except:

(a)
(b)
(c)
(d)
Your Response
:
Correct Answer
:
Exp:

Subacromial bursa
Long head of biceps
Suprascapular nerve
Supraspinatus tendon

C
Suprascapular nerve
(Ref. Ebnezar textbook of Orthopedics 2nd/pg. 192)
Painful arc syndrome or Impingement syndrome
- Impingement of following structures beneath coracoacromial arch, between
undersurface of acromion and head and greater tuberosity of humerus.
a.

Supraspinatous tendon

b.

Long head of biceps

c.

Subacromial bursa

Etiology:

a. Partial tear, inflammation or calcific deposits involving Rotator cuff


muscles, bicipital tendon,
subacromialsubdeltoid bursitis or fracture of
greater tuberosity.

(Q.123) The least common type of female pelvis is?


Gynecoid
(a)
Android
(b)
Anthropoid
(c)
Platypelloid
(d)
Your Response
:
Correct Answer
D
:
Platypelloid
Exp:
The Female Pelvis
Deformities of the pelvis may be responsible for dystocia (difficult labor). A
contracted pelvis may obstruct the normal passage of the fetus. It may be
indirectly responsible for dystocia by causing conditions such as
malpresentation or malposition of the fetus, premature rupture of the fetal
membranes, and uterine inertia.
The cause of pelvic deformities may be congenital (rare) or acquired from
disease, poor posture, or fractures caused by injury.
Pelvic deformities are more common in women who have grown up in a poor

environment and are undernourished. It is probable that these women suffered


in their youth from minor degrees of rickets.
In 1933, Caldwell and Moloy classified pelves into four groups: gynecoid,
android, anthropoid, and platypelloid (Fig. 6-5C).
The gynecoid type, present in about 41% of women, is the typical female
pelvis, which was previously described.
The android type, present in about 33% of white females and 16% of black
females, is the male or funnel-shaped pelvis with a contracted outlet.
The anthropoid type, present in about 24% of white females and 41% of black
females, is long, narrow, and oval shaped.
The platypelloid type, present in only about 2% of women, is a wide pelvis
flattened at the brim, with the promontory of the sacrum pushed forward

(Q.12 Wagner and Grossmann hypothesis concerns ?


Recurrent laryngeal nerve palsy
(a)
4)
Nerve degeneration
(b)
Bone density modulation in response to stress
(c)
Joint innervation
(d)
Your Response :
Correct Answer
A
:
Ex Recurrent laryngeal nerve palsy
p:
Laws/Formulations/Concepts in Anatomy :
Hiltons law : Nerve supply to a muscle which lies across a joint not only supplies that muscle
but also supllies the joint underneath and the skin overlying the muscle.
Lasts formulation : 4 contiguous spinal segments regulate the movement of a joint. Upper 2
segments control one movements and lower 2 segments regulate the opposite movement.
Wolffs law : A law stating that bone density changes in response to changes in the functional
forces on the bone. Wolff (1836-1902) proposed that changes in the form and function of bones,
or changes in function alone, are followed by changes in the internal structure and shape of the
bone in accordance with mathematical laws. Thus, in mature bone where the general form is
established, the bone elements place or displace themselves, and decrease or increase their mass,
in response to the mechanical demands imposed on them. The theory is supported by the
observation that bones atrophy when they are not mechanically stressed and hypertrophy when
they are stressed. Although Wolff's proposal relates specifically to bone, the law has also been
applied to other connective tissues such as ligaments and tendons.
Gardners concept of joint innervation : Each nerve innervates a specific region of the joint
capsule and that the part of the capsule which is made taut by a given muscle is innervated by
the nerve supplying its antagonist.
Du Bois formula : for calculating surface area of the body.
A=WxHx
71.84

where W = weight in kg, H = height in cm.


Neuron theory of Waldeyer : Contact between neurons is by contiguity and not by continuity.
Semonslaw : In progressive lesions of the recurrent laryngeal nerve abductor muscles of larynx
(Post crico-arytenoid) are 1st to be paralysed last to recover as compared to adductors. In
functional paralysis of recurrent laryngeal nerve adductors are 1stparalysed and recover the last.
Wallens law of degeneration : If a spinal nerve is completely divided the distal portion
undergoes fatty degeneration.
Wagner and Grossmans hypothesis : In recurrent laryngeal nerve paralysis the cricothyroid
muscle is spared which keeps the cords in paramedian position due to its adductor action.

(Q.125) Manubrio-sternal joint is a ?


Synchondrosis
(a)
Symphysis
(b)
Synovial
(c)
Gomphoses
(d)
Your Response
:
Correct Answer
B
:
Symphysis
Exp:
TYPES OF JOINTS IN BODY : HEAD-TO-TOE :
JOINT
TYPE
Temporomandibular joints
Condylar (Bicondylar) synovial joint
Cricothyroid and cricoareytenoid joint Synovial joint
Atlanto-occipital joint, Wrist joint and Ellipsoid joint
Metacarpo-phalangeal joint
Sternoclavicular joint
Saddle (sellar) joint
Ear ossicles
Saddle joint
Melleo-Incal joint

Ball and socket joint

Incudo-stapedial joint

Syndesmosis

Stapes footplate
Joint between ala of vomer and
Rostrum of sphenoid
Costo-vertebral joint
Costo-transverse joint
1stchondrosternal joint,
All Costochondral joints,
Spheno-occipital joint
Joint between epiphysis and diaphysis
of growing bone

Schindylesis (Wedge and groove


suture)
Plane synovial joint
Synovial joint
Primary cartilaginous joint /
Synchondrosis / Hyaline cartilaginous
joint.

2nd 7thchondrosternal joint


5th 9th costal cartilage articulation
10th costal cartilage is united to 9th
costal cartilage by
Superior and inferior Radio-ulnar joints
Median Atlanto-axial joint
Middle radio-ulnar joint
Sterno-clavicular joint
Elbow and Ankle joint
Hip joint
Knee joint

Synovial joint
Synovial joint
Fibrous tissue.
Pivot (Trochoid) joint

Syndesmosis
Saddle joint
Hinge synovial joint
Ball and socket synovial joint
Compound synovial joint
2 condylar joints between medial and
lateral condyles of femur and tibia.

Superior Tibio-fibular joint


Middle Tibio-fibular joint
Inferior Tibio-fibular joint
Talo-calcaneal / Subtalar joint
Calcaneo-cuboid joint
Talo-calcaneo-navicular joint
Other intertarsal joints
Symphysis pubis

1 saddle joint between femur and


patella.
Plane synovial joint
Fibrous joint
Syndesmosis`
Plane synovial joint
Saddle joint
Ball and socket synovial joint
Plane synovial joint
Secondary cartilaginous joint /
Symphyses / Fibrocartilagenous joint

Sacro-coccygeal joint
Manubriosternal joint
Intervertebral joints

(Q.126) Cadaveric position of vocal cords is seen in?


Incomplete unilateral recurrent laryngeal nerve palsy.
(a)
Complete unilateral recurrent laryngeal nerve palsy
(b)
Bilalateral recurrent laryngeal nerve palsy.
(c)
Superior with recurrent laryngeal nerve palsy.
(d)
Your Response
:
Correct Answer
D
:
Superior with recurrent laryngeal nerve palsy.
(Ref. Bhargaw 7th /pg. 306)
Exp:
Note that superior laryngeal nerve palsy usually occurs in combination with recurrent
laryngeal nerve palsy.
PARALYSED EXTENT OF
POSITION VOICE RESPIRATION SWALLOWING
LARYNGEAL INVOLVEMENT OF
NERVE
VOCAL
CORD
Unilateral
Incomplete
Median
Normal Normal
Normal
Recurrent L N
Unilateral
Complete
Paramedian Hoarse Normal
Normal

Recurrent L N
Bilateral
Recurrent L N
Superior with Unilateral or
Recurrent L N bilateral

or
Normal
Median or Normal Dyspnoea
Paramedian
Cadaveric, Feeble Normal
Slack,
or
Wavy
Rough

Normal
Inhalation into
larynx

(Q.127) Most common nerve injured in scapholunate dislocation is?


Radial nerve
(a)
Ulnar nerve
(b)
Musculocutaneous nerve
(c)
Median nerve
(d)
Your Response
:
Correct Answer
D
:
Median nerve
Exp:
Most common nerve injured in
supracondylar fracture of humerus
Tardy ulnar nerve palsy
Most Common Nerve involved in anterior
shoulder dislocation
Most Common Nerve involved in surgical neck
humerus fracture
Most Common Nerve involved in fracture shaft
of Humerus
Most Common Nerve involved in fracture
medial condyle of humerus
Most Common Nerve involved in Monteggia
fracture-dislocation
Most Common Nerve involved in scapholunate
dislocation

Anterior interosseous nerve


Cubitus valgus
Axillary (circumflex humeral)
nerve
Axillary nerve
Radial nerve
Ulnar nerve
Posterior interosseous nerve
Median nerve

(Q.128) Anal triangle does not include


Pudendal nerve
(a)
Anal canal
(b)
Bulbospongiosus
(c)
External anal sphincter
(d)
Your Response
:
Correct Answer
C
:
Bulbospongiosus
Exp:
Bulbospongiosus is one of the superficial muscles of the perineum (urogenital
triangle).
In males, it covers the bulb of the penis. In females, it covers the vestibular
bulb.
In both sexes, it is innervated by the deep/muscular branch of the perineal

nerve, which is a branch of the pudendal nerve.


In males it contributes to erection, ejaculation, and the feelings of orgasm.
In females it closes the vagina and contributes to the feelings of orgasm.
This muscle serves to empty the canal of the urethra, after the bladder has
expelled its contents; during the greater part of the act of micturition its fibers
are relaxed, and it only comes into action at the end of the process.
The anal triangle can be defined either by itsverticesor its sides.
Vertices
one vertex at thecoccyxbone
the twoischial tuberositiesof thepelvic bone
Sides
perineal membrane(posterior border of perineal membrane forms anterior
border of anal triangle)
the twosacrotuberous ligaments
Contents :
Some components of the anal triangle include:
Ischioanal fossa
Anococcygeal body
Sacrotuberous ligament
Sacrospinous ligament
Pudendal nerve
Internal pudendal arteryandInternal pudendal vein
Anal canal
Muscles
Sphincter ani externus muscle
Gluteus maximus muscle
Obturator internus muscle
Levator ani muscle
Coccygeus muscle

(Q.129) All of the following bones of the cranio-facial skeleton are paired except?
Parietal bone
(a)

Temporal bone
(b)
Palatine bone
(c)
Ethmoid bone
(d)
Your Response
:
Correct Answer
D
:
Ethmoid bone
Exp:
Bones of the Skull
Composition
The skull is composed of several separate bones united at immobile joints
called sutures. The connective tissue between the bones is called a sutural
ligament. The mandible is an exception to this rule, for it is united to the skull
by the mobile temporomandibular joint.
The bones of the skull can be divided into those of the cranium and those of the
face. The vault is the upper part of the cranium, and the base of the skull is the
lowest part of the cranium.
The skull bones are made up of external and internal tables of compact bone
separated by a layer of spongy bone called the diplo. The internal table is
thinner and more brittle than the external table. The bones are covered on the
outer and inner surfaces with periosteum.The cranium consists of the following
bones, two of which are paired:
a. Frontal bone: 1
d. Temporal bones: 2

b. Parietal bones: 2

c. Occipital bone: 1

e. Sphenoid bone: 1

f. Ethmoid bone: 1

The facial bones consist of the following, two of which are single:
a. Zygomatic bones: 2
d. Lacrimal bones: 2
g. Inferior conchae: 2

b. Maxillae: 2
e. Vomer: 1

c. Nasal bones: 2
f. Palatine bones: 2

h. Mandible: 1

(Q.130) Jugular foramen contains.


Glossopharyngeal nerve
(a)
Vagus nerve
(b)
Sigmoid sinus
(c)
All of the above.
(d)
Your Response
:
Correct Answer
D
:
All of the above.
Exp:
Summary of the More Important Openings in the Base of the Skull and
the Structures That Pass Through Them

Opening in Skull Bone of Skull


Anterior Cranial Fossa
Perforations in Ethmoid
cribriform plate
Middle Cranial Fossa
Optic canal
Lesser wing of
sphenoid
Superior orbital Between lesser and
fissure
greater wings of
sphenoid
Foramen
rotundum
Foramen ovale

Structures Transmitted
Olfactory nerves

Optic nerve, ophthalmic artery


Lacrimal, frontal, trochlear,
oculomotor, nasociliary, and
abducent nerves; superior ophthalmic
vein
Maxillary division of the trigeminal
nerve
Mandibular division of the trigeminal
nerve, lesser petrosal nerve
Middle meningeal artery

Greater wing of
sphenoid
Greater wing of
sphenoid
Foramen
Greater wing of
spinosum
sphenoid
Foramen lacerum Between petrous part Internal carotid artery
of temporal and
sphenoid
Posterior Cranial Fossa
Foramen
Occipital
Medulla oblongata, spinal part of
magnum
accessory nerve, and right and left
vertebral arteries
Hypoglossal
Occipital
Hypoglossal nerve
canal
Jugular foramen Between petrous part Glossopharyngeal, vagus, and
of temporal and
accessory nerves; sigmoid sinus
condylar part of
becomes internal jugular vein
occipital
Internal acoustic Petrous part of
Vestibulocochlear and facial nerves
meatus
temporal

(Q.131) False about Pudendal nerve block ?


It abolishes pain from uterine contractions.
(a)
The bony landmark used is the ischial spine in transvaginal approach.
(b)
The bony landmark is the ischial tuberosity in transperineal approach.
(c)
(d)

This nerve block does not, however, abolish sensation from the anterior part of
the perineum.

Your Response
:
Correct Answer
A
:
It abolishes pain from uterine contractions.
Exp:
Ref : Clinical Anatomy by Regions : Richard Snell : 8th Ed.
Pudendal Nerve Block
Area of Anesthesia
The area anesthetized is the skin of the perineum; this nerve block does not,
however, abolish sensation from the anterior part of the perineum, which is
innervated by the ilioinguinal nerve and the genitofemoral nerve. Needless to

say, it does not abolish pain from uterine contractions that ascend to the spinal
cord via the sympathetic afferent nerves.
Indications
During the second stage of a difficult labor, when the presenting part of the
fetus, usually the head, is descending through the vulva, forceps delivery and
episiotomy may be necessary.
Transvaginal Procedure
The bony landmark used is the ischial spine. The index finger is inserted
through the vagina to palpate the ischial spine. The needle of the syringe is then
passed through the vaginal mucous membrane toward the ischial spine. On
passing through the sacrospinous ligament, the anesthetic solution is injected
around the pudendal nerve.
Perineal Procedure
The bony landmark is the ischial tuberosity. The tuberosity is palpated
subcutaneously through the buttock, and the needle is introduced into the
pudendal canal along the medial side of the tuberosity. The canal lies about 1
in. (2.5 cm) deep to the free surface of the ischial tuberosity. The local
anesthetic is then infiltrated around the pudendal nerve.

(Q.132) The large intestine is lined by?


Simple columnar epithelium
(a)
Pseudostratified columnar epithelium
(b)
Ciliated columnar epithelium
(c)
Cuboidal epithelium
(d)
Your Response
:
Correct Answer
A
:
Simple columnar epithelium
Exp:
EPITHELIAL LINING AT VARIOUS SITES IN THE BODY :
SITE
EPITHELIUM TYPE
Pleura,Pericardium, Peritoneum,
Simple squamous epithelium
Endocardium
Throglossal cyst
Pseudostratified columnar and
Squamous epithelium
Alveoli in premature/preterms
Low cuboidal epithelium
Ethmoidal nasal polyp
Early stage : Ciliated columnar
epithelium

Paranasal sinuses
Nasolabial cysts
Bartholins duct
Bartholins gland acini
Endocervix / cervical canal
Ectocervix

Late stage : Transitional and squamous


epithelium(Metaplastic)
Ciliated columnar epithelium
Pseudostratified columnar epithelium
Multilayered columnar epithelium
Low columnar / Cuboidal single layer
Ciliated High columnar epithelium
Squamous epithelium

Endometrium
Columnar epithelium
Adult vagina
Stratified squamous epithelium
Newborn vagina
Transitional epithelium
Endosalpinx
Tall ciliated columnar epithelium
Chocolate cyst of ovary in
Columnar cells with tendency to form
endometriosis
papillae
Vocal cord and upper part of vestibule Stratified squamous epithelium
of larynx
Rest of the larynx
Cilitaed columnar epithelium
Bowmans capsule
Simple squamous epithelium
PCT
Simple cuboidal epithelium with brush
border
DCT
Simple cuboidal epithelium without
brush border
Thin Loop of Henle
Low cuboidal epithelium or squamous
epithelium
Thick LH
Cuboidal epithelium
Collecting duct
Simple cuboidal or columnar epithelium
Proximal urethra
Transitional epithelium
Mid urethra
Pseudostratified columnar epithelium
Most distal part of urethra
Stratified squamous epithelium
Mouth , oral cavity and esophagus
Stratified squamous nonkeratinized
epithelium
Stomach
Simple columnar epithelium
Small intestine
Columnar epithelium with brush border
Microvilli
Large intestine
Simple columnar epithelium
Transition zone of anal canal
Stratified columnar epithelium
Anal canal /anal verge
Stratified squamous epithelium
Thyroid follicles
Cuboidal cells
Ovarian germinal epithelium
Cuboidal cells
Gall bladder
Columnar cells with brush border
Secondary ovarian follicles
Stratified cuboidal epithelium
Ducts of salivary glands
Stratified columnar epithelium
Sensory epithelium of olfactory area Pseudostratified columnar epithelium
Trachea and large bronchi
Pseudostratified columnar epithelium
Part of Auditory tube
Ductus deferens

(Q.133) The following structures pass through both Greater as well as lesser sciatic foramen except ?
Tendon of obturator internus
(a)
Nerve to obturator internus
(b)
Pudendal nerve
(c)
Internal pudendal artery and vein.
(d)
Your Response
:
Correct Answer
A
:
Tendon of obturator internus
Exp:

Foramina of the Gluteal Region


The two important foramina in the gluteal region are the greater sciatic foramen
and the lesser sciatic foramen.
Greater Sciatic Foramen
The greater sciatic foramen (see Fig. 6-11) is formed by the greater sciatic
notch of the hip bone and the sacrotuberous and sacrospinous ligaments. It
provides an exit from the pelvis into the gluteal region.
The following structures exit the foramen (Fig. 10-15):
Piriformis
Sciatic nerve
Posterior cutaneous nerve of the thigh
Superior and inferior gluteal nerves
Nerves to the obturator internus and quadratus femoris
Pudendal nerve
Superior and inferior gluteal arteries and veins
Internal pudendal artery and vein
Lesser Sciatic Foramen
The lesser sciatic foramen (see Fig. 6-11) is formed by the lesser sciatic notch
of the hip bone and the sacrotuberous and sacrospinous ligaments. It provides
an entrance into the perineum from the gluteal region. Its presence enables
nerves and blood vessels that have left the pelvis through the greater sciatic
foramen above the pelvic floor to enter the perineum below the pelvic floor.
The following structures pass through the foramen (Fig. 10-14):
Tendon of obturator internus muscle
Nerve to obturator internus
Pudendal nerve
Internal pudendal artery and vein

(Q.134) The spinal segment present at the level of T12 vertebra is ?


L1-2 cord segment
(a)
L2-3 cord segment
(b)
Sacral cord segments
(c)
L5 cord segment
(d)

Your Response
:
Correct Answer
D
:
L5 cord segment
Exp:
Relationships of Spinal Cord Segments to Vertebral Numbers
Because the spinal cord is shorter than the vertebral column, the spinal cord
segments do not correspond numerically with the vertebrae that lie at the same
level. The following list helps determine which spinal segment is contiguous
with a given vertebral body.
Vertebrae
Cervical
Upper thoracic
Lower thoracic
(T7 to 9)
Tenth thoracic
Eleventh thoracic
Twelfth thoracic
First lumbar

Spinal Segment
Add 1
Add 2
Add 3
L1 and 2 cord segments
L3 and 4 cord segments
L5 cord segment
Sacral and coccygeal cord segments

(Q.135) The following factors predispose the appendix to infection except ?


It is a long, narrow, blind-ended tube.
(a)
It has a large amount of lymphoid tissue in its wall.
(b)
The lumen has a tendency to become obstructed by hardened intestinal contents.
(c)
(d)

Appendix is supplied by a long small artery that does not anastomose with other
arteries.

Your Response
:
Correct Answer
D
:
.Appendix is supplied by a long small artery that does not anastomose with
Exp:
other arteries.
REF : Clinical Anatomy by Regions : Richard Snell : 8th Ed.
Variability of Position of the Appendix
The inconstancy of the position of the appendix should be borne in mind when
attempting to diagnose an appendicitis. A retrocecal appendix, for example,
may lie behind a cecum distended with gas, and thus it may be difficult to elicit
tenderness on palpation in the right iliac region. Irritation of the psoas muscle,
conversely, may cause the patient to keep the right hip joint flexed.
An appendix hanging down in the pelvis may result in absent abdominal
tenderness in the right lower quadrant, but deep tenderness may be experienced
just above the symphysis pubis. Rectal or vaginal examination may reveal
tenderness of the peritoneum in the pelvis on the right side.
Predisposition of the Appendix to Infection

The following factors contribute to the appendix's predilection to infection:


It is a long, narrow, blind-ended tube, which encourages stasis of large-bowel
contents.
It has a large amount of lymphoid tissue in its wall.
The lumen has a tendency to become obstructed by hardened intestinal contents
(enteroliths), which leads to further stagnation of its contents.
Predisposition of the Appendix to Perforation
The appendix is supplied by a long small artery that does not anastomose with
other arteries. The blind end of the appendix is supplied by the terminal
branches of the appendicular artery. Inflammatory edema of the appendicular
wall compresses the blood supply to the appendix and often leads to thrombosis
of the appendicular artery. These conditions commonly result in necrosis or
gangrene of the appendicular wall, with perforation.
Perforation of the appendix or transmigration of bacteria through the inflamed
appendicular wall results in infection of the peritoneum of the greater sac.
Pain of Appendicitis
Visceral pain in the appendix is produced by distention of its lumen or spasm of
its muscle. The afferent pain fibers enter the spinal cord at the level of the 10th
thoracic segment, and a vague referred pain is felt in the region of the
umbilicus. Later, the pain shifts to where the inflamed appendix irritates the
parietal peritoneum. Here the pain is precise, severe, and localized

(Q.136) The hindgut artery is?


Coeliac artery
(a)
Superior mesenteric artery
(b)
Inferior mesenteric artery
(c)
Colic artery
(d)
Your Response
:
Correct Answer
C
:
. Inferior mesenteric artery.
Exp:
th

REF : Clinical Anatomy by Regions :

Richard Snell : 8 Ed

Explanation for the Blood Supply to the Gastrointestinal Tract


Foregut Arteries
The cephalic end of the foregut (which includes the pharynx) and the cervical
and thoracic portions of the esophagus are supplied by the ascending
pharyngeal arteries, palatine arteries, superior and inferior thyroid arteries,
bronchial arteries, and esophageal branches from the aorta. The caudal end of
the foregut (which includes the distal third of the esophagus, the stomach, and
the proximal half of the duodenum) is supplied by a number of vessels that fuse
to form a single trunk, the celiac artery . It is interesting to note that this artery

also supplies the liver and pancreas, which are glandular derivatives of this part
of the gut. The spleen is also supplied by the same artery, which is not
surprising, since this organ develops in the dorsal mesentery of the foregut; the
artery to the spleen runs in the splenic renal ligament.
Midgut Artery
The midgut, which extends from halfway along the second part of the
duodenum to the left colic flexure, is supplied by the superior mesenteric artery,
which represents the fused pair of vitelline arteries
Hindgut Artery
The hindgut, which extends from the left colic flexure to halfway down the anal
canal, is supplied by the inferior mesenteric artery. This represents a number of
ventral branches of the aorta that fuse to form a single artery.

(Q.137) False is ?
(a)
(b)
(c)
(d)

The caliber of the full small intestine is smaller than that of the filled large
intestine.
The wall of the small intestine is smooth, whereas that of the large intestine is
sacculate(d.)
The mucous membrane of the small intestine has villi, which are absent in the
large intestine.
Aggregations of lymphoid tissue called Peyer's patches are found in the
submucosa of the small intestine

Your Response
:
Correct Answer
D
:
Aggregations of lymphoid tissue called Peyer's patches are found in the
Exp:
submucosa of the small intestine
REF : Clinical Anatomy by Regions : Richard Snell : 8th Ed
Peyers patches are found in the mucosa of the terminal part of the small
intestine.
Differences Between the Small and Large Intestine
External Differences
The small intestine (with the exception of the duodenum) is mobile, whereas
the ascending and descending parts of the colon are fixed.
The caliber of the full small intestine is smaller than that of the filled large
intestine.
The small intestine (with the exception of the duodenum) has a mesentery that
passes downward across the midline into the right iliac fossa.
The longitudinal muscle of the small intestine forms a continuous layer around
the gut. In the large intestine (with the exception of the appendix) the
longitudinal muscle is collected into three bands, the teniae coli.

The small intestine has no fatty tags attached to its wall. The large intestine has
fatty tags, called the appendices epiploicae.
The wall of the small intestine is smooth, whereas that of the large intestine is
sacculated.
Internal Differences
The mucous membrane of the small intestine has permanent folds, called
plicaecirculares, which are absent in the large intestine.
The mucous membrane of the small intestine has villi, which are absent in the
large intestine.
Aggregations of lymphoid tissue called Peyer's patches are found in the mucous
membrane of the small intestine; these are absent in the large intestine.

(Q.138)

Parasympathetic secretomotor fibers to parotid come from all except


Otic ganglion
(a)

(b)
(c)
(d)
Your Response
:
Correct Answer
:
Exp:

Greater petrosal nerve


Auriculotemporal nerve
Tympanic plexus

B
Greater petrosal nerve
Parasympathetic secretomotor fibres to the parotid gland are carried by the
lesser petrosal and not the greater petrosal nerve (b).
Greater petrosal nerve carries secretomotor fibres to the pterygopalatine
ganglion and supplies the lacrimal, nasal & palatine glands.
Secretory fibres to the parotid gland start in the inferior salivatory nucleus
(brainstem) glossopharyngeal nerve tympanic branch tympanic
plexus (d) lesser petrosal nerve otic ganglion (a) auriculotemporal
nerve (c) parotid gland.
Tympanic plexus is present in the middle ear and receives the preganglionic
fibres from the glossopharyngeal nerve.
Otic ganglion lies just inferior to the foramen ovale, through which the lesser
petrosal nerve passes and carries the preganglionic fibres to the ganglion.
Auriculotemporal nerve is a branch of mandibular nerve, which carries the
postganglionic fibres from the otic ganglion to the parotid gland.
References: 1. Pg. 400; B D Chaurasia (4th Ed.); Vol-3
Anatomy (39th Ed.)

(Q.139)

Pelvic splanchnic nerve supplies all except


Appendix
(a)

2. Pg. 517; Gray's

Rectum
(b)
Urinary bladder
(c)
Uterus
(d)
Your Response
:
Correct Answer
A
:
Appendix
Exp:
Pelvic splanchnic nerves (PSNs) are the parasympathetic nerves to supply the
pelvic viscera like urinary bladder, uterus and rectum.
Appendix is a part of Mid-gut and is supplied by the Vagus nerve.
Parasympathetic nervous system
Functions:
Evacuation of body tubes by
Increase in peristalsis
Relaxation of sphincters
Erection* of genitalia
Parasympathetic nerve supply:
Till mid-gut: Vagus
Hind-gut: PSNs
Pelvic splanchnic nerves (also called as Nervi erigentes - NEs) are branches of
sacral plexus carrying parasympathetic fibres.
PSNs provide vasodilator fibres erectile tissue of penis & clitoris causing
erection.
Conus medullary syndrome: PSNs/NEs are lesioned leading to urinary
retention, constipation and impotence (erectile dysfunction).
Sympathetic nervous system
Functions:
Retention of the body tubes contents by
Decrease in peristalsis
Constriction of sphincters
Ejaculation*

Sympathetic supply:
Fore-gut: Greater splanchnic nerve (T: 5-9)
Mid-gut: Lesser splanchnic nerves (T-10,11)
Kidney: Least splanchnic nerves (T-12)
Hind-gut: Lumbar splanchnic nerves (L-1,2)
Visceral pain is usually carried by the sympathetic fibers and pain of acute
appendicitis (Mid-gut; T-10) is carried by the lesser splanchnic nerves and
referred to the corresponding dermatome (Peri-umbilical region: T-10).
References:
Pg. 1045, 1143; Gray's Anatomy (40th Ed.)
Pg. 258; B D Chaurasia (4th Ed.); Vol-2

(Q.140) False about Felon is ?


Pulp space infection.
(a)
Each space is subdivided into smaller compartments by fibrous sept(a.)
(b)
Necrosis of diaphysis occurs in untreated infection.
(c)
Epiphysis is commonly necrose(d.)
(d)
Your Response
:
Correct Answer
D
:
Epiphysis is commonly necrosed.
Exp:
Pulp-Space Infection (Felon)
The pulp space of the fingers is a closed fascial compartment situated in front of
the terminal phalanx of each finger (Fig. 9-70). Infection of such a space is
common and serious, occurring most often in the thumb and index finger.
Bacteria are usually introduced into the space by pinpricks or sewing needles.
Because each space is subdivided into numerous smaller compartments by
fibrous septa, it is easily understood that the accumulation of inflammatory
exudate within these compartments causes the pressure in the pulp space to
quickly rise. If the infection is left without decompression, infection of the
terminal phalanx can occur. In children, the blood supply to the diaphysis of the
phalanx passes through the pulp space, and pressure on the blood vessels could
result in necrosis of the diaphysis. The proximally located epiphysis of this
bone is saved because it receives its arterial supply just proximal to the pulp
space.
The close relationship of the proximal end of the pulp space to the digital
synovial sheath accounts for the involvement of the sheath in the infectious
process when the pulp-space infection has been neglected.

(Q.141) False is ?
(a)

The supraorbital notch transmits the superior ophthalmic vein.

(b)
(c)
(d)
Your Response
:
Correct Answer
:
Exp:

Optic canal transmits the optic nerve and the ophthalmic artery.
Infraorbital groove and canal they transmit the infraorbital nerve
Inferior orbital fissure communicates with the pterygopalatine fossa

A
The supraorbital notch transmits the superior ophthalmic vein.
Openings Into the Orbital Cavity
Orbital opening: Lies anteriorly About one-sixth of the eye is exposed; the
remainder is protected by the walls of the orbit.
Supraorbital notch (Foramen): The supraorbital notch is situated on the superior
orbital margin. It transmits the supraorbital nerve and blood vessels.
Infraorbital groove and canal: Situated on the floor of the orbit in the orbital
plate of the maxilla they transmit the infraorbital nerve (a continuation of the
maxillary nerve) and blood vessels.
Nasolacrimal canal: Located anteriorly on the medial wall; it communicates
with the inferior meatus of the nose. It transmits the nasolacrimal duct.
Inferior orbital fissure: Located posteriorly between the maxilla and the greater
wing of the sphenoid; it communicates with the pterygopalatine fossa. It
transmits the maxillary nerve and its zygomatic branch, the inferior ophthalmic
vein, and sympathetic nerves.
Superior orbital fissure: Located posteriorly between the greater and lesser
wings of the sphenoid it communicates with the middle cranial fossa. It
transmits the lacrimal nerve, the frontal nerve, the trochlear nerve, the
oculomotor nerve (upper and lower divisions), the abducent nerve, the
nasociliary nerve, and the superior ophthalmic vein.
Optic canal: Located posteriorly in the lesser wing of the sphenoid it
communicates with the middle cranial fossa. It transmits the optic nerve and the
ophthalmic artery.

(Q.142) All are purely motor cranial nerves except ?


Oculomotor
(a)
Trochlear
(b)
Trigeminal
(c)
Abducent
(d)
Your Response
:
Correct Answer
C
:
Trigeminal
Exp:
Cranial Nerves
Nerve

Components

Function

Opening in
Skull

I.

Olfactory

Sensory

II.
III.

Optic
Oculomotor

Sensory
Motor

IV.

Trochlear

Motor

V.

Trigeminal
Ophthalmic
division

Sensory

Maxillary
division

Sensory

Mandibular
division

Motor

Sensory

VI.

Abducent

Motor

VII.

Facial

Motor

Sensory

Smell

Openings in
cribriform
plate of
ethmoid
Vision
Optic canal
Lifts upper eyelid,
Superior
turns eyeball upward, orbital fissure
downward, and
medially; constricts
pupil; accommodates
eye
Assists in turning
Superior
eyeball downward
orbital fissure
and laterally
Cornea, skin of
Superior
forehead, scalp,
orbital fissure
eyelids, and nose;
also mucous
membrane of
paranasal sinuses and
nasal cavity
Skin of face over
Foramen
maxilla and the upper rotundum
lip; teeth of upper
jaw; mucous
membrane of nose,
the maxillary air
sinus, and palate
Muscles of
Foramen
mastication,
ovale
mylohyoid, anterior
belly of digastric,
tensor veli palatini,
and tensor tympani
Skin of cheek, skin
over mandible, lower
lip, and side of head;
teeth of lower jaw
and
temporomandibular
joint; mucous
membrane of mouth
and anterior two
thirds of tongue
Lateral rectus
Superior
muscle: turns eyeball orbital fissure
laterally
Muscles of face,
Internal
cheek, and scalp;
acoustic
stapedius muscle of meatus, facial
middle ear;
canal,
stylohyoid; and
stylomastoid
posterior belly of
foramen
digastric
Taste from anterior
two thirds of tongue,

Secretomotor

VIII. Vestibulocochlear
Vestibular
Sensory

IX.

Cochlear
Sensory
Glossopharyngeal Motor

X.

Vagus

XI.

Accessory
Cranial root

Motor

Spinal root

Motor

Hypoglossal

Motor

XII.

floor of mouth, and


palate
Submandibular and
sublingual salivary
parasympathetic
glands, lacrimal
gland, and glands of
nose and palate
Position and
movement of head

Internal
acoustic
meatus

Hearing
Stylopharyngeus
muscle: assists
swallowing
Secretomotor Parotid salivary gland Jugular
parasympathetic
foramen
Sensory
General sensation
and taste from
posterior third of
tongue and pharynx;
carotid sinus and
carotid body
Motor
Constrictor muscles Jugular
of pharynx and
foramen
intrinsic muscles of
larynx; involuntary
muscle of trachea and
bronchi, heart,
alimentary tract from
pharynx to splenic
flexure of colon; liver
and pancreas
Sensory
Taste from epiglottis
and vallecula and
afferent fibers from
structures named
above

(Q.143) The anterior ethmoidal air cells drain into the ?


Superior meatus
(a)
Infundibulum
(b)
Spheno ethmoidal recess
(c)
Inferior meatus
(d)

Muscles of soft
Jugular
palate, pharynx, and foramen
larynx
Sternocleidomastoid
and trapezius muscles
Muscles of tongue
Hypoglossal
controlling its shape canal
and movement
(except
palatoglossus)

Your Response
:
Correct Answer
B
:
Infundibulum
Exp:
Paranasal Sinuses and Their Site of Drainage Into the Nose
Sinus
Site of Drainage
Maxillary sinus
Middle meatus through hiatus semilunaris
Frontal sinuses
Middle meatus via infundibulum
Sphenoidal sinuses
Sphenoethmoidal recess
Ethmoidal sinuses
a
Note that maxillary and sphenoidal sinuses are present in rudimentary form at
birth, enlarge appreciably after the eighth year, and are fully formed in
adolescence.

(Q.144) In the lower limb, a bunion is ?


An adventitial bursa located over the medial side of the head of the first
(a)
metatarsal bone.
(b)
(c)
(d)
Your Response
:
Correct Answer
:
Exp:

Articular bursa at anterior knee.


Suppurative infection of the metatarsal burs(a.)
Hard fibrous overgrowth of the sole.

A
An adventitial bursa located over the medial side of the head of the first
metatarsal bone.
Bursae and Bursitis in the Lower Limb
A variety of bursae are found in the lower limb where skin, tendons, ligaments,
or muscles repeatedly rub against bony points or ridges.
Bursitis, or inflammation of a bursa, can be caused by acute or chronic trauma,
crystal disease, infection, or disease of a neighboring joint that communicates
with the bursa. An inflamed bursa becomes distended with excessive amounts
of fluid. The following bursae are prone to inflammation: the bursa over the
ischial tuberosity; the greater trochanter bursa; the prepatellar and superficial
infrapatellar bursae; the bursa between the tendons of insertion of the sartorius,
gracilis, and semitendinosus muscles on the medial proximal aspect of the tibia;
and the bursa between the tendo calcaneus and the upper part of the calcaneum
(long-distance runner's ankle).
Two important bursae communicate with the knee joint, and they can become
distended if excessive amounts of synovial fluid accumulate within the joint.
The suprapatellar bursa extends proximally about three fingerbreadths above
the patella beneath the quadriceps femoris muscle. The bursa, which is
associated with the insertion of the semimembranosus muscle, may enlarge in
patients with osteoarthritis of the knee joint.
The anatomic bursae described should not be confused with adventitious
bursae, which develop in response to abnormal and excessive friction. For
example, a subcutaneous bursa sometimes develops over the tendo calcaneus in

response to badly fitting shoes. A bunion is an adventitial bursa located over the
medial side of the head of the first metatarsal bone.

(Q.145) A Spigelian hernia occurs at?


Deep inguinal ring
(a)
Linea semilunaris
(b)
Umbilical defect
(c)
Femoral ring
(d)
Your Response
:
Correct Answer
B
:
. Linea semilunaris
Exp:
Separation of the Recti Abdominis
Separation of the recti abdominis occurs in elderly multiparous women with
weak abdominal muscles. In this condition, the aponeuroses forming the rectus
sheath become excessively stretched. When the patient coughs or strains, the
recti separate widely, and a large hernial sac, containing abdominal viscera,
bulges forward between the medial margins of the recti. This can be corrected
by wearing a suitable abdominal belt.
Incisional Hernia
A postoperative incisional hernia is most likely to occur in patients in whom it
was necessary to cut one of the segmental nerves supplying the muscles of the
anterior abdominal wall; postoperative wound infection with death (necrosis) of
the abdominal musculature is also a common cause. The neck of the sac is
usually large, and adhesion and strangulation of its contents are rare
complications. In very obese individuals the extent of the abdominal wall
weakness is often difficult to assess.
Hernia of the Linea Semilunaris (Spigelian Hernia)
The uncommon hernia of the linea semilunaris occurs through the aponeurosis
of the transversus abdominis just lateral to the lateral edge of the rectus sheath.
It usually occurs just below the level of the umbilicus. The neck of the sac is
narrow, so that adhesion and strangulation of its contents are common
complications.
Lumbar Hernia
The lumbar hernia occurs through the lumbar triangle and is rare. The lumbar
triangle (Petit's triangle) is a weak area in the posterior part of the abdominal
wall. It is bounded anteriorly by the posterior margin of the external oblique
muscle, posteriorly by the anterior border of the latissimus dorsi muscle, and
inferiorly by the iliac crest. The floor of the triangle is formed by the internal
oblique and the transversus abdominis muscles. The neck of the hernia is
usually large, and the incidence of strangulation low.
Internal Hernia
Occasionally, a loop of intestine enters a peritoneal recess (e.g., the lesser sac or
the duodenal recesses) and becomes strangulated at the edges of the recess.

(Q.146) Which of the following is concerned with the genotype of offspring?


Inverse square
(a)
Punnet square
(b)
Mandelian square
(c)
Hardy Winbergs square
(d)
Your Response
:
Correct Answer
B
:
Punnet square. (Ref. Emerys elements of medical genetics 11th/ pg. 4;
Exp:
97)
An alternative method to Medelian representation, another method of
determining the genotype in the offspring involves construction of so called
Punnet square.
A Punnettsquare is a chart which shows/predicts all possible gene
combinations in a cross of parents (whose genes are known). Punnett squares
are named for an English geneticist, Reginald Punnett. He discovered some
basic principles of genetics, including sex linkage and sex determination.

(Q.147) Immediately after birth, three major changes occur in the newborn except?
Fluid in the alveoli is absorbed into lung tissue.
(a)
Increase in the systemic blood pressure.
(b)
Increase in the pulmonary blood flow.
(c)
(d)

Majority of the blood is shunted to the aorta through the foramen ovale and
patent ductus arteriosus.

Your Response
:
Correct Answer
D
:
Majority of the blood is shunted to the aorta through the foramen ovale
Exp:
and patent ductus arteriosus.
The bronchial arterial blood is oxygenated blood, in contrast to the partially
deoxygenated blood in the pulmonary arteries.
In utero, the lungs do not perform gas exchange and accordingly, pulmonary
blood vessels are markedly constricted. Oxygenated blood flows from the
placenta through the umbilical vein to the right heart where the majority of
the blood is shunted to the aorta through the foramen ovale and patent ductus
arteriosus.
Having circulated through the capillaries of the placental villi, the fetal blood
returns through the umbilical vein to the fetus about 80% saturated with
oxygen and containing many important nutrients, antibodies, and hormones.
The oxygenated blood then passes toward the liver. However, the greater
volume of it bypasses the liver, since this organ is not fully functioning, and
travels to the inferior vena cava by way of the ductus venosus. At the same
time, the ductus venosus receives poorly oxygenated blood from the gut by way
of the left branch of the portal vein. In addition, the inferior vena cava already
contains venous blood from the lower part of the trunk and the lower limbs of

the fetus.
As a consequence of this admixture of blood from these various sources, the
inferior vena cava contains blood about 67% saturated with oxygen.
Seconds after birth, three major changes occur in the newborn in order to
transition to extrauterine life.
First, fluid in the alveoli is absorbed into lung tissue and is replaced by air.
Second, the umbilical cord is clamped, disconnecting the infant from the low
resistance placental circulation and increasing systemic blood pressure.
Lastly, the pulmonary vasculature relaxes in response to increased oxygen
levels in the lungs causing a dramatic increase in pulmonary blood flow.
The right to left shunt through the patent ductus arteriosus decreases, becoming
bi-directional and the foramen ovale functionally closes in association with the
increase in blood return to the left atrium

(Q.148) Which of the following is not correct regarding embryological development of the muscles?
Skeletal muscle are derived from paraxial mesoderm
(a)
Smooth muscle differentiates from splanchnic mesoderm.
(b)
(c)

Pupillary, mammary gland and sweat gland muscles are derived from
intermediate mesoderm
Cardiac muscles are derived from splanchnic mesoderm.

(d)
Your Response
:
Correct Answer
C
:
Pupillary, mammary gland and sweat gland muscles are derived from
Exp:
intermediate mesoderm.
(Ref. Human Embryology by IB Singh 6th/pg. 46; Langmans embryology
9th/pg. 199)
With the exception of some smooth muscle tissue, the muscular system
develops from the mesodermal germ layer and consists of skeletal, smooth, and
cardiac muscle.
Derived from paraxial mesoderm which forms
1 Skeletal muscle
somites from the occipital to the sacral regions
and somitomeres in the head.
Differentiates from splanchnic mesoderm
2 Smooth muscle
(which is a part of lateral plate mesoderm)
surrounding the gut and its derivatives.
3 Pupillary, mammary
From ectoderm
gland and sweat gland
muscles
4 Cardiac muscle
Derived from splanchnic mesoderm surrounding
the heart tube.
mesoderm
derivatives
1 Splanchnopleuric
Mesothelial covering of visceral organs
mesoderm

(from lateral plate


mesoderm)
2 Intermediate plate
mesoderm
3 Paraxial mesoderm

Urinary and genital systems


Axial skeleton, voluntary musculature, etc.

(Q.149) The central tendon of the diaphragm develops from:


Occipital myotome
(a)
Dorsal mesogastrium
(b)
Pleuro-peritoneal membrane
(c)
Septum transversum
(d)
Your Response
:
Correct Answer
D
:
Septum transversum. (Ref. Langmans embryology 9th/pg. 218)
Exp:
The septum transversum is a thick plate of mesodermal tissue occupying the
space between the thoracic cavity and the stalk of the yolk sac. This septum
does not separate the thoracic and abdominal cavities completely but leaves
large openings, the pericardioperitoneal canals, on each side of the foregut.
The diaphragm is derived from the following structures:
Septum transversum (which forms the central tendon of the diaphragm);
Two pleuroperitoneal membranes;
Muscular components from the lateral and dorsal body walls; and
The mesentery of the esophagus, in which the crura of the diaphragm develop.

(Q.150) Which of the following is a branch from the root of the brachial plexus?
Nerve to subclavius
(a)
Suprascapular nerve
(b)
Dorsal scapular nerve
(c)
Anterior thoracic nerve
(d)
Your Response
:
Correct Answer
C
:
Dorsal scapular nerve
REF : CLINICAL ANATOMY BY REGIONS
Exp:
th

SNELL 8 Ed.
Summary of the Branches of the Brachial Plexus and Their Distribution
Branches
Roots
Dorsal scapular nerve (C5)

Distribution

Rhomboid minor, rhomboid major, levator


scapulae muscles
Long thoracic nerve (C5, 6, 7) Serratus anterior muscle
Upper Trunk

Suprascapular nerve (C5, 6)


Nerve to subclavius (C5, 6)
Lateral Cord
Lateral pectoral nerve (C5, 6,
7)
Musculocutaneous nerve (C5,
6, 7)

Supraspinatus and infraspinatus muscles


Subclavius
Pectoralis major muscle

Coracobrachialis, biceps brachii, brachialis


muscles; supplies skin along lateral border of
forearm when it becomes the lateral cutaneous
nerve of forearm
Lateral root of median nerve See medial root of median nerve
(C5, 6, 7)
Posterior Cord
Upper subscapular nerve (C5, Subscapularis muscle
6)
Thoracodorsal nerve (C6, 7, Latissimus dorsi muscle
8)
Lower subscapular nerve (C5, Subscapularis and teres major muscles
6)
Axillary nerve (C5, 6)
Deltoid and teres minor muscles; upper lateral
cutaneous nerve of arm supplies skin over
lower half of deltoid muscle
Radial nerve (C5, 6, 7, 8; T1) Triceps, anconeus, part of brachialis, extensor
carpi radialis longus; via deepradial nerve
branch supplies extensor muscles of forearm:
supinator, extensorcarpi radialis brevis,
extensor carpi ulnaris, extensor digitorum,
extensor digitiminimi, extensor indicis,
abductor pollicis longus, extensor pollicis
longus, extensor pollicis brevis; skin, lower
lateral cutaneous nerve of arm,
posteriorcutaneous nerve of arm, and posterior
cutaneous nerve of forearm; skin onlateral side
of dorsum of hand and dorsal surface of lateral
three and a halffingers; articular branches to
elbow, wrist, and hand
Medial Cord
Medial pectoral nerve (C8;
Pectoralis major and minor muscles
T1)
Medial cutaneous nerve of
Skin of medial side of arm
arm joined by intercostal
brachial nerve from second
intercostal nerve (C8; T1, 2)
Medial cutaneous nerve of
Skin of medial side of forearm
forearm (C8; T1)
Ulnar nerve (C8; T1)
Flexor carpi ulnaris and medial half of flexor
digitorum profundus, flexor digitiminimi,
opponens digiti minimi, abductor digiti minimi,
adductor pollicis, third and fourth lumbricals,
interossei, palmaris brevis, skin of medial half
ofdorsum of hand and palm, skin of palmar and
dorsal surfaces of medial oneand a half fingers
Medial root of median nerve Pronator teres, flexor carpi radialis, palmaris
(with lateral root) forms
longus, flexor digitorum superficialis, abductor
median nerve (C5, 6, 7, 8; T1) pollicis brevis, flexor pollicis brevis, opponens
pollicis, first twolumbricals (by way of anterior
interosseous branch), flexor pollicis longus,
flexor digitorum profundus (lateral half),

pronator quadratus; palmar cutaneousbranch to


lateral half of palm and digital branches to
palmar surface of lateralthree and a half
fingers; articular branches to elbow, wrist, and
carpal joints

(Q.151) Anaesthesia on the sole of the foot is caused by injury to:


Common peroneal nerve
(a)
Tibial nerve
(b)
Superficial peroneal nerve
(c)
Deep peroneal nerve
(d)
Your Response
:
Correct Answer
B
:
Tibial nerve
(Ref. BDC Anatomy 4th/ vol. 2 pg.86 & 174 & Fig. 8.3)
Exp:
NERVE palsy
Features
Superior Gluteal Causes loss of abduction of the limb;
Nerve
Impairment of gait;

Inferior Gluteal
Nerve

Femoral Nerve

Patient cannot keep pelvis level when standing on one leg.


This is called "Trendelenburg sign or gait Q
Produces a weakned hip extension;
Patient has difficulty rising from a sitting position or
climbing stairs.
Induces weakened hip flexion;
Loss of extension of the knee.

Sensory loss occurs on the anterior thigh, medial leg, and


foot.
Obturator Nerve Causes a loss of adduction of the thigh as well as sensory
loss on medial thigh.
Brings about a weakened extension of the thigh;
Sciatic Nerve
Loss of flexion of the knee; and loss of function below the
knee.

Tibial nerve

Sensory loss on the posterior thigh, leg (except medial


side), and foot is also observed.
Causes a loss of flexion of the knee and digits;
Loss of plantar flexion;
Weakened inversion

Common
peroneal nerve

Sensory loss on the leg (except medial) and plantar [sole]


foot.Q
associated with fracture neck of fibula (not generally with
#head of fibula).Q

Produces a combination of deficits of lesion of the deep and


superficial peroneal nerves.
Loss of extension of great toe.Q

Deep peroneal
nerve

Sign is "foot drop."Q


Weakened inversion;
Loss of extension of the digits;
Loss of dorsiflexion "foot''.

Superficial
peroneal nerve

Sensory loss at the first web space.


Loss of eversion of the foot.
Sensory loss on dorsum of foot except the first web space.Q

(Q.152) The arteries supplying the femoral head include the following except:
Medial circumflex artery
(a)
Lateral circumflex artery
(b)
Profunda femoral artery
(c)
Ligamentum teres artery
(d)
Your Response
:
Correct Answer
C
:
Profunda femoral artery
Exp:
The vascular supply of the femoral head is also of paramount importance.
(1). The retinacular vessels arising from the lateral femoral circumflex artery
and the medial circumflex artery and then running beneath the synovium
along the neck, which they penetrate proximally both anteriorly and posteriorly;
(2). The interosseous circulation crossing the marrow spaces from distal to
proximal;
(3). Unreliably, the ligamentum teres artery.
Fractures of the femoral neck always disrupt the interosseous circulation; the
femoral head then relies only on the retinacular arteries, which may also be
disrupted or thrombosed. Secondary avascular necrosis of part or all of the
femoral head can result. Union of a fracture can occur in the presence of an
avascular fragment, but the incidence of nonunion is higher.
Intertrochanteric fractures usually do not suffer this same fate. The capsule (and
vessels) are still attached to the proximal fragment after fracture, and thus the
blood supply remains patent.
The hip capsule is a strong thick fibrous structure that attaches on the
intertrochanteric line anteriorly and somewhat more proximally posteriorly. The
intracapsular portion of the neck is not covered with periosteum, and fractures

of the intracapsular part of the neck cannot heal with periosteal callus
formation, only with endosteal union. Interposition of synovial fluid between
fracture fragments, as in any joint, can delay or altogether prevent bony union.

(Q.153) Presacral fascia is derivative of?


Colles fascia
(a)
Scarpas fascia
(b)
Pelvic fascia
(c)
Hypogastric sheath
(d)
Your Response
:
Correct Answer
D
:
Hypogastric sheath
Exp:
th

(Ref. Greys Anatomy 12th/pg. 251; BDC anatomy

4 vol. 2/ pg. )

HYPOGASTRIC SHEATH
The hypogastric sheath is a thick band of condensed pelvic fascia with
following features:
A fascial condensation which does not merely separate two potential spaces
Gives passage to the vessels and nerves passing from the lateral wall of the
pelvis to the pelvic viscera, along with the ureters and, in the mate, the ductus
deferens.
As it extends medially from the lateral pelvic wall, the hypogastric sheath
divides into three laminae (leaflets or wings) which pass to or between the
pelvic organs, conveying neurovascular structures and providing support.
(Because of the latter function, they are also referred to as ligaments):
anteriormost lamina lateral ligament of the bladder.
passes to the bladder, conveying superior vesical arteries and veins/
posteriormost lamina = presacral fascia.
passes to the rectum, conveying the middle rectal artery and vein.
middle lamina
in the male, forms a relatively thin fascial partition, the rectovesical septum,
between the back of the bladder and prostate anteriorly and the rectum
posteriorly.
In the female, the middle lamina is most substantial of the three, passing
medially to the uterine cervix and vagina as the transverse (cardinal) cervical
ligament (lateral cervical or Mackenrodts ligament).

(Q.154) Which of the following is in anterior relation to the head of pancreas?


Common bile duct
(a)
Gastroduodenal artery
(b)

(c)
(d)
Your Response
:
Correct Answer
:
Exp:

Inferior vena cava


Aorta

B
Gastroduodenal artery.
Relations of pancreas:
Head

(Ref. BDC 4th/Vol2 pg. 284)

Neck

Anteriorly -Gastroduodenal -Peritoneum of


artery
postreior wall of
lesser sac.
-Transverse
colon
-Pylorus.
-Jejunum

Posteriorly -Right crus of


diaphragm
-Renal veins
-Bile duct
-IVC

Uncinate Body
process
-Superior -Covered by
mesenteric peritoneum
vessels
-Lesser sac
and stomach

-At its junction with


head lies
gastroduodenal and
superior
pancreaticoduodenal
arteries.
-Termination of
-Aorta
superior mesenteric
vein and beginning of
portal vein.

-Devoid of
peritoneum.
-Aorta and
origin of
SMA.
- Left crus of
diaphragm.
- Left adrenal,
left kidney,
left renal
vessels,.
- Splenic vein.
Inferiorly:
- Covered by
peritoneum.
- DJ flexure.
- Jeunum.
- Left colic
flexure.

(Q.155) Which of the following muscle is supplied by a direct branch from the trunk of mandibular
division of trigeminal nerve?

(a)
(b)
(c)
(d)
Your Response
:
Correct Answer
:
Exp:

Temporalis
Medial pterygoid
Lateral pterygoid
Massater

B
Medial pterygoid.

(Ref. BDC Vol. 3. 4th/ pg. 145)

Mandibular nerve:
Largest branch of trigeminal nerve.
Sensory as well as motor distribution.
Nerve of first pharyngeal arch.
Branches:
From main trunk:
Meningeal branch
Nerve to medial pterygoid
From anterior division:
Buccal nerve (a sensory branch)
Nerve to lateral pterygoid
Nerve to temporalis
From posterior division:
Auriculotemporal nerve
Lingual nerve
Inferior alveolar nerve

(Q.156) The branches of nasociliary nerve include following except:


Anterior ethmoidal nerve
(a)
Posterior ethmoidal nerve
(b)
Supratrochlear nerve
(c)
Infratrochlear nerve
(d)
Your Response
:
Correct Answer
C
:

Exp:

Supratrochlear nerve .
(Ref. BDC Vol. 3. 4th/ pg. 118)
SENSORY INNERVATION
Nasociliary The nasociliary nerve passes above and medial to the eye before
giving off following branches to the nose and the eye:
Nerve
(1) Branches to ciliary ganglion
(2) Anterior ethmoidal
(3) Posterior ethmoidal
(4) Infratrochlear
(5) 2-3 long ciliary nerves
The nasal component is made of the ethmoidal and nasal nerves
that carry sensation from the roof of the nasal cavity, the skin of
the nose, and the sphenoid and ethmoid sinuses.

Lacrimal
Nerve
Frontal
Nerve

The ciliary component is made of the long and short ciliary nerves
that carry sensation from the eye and cornea.
The lacrimal nerve passes above and lateral to the eye.
It carries sensation from the lateral part of the upper eyelid.
The frontal nerve passes over the eye and divides into
(1) Supratrochlear and
(2) Supraorbital nerves.
The supratrochlear nerve exits the orbit above the trochlea and
carries sensation from the skin of the forehead.
The supraorbital nerve exits the orbit through the supraorbital
notch (foramen) and carries sensation from the skin of the
forehead that lies lateral to the area served by the supratrochlear
nerve.
The supraorbital nerve also carries sensation from the frontal
sinuses.

(Q.157)

Peroneus brevis is inserted into


Base of I metatarsal
(a)

(b)
(c)
(d)
Your Response
:
Correct Answer
:
Exp:

Base of V metatarsal
Base of proximal phalanx of big toe
Lateral cuneiform bone

B
Base of V metatarsal
Peroneus brevis is at the lateral side, whereas, peroneus tertius (anterior leg

muscle) inserts on the dorsum of base of 5th metatarsal


References:
1. Pg. 106; B D Chaurasia (4th Ed.); Vol-2.

(Q.158) Posterior cerebral artery (PCA) supplies the following structures except:
Ipsilateral subthalamus and medial thalamus
(a)
Ipsilateral cerebral peduncle
(b)
Medulla oblongata
(c)
Striate cortex
(d)
Your Response
:
Correct Answer
C
:
. Medulla oblongata.
(Ref. H-17th/pg. 2525)
Exp:
PCA (especially P1) supplies ipsilateral subthalamus and medial thalamus and
in the ipsilateral cerebral peduncle and midbrain; and it also supplies (especilay
P2) the medial temporal and occipital lobes, in which is striate cortex.
POSTERIOR CEREBRAL ARTERY
In 75% of cases, both PCAs arise from the bifurcation of the basilar artery; in
20%, one has its origin from the ipsilateral internal carotid artery via the
posterior communicating artery; in 5%, both originate from the respective
ipsilateral internal carotid arteries, when the precommunal, or P1, segment of
the true posterior cerebral artery is atretic.
Clinical
Anatomical Clinical syndromes
Summary
syndromes areas
encountered
due to
involved
occlusion of
PCA
Infarction
Third nerve palsy with
Midbrain, sub(1) P1 /
usually
contralateral ataxia (Claudes thalamic, and
proximal
occurs in the syndrome)
thalamic signs,
PCA
which are due to
syndrome ipsilateral
subthalamus Third nerve palsy with
disease of the
and medial contralateral hemiplegia
proximal P1 segthalamus and (Webers syndrome).
ment of the PCA or
in the
its penetrating
ipsilateral
branches
Thalamic Djerine-Roussy
cerebral
(thalamogeniculate,
syndrome: consists of
peduncle and
Perche- ron, and
contralateral hemisensory loss
midbrain.
followed later by an agonizing, posterior choroidal
searing or burning pain in the arteries);
affected areas. It is persistent
and responds poorly to
analgesics. Anticonvulsants
(carbamazepine or gabapentin)
or tricyclic antidepressants
may be beneficial.
The ataxia indicates
involvement of the red nucleus

or dentatorubrothalamic tract;
the hemiplegia is localized to
the cerebral peduncle.
If the subthalamic nucleus is
involved, contralateral
hemiballismus may occur.
Occlusion of the artery of
Percheron produces paresis of
upward gaze and drowsiness,
and often abulia.
Extensive infarction in the
midbrain and subthalamus
occurring with bilateral
proximal PCA occlusion
presents as coma, unreactive
pupils, bilateral pyramidal
signs, and decerebrate rigidity.
Occlusion of Contralateral homonymous
Cortical temporal
(2) P2 or
hemianopia with macula
and occipital lobe
Distal PCA the distal
signs, due to
syndrome PCA causes sparing is the usual
infarction of manifestation.
occlusion of the P2
the medial
segment distal to
temporal and Occasionally, only the upper the junction of the
occipital
PCA with the
quadrant of visual field is
lobes.
posterior
involved. If the visual
communicating
association areas are spared
and only the calcarine cortex is artery.
involved, the patient may be
aware of visual defects.
Medial temporal lobe and
hippocampal involvement may
cause an acute disturbance in
memory, particularly if it
occurs in the dominant
hemisphere. The defect usually
clears because memory has
bilateral representation.
If the dominant hemisphere is
affected and the infarct
extends to involve the
splenium of the corpus
callosum, the patient may
demonstrate alexia without
agraphia. Visual agnosia for
faces, objects, mathematical
symbols, and colors and
anomia with paraphasic errors
(amnestic aphasia) may also
occur in this setting, even
without callosal involvement.
Occlusion of the posterior

cerebral artery can produce


peduncularhallucinosis
(visual hallucinations of
brightly colored scenes and
objects).
Bilateral infarction in the distal
PCAs produces cortical
blindness (blindness with
preserved pupillary light
reaction). The patient is often
unaware of the blindness or
may even deny it (Antons
syndrome).
Tiny islands of vision may
persist, and the patient may
report that vision fluctuates as
images are captured in the
preserved portions. Rarely,
only peripheral vision is lost
and central vision is spared,
resulting in gun- barrel
vision.
Bilateral visual association
area lesions may result in
Balints syndrome, a disorder
of the orderly visual scanning
of the environment, usually
resulting from infarctions
secondary to low flow in the
watershed between the distal
PCA and MCA territories, as
occurs after cardiac arrest.
Patients may experience
persistence of a visual image
for several minutes despite
gazing at another scene
(palinopia) or an inability to
synthesize the whole of an
image (asimultanagnosia).
The hallmark of Embolic
occlusion of the top of the
basilar artery is the sudden
onset of bilateral signs,
including ptosis, pupillary
asymmetry or lack of reaction
to light, and somnolence.
rd

(Q.159) The following lie in the floor of 3 ventricle except:


Tuber cinereum
(a)
Tegmentum of Mid Brian
(b)
Posterior perforated Substance
(c)
Ependymal undersurface of tela choroidea
(d)
Your Response

:
Correct Answer
D
:
Ependymal undersurface of tela choroidea.
Exp:

(Ref. BDC Vol. 3. 4th/ pg.

367)
Boundaries of 3rd ventricle:
Anteriorly

Posteriorly

Roof

Lamina terminalis. Pineal body.


choroidea. Infundibulum.
Anterior commissure.
bodies.
Anterior column of falx.
MB.

Floor
Ependymal undersurface of tela

Posterior commissure.

Cerebral aqueduct.

Mammillary

Tegmentum of

Post perforated Sub


Tubercinereum.

(Q.160) Which of the following is not true about anatomy related to urethra?
Prostatic utricle is homologus to uterus.
(a)
Membranous urethra is narrowest and least dilatable part of male urethra
(b)
(c)
(d)

The lymphatics from the spongy part of urethra drain to the superficial inguinal
and the external iliac nodes.
Posterior wall of prostatic urethra is characterized by Urethras crest, Colliculus
seminalis and Prostatic sinuses.

Your Response
:
Correct Answer
C
:
The lymphatics from the spongy part of urethra drain to the superficial
Exp:
inguinal and the external iliac nodes.
(Ref. BDC anatomy 4th ed. vol.2- pg.349; Oxford Textbook of Surgery /2nd
pg. 1430)
The lymphatics from the spongy part of urethra pass mostly to deep inguinal
lymph nodes, but some may end in the superficial inguinal and the external iliac
nodes.
The male urethra
In males the urethra is longer, but in the female urethra is about 3 cm long and
lies in the anterior vaginal wall.
It is surrounded at its midpoint by the somatic external sphincter (supplied by
the pudendal nerve S2, 3, 4), which, in combination with the internal sphincter at
the level of the bladder neck, maintains continence.

The male urethra is divided into four segments.


The prostatic urethra, extending from the bladder neck to the apex of the
prostate.
It is the widest part of the urethra
It is notable for a small protuberance on its posterior surface near its distal end,
known as the verumontanum. Q
More than 95 percent are type 1 Posterior Urethral valves, running from the
verumontanum to the distal urethra. Q
Verumontanum serves as an important endoscopic landmark indicating the
most proximal aspect of the somatic external sphincter.
The ejaculatory ducts and the prostatic glands drain into the floor of the urethra
at the level of the verumontanum.
It is semilunar in shape on cross-section. Q
The membranous urethra, running between the apex of the prostate and the
external perineal membran
It is 2 cm long and is surrounded by the somatic external sphincter (innervated
by the pudendal nerve, S234).
The prostatomembranous junction is the most common site of urethral
disruption associated with pelvic trauma. Q
The bulbar urethra extends from the perineal membrane to the beginning of
the pendulous urethra.
It is surrounded by the bulbospongiosus muscle.
It is the area most often injured when a person falls astride an object.Q
The penile/ anterior/ pendulous urethra is contained within the penis.
Just proximal to the meatus (the narrowest part of the urethra) there is a slight
dilatation: this area is known as the navicular fossa.

(Q.161) In the neck, Vagus nerve lies ___________ to common carotid artery ?
Anterolateral
(a)
Posterolateral
(b)
Anteromedial
(c)
Posteromedial
(d)
Your Response
:
Correct Answer
B
:
Posterolateral.
(Ref. BDC Vol.3_5th/pg. 181)
Exp:

Relation of CCA in neck:


Medial to CCA = thyroid
Lateral to CCA = IJV
Anterior to CCA = Sup belly of omohoid crosses CCA at the level of cricoid.
Posterior to CCA = Transverse process of C4-C4, and muscles attached to their
anterior tubercles.
Posterolateral to CCA = runs vagus nerve.

(Q.162) Largest branch of vertebral artery?


Posterior inferior cerebellar artery
(a)
Posterior spinal artery
(b)
Anterior spinal artery
(c)
Medullary arteries
(d)
Your Response
:
Correct Answer
A
:
Posterior inferior cerebellar artery.
Exp:

(Ref. BDC Vol.3_5th/pg. 201)

Largest branch of vertebral artery is Posterior inferior cerebellar artery. It


arises at the lower end of olive. It supplies wedge shaped are on the
posterolateral aspect of medulla, the lower part of Pons and the cerebellum.
Branches of vertebral artery:
Cervical branches:
Cranial branches:
spinal D. PICA

A. Spinal branches

B. Muscular branches

A. Meningeal
B. Posterior spinal
E. Medullary arteries

C. Anterior

(Q.163) Not true about Omphalocele is ?


Umbilical cord is at apex of membrane
(a)
Bowel is normal
(b)
Associated anomalies are very common
(c)
Defect is open
(d)
Your Response
:
Correct Answer
D
:
Defect is open
Exp:
GASTROSCHISIS
Incidence 1 in 6000, Incidence is
increasing.
Defect is open
Defect size 2 to 5 cm
Umbilical cord is to left of defect at

OMPHALOCELE
1 in 4000, Incidence is static
Defect is covered by membrane
Defect size 2 to 15cm
Umbilical cord is at the apex of

its normal attachment site.


Bowel inflamed
Alimentation is delayed.
Associated anomalies 10 %.
Important Associations : Intestinal
atresia.

membrane
Bowel normal.
Alimentation is normal.
Associated anomalies 60%.
Cardiac anomalies, Chromosomal
anomalies, Downs syndrome.

(Q.164) Which of the following branch of CN V3 runs in the mandibular canal?


Inferior alveolar nerve
(a)
Auriculotemporal nerve
(b)
Lingual nerve
(c)
Buccal nerve
(d)
Your Response
:
Correct Answer
A
:
Inferior alveolar nerve
(Ref. BDC Vol.3_5th/pg.159)
Exp:
Inferior alveolar nerve is the largest terminal branch of the posterior division of
the mandibular nerve. It runs vertically down lateral to medial pterygoid and to
the sphenomandibular ligament. It enters mandibular foramen and runs in the
mandibular canal. It is accompanied by inferior alveolar artery.

(Q.165) All the following is true about relations left kidney except:
It is related to pancreas without intervening peritoneum
(a)
Lateral part of lower pole is related to Splenic flexure
(b)
Medial part of lower pole is related to Jejunum and left colic artery.
(c)
Hilum is related to duodenum
(d)
Your Response
:
Correct Answer
D
:
Hilum is related to duodenum
Exp:
Relations of Kidneys:
Areas
Right kidney
Hilum
Second part of
1
duodenum
Lateral part of Hepatic flexure
2
lower pole
Medial part of Jejunum and ascending
3
lower pole
branch of right colic
artery
Upper
part
Under surface of liver
4
hepato-renal pouch and
adrenals

Left kidney
Tail of pancreas
Splenic flexure
Jejunum and left colic artery

Stomach bed, i.e., adrenals,


spleen & pancreas (without
intervening peritoneum).

(Q.165) All the following is true about relations left kidney except:
It is related to pancreas without intervening peritoneum
(a)
Lateral part of lower pole is related to Splenic flexure
(b)
Medial part of lower pole is related to Jejunum and left colic artery.
(c)
Hilum is related to duodenum
(d)

Your Response
:
Correct Answer
D
:
Axillary nerve.
Exp:
MUSCLE GROUP SUPPLIED
Serratus anterior
Supraspinatus and Infraspinatus
Teres major
Deltoid and teres minor
Extensors of forearms, wrist, proximal phalanges and
thumb
6 Musculocutaneous Flexors of arm & forearm
7 Median
Wrist and hand flexors
8 Ulnar
Wrist and hand flexors
1
2
3
4
5

NERVE
Long thoracic
Suprascapular
Lower subscapular
Axillary
Radial

(Q.166) Teres minor muscle is supplied by?


Upper subscapular nerve
(a)
Lower subscapular nerve
(b)
Dorsal subscapular nerve
(c)
Axillary nerve
(d)
Your Response
:
Correct Answer
D
:
NERVE
MUSCLE GROUP SUPPLIED
Exp:
1
2
3
4
5

Long thoracic
Suprascapular
Lower subscapular
Axillary
Radial

Serratus anterior
Supraspinatus and Infraspinatus
Teres major
Deltoid and teres minor
Extensors of forearms, wrist, proximal phalanges and
thumb
6 Musculocutaneous Flexors of arm & forearm
7 Median
Wrist and hand flexors
8 Ulnar
Wrist and hand flexors

(Q.166) Teres minor muscle is supplied by?


Upper subscapular nerve
(a)
Lower subscapular nerve
(b)
Dorsal subscapular nerve
(c)
Axillary nerve
(d)
Your Response
:
Correct Answer
D
:
Axillary nerve.
Exp:
NERVE
1 Long thoracic
2 Suprascapular
3 Lower subscapular

MUSCLE GROUP SUPPLIED


Serratus anterior
Supraspinatus and Infraspinatus
Teres major

4 Axillary
5 Radial

Deltoid and teres minor


Extensors of forearms, wrist, proximal phalanges and
thumb
6 Musculocutaneous Flexors of arm & forearm
7 Median
Wrist and hand flexors
8 Ulnar
Wrist and hand flexors

(Q.167) Vein of Galen is formed by union of?


Sigmoid sinus & straight sinus
(a)
Straight sinus & inferior sagittal sinus
(b)
Transverse sinus & Basal vein
(c)
Two internal cerebral veins
(d)
Your Response
:
Correct Answer
D
:
Two internal cerebral veins.
(Ref. BD Chaurasia, Anatomy, 3rded., Vol
Exp:
III 303)
Great Cerebral Vein Of Galen
Single median vein formed by union of two internal cerebral veins.
Terminates into straight sinus.
Its tributaries include the basilic vein, veins from pineal body, colliculi,
cerebellum and adjoining part of occipital lobes of cerebrum.
Vein of Galen malformation is one of the most common cause of congestive
cardiac failure in neonates.
Cerebral venous system can be divided into two basic components.
A Superficial System; The superficial system comprises of sagittal sinuses and
cortical veins and these drain superficial surfaces of both cerebral hemispheres.
A Deep System; The deep system comprises of lateral sinus, straight sinus and
sigmoid sinus along with draining deeper cortical veins.
Both these systems mostly drain themselves into internal jugular veins.
A. Superficial cerebral venous system The superficial cerebral veins can be
divided into three collecting systems. First, a mediodorsal group draining into
superior sagittal sinus (SSS) and the straight sinus (SS); Second, a lateroventral
group draining into the lateral sinus; and Third, an anterior group draining into
the cavernous sinus. These veins are linked by the great anastomotic vein of
Trolard, which connects the SSS to the middle cerebral veins. These are
themselves connected to the lateral sinus (LS) by the vein of Labbe. The veins
of the posterior fossa may again be divided into three groups:
Superior group draining into the Galenic system,
Anterior group draining into Petrosal sinus and

Posterior group draining into the torcular Herophili and neighbouring transverse
sinuses.
The veins of the posterior fossa are variable in course and angiographic
diagnosis of their occlusion is extremely difficult.
The Superior Sagittal Sinus (SSS) starts at the foramen cecum and runs
backwards towards the internal occipital protuberance, where it joins with the
straight sinus and lateral sinus to form the torcular Herophili. Its anterior part is
narrow or sometimes absent, replaced by two superior cerebral veins that join
behind the coronal suture. This fact should be borne in mind while evaluating
for cerebral venous thrombosis (CVT). The SSS drain major part of the cerebral
hemispheres.
The cavernous sinuses drain blood from the orbits, the inferior parts of the
frontal and parietal lobe and from the superior and inferior petrosal sinuses.
Blood from them flow into the internal jugular veins.
The straight sinus is formed by the union of inferior sagittal sinus and the great
vein of Galen. The inferior sagittal sinus runs in the free edge of falx cerebri8
and unites with the vein of Galen to form the straight sinus. It runs backwards
in the center of the tentorium cerebelli at the attachment of the falx cerebri,
emptying into the torcular Herophili at the internal occipital protuberance. The
lateral sinuses extend from torcular Herophili to jugular bulbs and consist of a
transverse and sigmoid portion. They receive blood from the cerebellum, the
brain stem and posterior parts of the hemisphere. They are also joined by some
diploic veins and small veins from the middle ear. There are numerous LS
anatomic variations that may be misinterpreted as sinus occlusion.
B. Deep cerebral venous system
The deep cerebral veins are more important than superficial veins from the
angiographic point of view. Three veins unite just behind the interventricular
foramen of Monro to form the internal cerebral vein. These include choroid
vein, septal vein and thalamostriate vein. The Choroid vein runs from the
choroid plexus of the lateral ventricle. The Septal vein runs from the region of
the septum pellucidum in the anterior horn of the lateral ventricle and the
thalamostriate vein runs anteriorly in the floor of the lateral ventricle in the
thalamostriate groove between the thalamus and lentiform nucleus. The point of
union of these veins is called the venous angle.
The internal cerebral veins of each side run posteriorly in the roof of the third
ventricle and unite beneath the splenium of the corpus callosum to form the
great cerebral vein. The internal cerebral veins, which lie within 2 mm of the
midline, are the most important deep veins since they can be used to diagnose
midline shifts.
The great cerebral vein of Galen is a short (1-2 cm long), thick vein that passes
posterosuperiorly behind the splenium of corpus callosum in the quadrigeminal
cistern. It receives the basal veins and the posterior fossa veins and drains to the
anterior end of the straight sinus where this unites with the inferior sagittal
sinus.
The basal vein of Rosenthal begins at the anterior perforated substance by the
union of anterior cerebral vein, middle cerebral vein and the striate vein. The
basal vein on each side passes around the midbrain to join the great cerebral

vein.
In summary, blood from the deep white matter of the cerebral hemisphere and
from the basal ganglia, is drained by internal cerebral veins. and basal veins of
Rosenthal, which join to form the great vein of Galen that drains into the
straight sinus. With the exception of wide variations of basal vein, the deep
system is rather constant compared to the superficial venous system. Hence
their thrombosis is easy to recognize.

(Q.168) Inferior tibiofibular joint is which type of joint ?


Gomphosis
(a)
Condylar
(b)
Syndesmosis
(c)
Synchondrosis
(d)
Your Response
:
Correct Answer
C
:
Syndesmosis.
Exp:
Thedistal tibiofibular joint(tibiofibularsyndesmosis) is formed by the rough,
convex surface of the medial side of the distal end of thefibula, and a rough
concave surface on the lateral side of thetibia.
Below, to the extent of about 4 mm. these surfaces are smooth, and covered
with cartilage, which is continuous with that of theankle-joint.
Theligamentsare:
Anterior ligament of the lateral malleolus
Posterior ligament of the lateral malleolus
Interosseous membrane of leg
TheInferior transverse ligament of the tibiofibular syndesmosisis included in
older versions ofGray's Anatomy, but not inTerminologia
Anatomica.[1]However, it still appears in some anatomy textbooks.
It should not be confused with thesuperior tibiofibular joint, which is the
onlysynovialtibiofibular joint, and is sometimes simply called the "tibiofibular
articulation".
Also kindly refer to explanation provided for Q no 125.

(Q.169) Derivative of 3rd pharyngeal arch?


Hyoid artery.
(a)
On left, aortic arch; on right, proximal part of right subclavian artery.
(b)
Common Carotid artery and proximal part of internal carotid artery.
(c)
Proximal part of pulmonary arteries and (on the left only) ductus arteriosus .
(d)
Your Response
:
Correct Answer
C
:
Common Carotid artery and proximal part of internal carotid artery.
Exp:
(Ref. Human Embryology by IB Singh, 6th ed., 212)

Aortic arch Derivatives


1st:

Part of Maxillary artery.

2nd:

Stapedial artery and hyoid artery.

3rd:

Common Carotid artery and proximal part of internal carotid artery.

4th:

On left, aortic arch; on right, proximal part of right subclavian artery.

6th: Proximal part of pulmonary arteries and (on the left only) ductus
arteriosus
6th arch = pulmonary and pulmonary-to-systemic shunt (ductus arteriosus).

(Q.170) A 31-year-old man had shrapnel wound in the neck. O/E, it was noticed that when he blew his
nose or sneezed, the skin above the right clavicle bulged upward. The upward bulging of skin
could be explained by?
Injury to the cervical pleur(a.)
(a)

(b)
(c)
(d)
Your Response
:
Correct Answer
:
Exp:

Damage to the suprapleural membrane.


Damage to the deep fascia in the root of the neck.
Nonunited fracture of the first ri(b.)

B
Damage to the suprapleural membrane.

(Ref. Snells anatomy pg. 58)

The shrapnel had torn the suprapleural membrane, which normally prevents the
cervical dome of the pleura from bulging up into the neck.
The suprapleural membrane is a strong fascial connective tissue layer overlying
the apices of the lungs above the superior thoracic inlet on each side. It can be
considered as an extension of the tendinous insertion of the scalenus minimus
muscle. It slopes down at the same oblique angle to the horizontal as the inlet.
Its attachments include:
anteriorly: 1st costal cartilage, deep surface of first rib
laterally: first rib
medially: merges with mediastinal pleura projecting inferiorly
posteriorly: transverse process of C7 vertebra
Inferiorly is the parietal pleura of the lung apices. Superiorly, are the structures
at the base of the neck e.g. the subclavian vessels.
The purpose of the suprapleural membrane is to act as a rigid barrier so as to
prevent changes in intrathoracic pressure drawing upon the contents of the
neck.

(Q.171) True about spleen is all except ?


The long axis of the spleen lies along the twelfth ri(b.)
(a)
(b)
(c)
(d)

The spleen is situated in the upper left quadrant of the abdomen beneath the
diaphragm.
A pathologically enlarged spleen extends downward and forward, toward the
umbilicus.
Because of the presence of the left colic flexure and the phrenicocolic ligament,
the spleen is unable to expand vertically downward.

Your Response
:
Correct Answer
A
:
The long axis of the spleen lies along the twelfth rib.
Exp:

(Ref. Snells

anatomy pg. 364)


The long axis of the spleen lies along the tenth rib.
The spleen, in healthy adult humans, is approximately 7 centimetres (2.8 in) to
14 centimetres (5.5 in) in length. It usually weighs between 150 grams
(5.3 oz)[10]and 200 grams (7.1 oz).An easy way to remember the anatomy of the
spleen is the 1357911 rule. The spleen is 1" by 3" by 5", weighs
approximately 7 oz, and lies between the 9th and 11th ribs on the left hand side.

Surfaces :
Thediaphragmatic surfaceof the spleen (or phrenic surface) is convex,
smooth, and is directed upward, backward, and to the left, except at its upper
end, where it is directed slightly to the middle. It is in relation with the under
surface of the diaphragm, which separates it from the ninth, tenth, and eleventh
ribs of the left side, and the intervening lower border of the left lung and pleura.
Thevisceral surfaceof the spleen is divided by a ridge into two regions: an
anterior or gastric and a posterior or renal. The gastric surface (facies gastrica)
is directed forward, upward, and toward the middle, is broad and concave, and
is in contact with the posterior wall of the stomach. Below this it is in contact
with the tail of the pancreas. Near to its mid-border is a long fissure, termed
thehilum. This is pierced by several irregular openings, for the entrance and exit
of vessels and nerves. The renal surface (facies renalis) is directed medialward
and downward. It is somewhat flattened, considerably narrower than the gastric
surface, and is in relation with the upper part of the anterior surface of the left
kidney and occasionally with the left suprarenal gland.
Like thethymus, the spleen possesses onlyefferent lymphatic vessels. The
spleen is part of the lymphatic system. Both the short gastric arteries and the
splenic artery supply it with blood.Thegerminal centersare supplied
byarteriolescalledpenicilliary radicles.

(Q.172) Which of the following muscles plays an important role in lifting the left foot off the ground
while walking?

(a)
(b)
(c)
(d)
Your Response
:
Correct Answer
:
Exp:

Left gluteus medius


Left gluteus maximus
Right adductor longus
Right gluteus medius

D
The right gluteus medius

(Ref. Snells anatomy pg. 207)

The right gluteus medius and the right gluteus minimus tilt the pelvis so that the
left lower limb is raised, thus permitting the left foot to be advanced forward
clear of the ground.

(Q.173) Which of the following muscle is evertor of the foot?


Tibialis anterior
(a)
Flexor hallucis longus
(b)
Peroneus longus
(c)
Tibialis posterior
(d)
Your Response
:
Correct Answer
C
:
Peroneus longus
(Ref. Snells anatomy pg. 207)
Exp:
The peroneus longus everts the foot.
The two tibialis muscles cause inversion.

(Q.174) Stab wounds of the kidneys involve other abdominal organs in a high percentage of cases. Of
the organs listed, which one is least likely to be damaged in this patient?
Stomach
(a)

(b)
(c)
(d)
Your Response
:
Correct Answer
:
Exp:

Spleen
Inferior vena cava
Left adrenal gland

C
Inferior vena cava

(Ref. Snells anatomy pg. 207)

The organs that are closely related to the kidneys are at a risk of direct
injury by a stab wound that traverses the renal parenchyma.
Relations of Kidneys:
Areas
Right kidney
Hilum
Second part of
1
duodenum
Lateral part of Hepatic flexure
2
lower pole
Medial part of Jejunum and ascending
3
lower pole
branch of right colic
artery
Upper part
Under surface of liver
4
hepato-renal pouch and
adrenals

Left kidney
Tail of pancreas
Splenic flexure
Jejunum and left colic artery

Stomach bed, i.e., adrenals,


spleen & pancreas (without
intervening peritoneum).

Thus from the this table it is apparent that Adrenals, Stomach and Spleen
may be directly involved rather than the IVC.

(Q.175) Which of the following nerve is most commonly damaged during wrist slash injury?

(a)
(b)
(c)
(d)
Your Response
:
Correct Answer
:
Exp:

Ulnar nerve
Radial nerve
Median nerve palmar cutaneous branch
Median nerve.

D
Median nerve.

(Ref. Kieth L Moore Anatomy pg. 790)

The median nerve lies very superficial at the wrist, in the interval between the
tendons of the flexor digitorum superficialis and the flexor carpi radialis
muscles just proximal to ts entry under the flexor retinaculum in the hand.
During wrist slash injuries, the other nerve which could be damaged is ulnar
nerve, if the injury was extended more medial.

(Q.176)

Glomeruli are present in:


Brain
(a)
Spleen
(b)
Adrenal cortex
(c)
None of the above
(d)
Your Response
:
Correct Answer
A
:
Brain
Exp:
In the cerebellum, Mossy fibersenter the granular layer from their points of
origin, many arising from the pontine nuclei, others from the spinal cord,
vestibular nuclei, etc. In the human cerebellum, the total number of mossy
fibers has been estimated at about 200 million. These fibers form excitatory
synapses with the granule cells and the cells of the deep cerebellar nuclei.
Within the granular layer, a mossy fiber generates a series of enlargements
called rosettes. The contacts between mossy fibers and granule cell dendrites
take place within structures calledglomeruli. Each glomerulus has a mossy
fiber rosette at its center, and up to 20 granule cell dendritic claws contacting it.
Terminals fromGolgi cells infiltrate the structure and make inhibitory synapses
onto the granule cell dendrites. The entire assemblage is surrounded by a sheath
of glial cells. Each mossy fiber sends collateral branches to several cerebellar
folia, generating a total of 2030 rosettes; thus a single mossy fiber makes
contact with an estimated 400600 granule cells.
Dear friends:- Glomeruli are present in cerebellum also!!!. Dr.
Bhatia.

(Q.177) The spinocerebellar fibres traverse through the ?


Superior cerebellar peduncle
(a)
Middle cerebellar peduncle
(b)
Inferior cerebellar peduncle
(c)
Both A and C
(d)
Your Response

:
Correct Answer
D
:
Both A and C
Exp:
The cerebellar connections.
Peduncle

Afferent pathway

Efferent pathway

Superior
(crossed) to:

Anterior spinocerebellar

From dentate nucleus

(uncrossed)
1. Thalamus
2. Cerebral cortex
3. Red nucleus
Middle

Pontocerebellar (crossed)relays

from cerebral cortex via pontinenuclei


Inferior
1.Vestibulocerebellar (uncrossed)
cortex andfastigial
2Posterior spinocerebellar (uncrossed)

From cerebellar

nucleus to vestibular nuclei

3. Olivocerebellar (crossed)
1. Function unknown

(Q.17 Which of the following opening is for?


8)

A
(a)
B
(b)
C
(c)
D
(d)
Your Response :
Correct Answer
B
:
Ex The prostatic urethra is the widest and most dilatable portion of the entire urethra. On the

p: posterior wall is a longitudinal ridge called the urethral crest. On each side of this ridge is a
groove called the prostatic sinus; the prostatic glands open into these grooves. On the summit of
the urethral crest is a depression, the prostatic utricle, which is an analog of the uterus and
vagina in females. On the edge of the mouth of the utricle are the openings of the two
ejaculatory ducts.

(Q.179) The tendon of which muscle is involved when the tuberosity of the fifth metatarsal bone is
avulsed in an inversion fracture?
Abductor digiti minimi
(a)

(b)
(c)
(d)
Your Response
:
Correct Answer
:
Exp:

Peroneus brevis
Peroneus longus
Tibialis anterior

B
Peroneus brevis
The peroneus (fibularis) brevis, a pronator and everter of the foot, inserts into
the tubercle at the base of the fifth metatarsal. The peroneus longus passes
under the tarsal arch to insert onto the plantar aspect of the first metatarsal. The
tibialis posterior inserts onto the navicular bone, whereas the tibialis anterior
inserts into the first cuneiform and first metatarsal. The abductor digiti minimi
inserts onto the proximal phalanx of the fifth toe.
Avulsion fracture of the 5thmetatarsal styloid, also known as apseudo-Jones
fracture, is one of the more common footavulsion injuries, and accounts for
over 90% offractures of the base of the 5thmetatarsal.
Despite what should be a simple entity, controversy exists, as well as confusion
in the literature, with the termJones fracturesometimes liberally (and
incorrectly) applied to this fracture. Traditionally this avulsion fracture has been

ascribed to the insertion ofperoneus brevisand is caused by forcible inversion of


the foot in plantar flexion, as may occur while stepping on a curb or climbing
steps. It is also relatively common among tennis players, accounting for it
sometimes being referred to as a "tennis fracture". Some authors believe that it
is due to the lateral cord of the plantar aponeurosis which also inserts at the
base, rather than the peroneus brevis tendon.
Radiographic features
Small fracture usually of the tip of the proximal 5thmetatarsal, oriented mostly
transversely (c.f apophysis which parallels the shaft). It usually does not reach
the articular surface (metatarsocuboid), but occasionally does.
In some instances the fracture may be occult.
Treatment and prognosis
In general these fractures can be treated conservatively, and heal well. For large
or very displaced fragments with intra-articular extension then operative
fixation may be indicated.
Differential diagnosis
A number of fractures occur at the base of the 5thmetatarsal (seefractures of the
proximal fifth metatarsal) as well as entities that mimic fractures:
stress fracture of the 5thmetatarsal
Jones fracture
os peroneum
os vesalianum
normalapophysis of the proximal 5thmetatarsal

(Q.180) Type of femoral fracture most likely to result in avascular necrosis of the femoral head in
adults is?

(a)
(b)
(c)
(d)
Your Response
:
Correct Answer
:
Exp:

Acetabular
Cervical
Intertrochanteric
Subtrochanteric

B
Cervical
Fractures of the femoral neck will completely interrupt the blood supply to the
femoral head in adults. If the capsular retinaculum also is torn, avascular
necrosis of the head will certainly occur because the only remaining blood
supply to the head (through the ligamentum teres) is inadequate to sustain it.
The nearer the fracture to the femoral head, the more likely will be disruption of
the retinacular blood supply.

(Q.181) Thoracic duct receives tributaries from all the following except
Bilateral ascending lumbar ducts
(a)
Bilateral descending thoracic ducts
(b)
Left upper intercostal ducts
(c)
Right bronchomediastinal lymphatic trunk
(d)
Your Response

:
Correct Answer
D
:
Right bronchomediastinal lymphatic trunk
Exp:
Right broncho-mediastinal lymphatic trunk (d) drains into right lymphatic
duct (and not thoracic duct).
THEME: Right half of the body above diaphragm (right superior quadrant)
drains into the right lymphatic duct.
-Right lymphatic duct receives the lymphatic drainage from the right half of the
head& neck (Jugular lymphatic trunk); right upper limb (right subclavian
lymphatic trunk) and the right thoracic cavity (right broncho-mediastinal
lymphatic trunk & the right upper intercostal ducts).
-Thoracic duct drains the body below the diaphragm and also the remaining left
superior quadrant of the body. Hence, it receives the left sided jugular,
subclavian and broncho-mediastinal lymphatic trunks). It also receives Left
upper intercostal ducts (c) whereas, right upper intercostal ducts empty into the
right lymphatic duct.
Thoracic duct receives three major lymphatic vessels at its commencement:
Bilateral ascending lumbar ducts (a);
Bilateral descending thoracic ducts (b) and Intestinal lymphatic trunks.
Bilateral descending thoracic ducts (b) drain the lower 6 intercostal spaces
(embryologically this region comes below diaphragm) and empty into the
thoracic duct. As noted above upper 6 intercostal spaces on the right drain into
the right lymphatic duct and left upper intercostals (c) into thoracic duct
accordingly.
Important: Right thoracic duct not only receives the lymphatics of right lung
(lungs lie above diaphragm) but also lower lobe of left lung ! Upper lobe of left
lung drains into the thoracic duct only.
Externally umbilicus (in place of diaphragm) is taken to be the watershed line
i.e., skin above the umbilicus on right side of the body drains into the right
lymphatic duct.

(Q.182) Which of the following structures does not penetrate perineal membrane
Deep dorsal vein of penis
(a)
Dorsal Artery of penis
(b)
Duct of bulbourethral glands
(c)
Urethra
(d)
Your Response
:
Correct Answer
A
:
Deep dorsal vein of penis
Exp:
The perineal membrane is an anatomical term for a fibrous membrane in the
perineum. The term "inferior fascia of urogenital diaphragm", is considered

equivalent to the perineal membrane.


It is the superior border of the superficial perineal pouch, and the inferior border
of the deep perineal pouch.
It is triangular in shape, and about 4 cm. in depth.
Its apex is directed forward, and is separated from the arcuate pubic ligament by
an oval opening for the transmission of the deep dorsal vein of the penis.
Its lateral margins are attached on either side to the inferior rami of the pubis
and ischium, above the crus penis.
Its base (posterior border) is directed toward the rectum, and connected to the
central tendinous point of the perineum (Perineal body).
It is perforated, about 2.5 cm. below the symphysis pubis, by the urethra, the
arteries to the bulb and the ducts of the bulbourethral glands close to the
urethral orifice; by the deep arteries of the penis; by the dorsal arteries and
nerves of the penis near the apex of the fascia.
Its base is also perforated by the perineal vessels and nerves, while between its
apex and the arcuate pubic ligament the deep dorsal vein of the penis passes
upward into the pelvis(i.e., doesnt perforate the perineal membrane).

(Q.183) All the following are branches of cavernous part of Internal carotid artery except
Ophthalmic Artery
(a)
Branches to cavernous sinus
(b)
Hypophyseal artery
(c)
Meningeal artery
(d)
Your Response
:
Correct Answer
A
:
Ophthalmic artery
Exp:
Ophthalmic artery (a) is a branch from the cerebral part of internal carotid
artery (and not cavernous).
There are several classification systems, the most recent of which was described
by Bouthillier et al. Their classification system is used clinically by
neurosurgeons, neuroradiologists and neurologists and relies on the
angiographic appearance of the vessel and histological comparison rather than
on the embryonic development.
There are seven segments in theBouthillier classification:
cervical segment
petrous (horizontal) segment
lacerum segment
cavernous segment
clinoid segment

ophthalmic (supraclinoid) segment


communicating (terminal) segment
Branches
Except for the terminal segment (C7) the odd numbered segments usually have
no branches, whereas the even numbered segments (C2, C4, C6) each have two
branches.
SEGMENT
BRANCHES
None
C1: cervical segment
C2: petrous (horizontal) segment Caroticotympanic artery

C3: lacerum segment


C4: cavernous segment

C5: clinoid segment


C6: ophthalmic (supraclinoid)
segment
C7: communicating segment

Vidian artery
None
Meningohypophyseal trunk
Inferolateral trunk
None
Ophthalmic artery
Superior hypophyseal artery
Posterior communicating artery
Anterior choroidal artery
Anterior cerebral artery
Middle cerebral artery

(Q.184) Venous return of lower limb on standing from the supine posture depends on all except
Deep fascia sleeve
(a)
Arterial pressure
(b)
Valves of perforators
(c)
Calf muscle contraction
(d)
Your Response
:
Correct Answer
B
:
Arterial pressure
Exp:
Arterial pressure (b) falls when an individual attains standing posture from
supine. Hence, it doesnt remain a significant factor for the venous return at this
moment.
Factors helping venous return
A. General factors
1. Negative intrathoracic pressure, which is made more negative during
inspiration.
2. Arterial pressure (b) and overflow from capillary bed.

3. Compression of veins accompanying arteries by arterial pulsation.


4. The presence of valves in veins which support the column of blood and
maintain unidirectional flow.
B. Local Factors (venous, muscular and fascial)
1. Veins of lower limb- more muscular, more valves (c) and superficial veins
connected to deep veins by perforators.
2. When the limb is active muscular contraction (d) compresses the deep
veins and drives blood upwards.
3. Tight sleeve of deep fascia (a) makes muscular compression of the veins
much more effective by limiting outward bulging of the muscles. (Weakness of
this sheath leads to accumulation of venous blood in the superficial veins of leg
reducing the venous return).
On standing from supine, immediately there is fall in venous return due to
gravity which leads to fall in cardiac output and consecutively fall in arterial
blood pressure. This is countered by autoregulatory mechanisms (baroreceptors)
increasing heart rate to maintain cardiac output.

(Q.185) Which of the following movements doesnt not happen in abduction of shoulder
Medial rotation of scapula
(a)
Elevation of humerus
(b)
Rotation of Clavicle at the sterno clavicular joint
(c)
Rotation at the axis of acromioclavicular joint
(d)
Your Response
:
Correct Answer
A
:
Medial rotation of scapula
Exp:
Lateral (and not medial a) rotation of scapula occurs during abduction of
shoulder. This is carried by the serratus anterior and the upper fibres of
trapezius.
Elevation (b) of the humerus is an integral part of abduction at shoulder and is
carried out by the deltoid muscle. This elevation occurs along with the scapular
rotation and makes the humerus go into its secondary socket i.e., sub-acromial
space.
Abduction at shoulder involves rotation of clavicle at its longitudinal axis. This
rotation occurs at both the sterno-clavicular (c) and the acromio-clavicular
(d) joints. This contributes about 60 rotation to the clavicle in total.

(Q.186) Which of the following brainstem nuclei is not derived from alar plate
Dentate
(a)
Inferior olivary
(b)
Hypoglossal
(c)
Substantia nigra
(d)

Your Response
:
Correct Answer
C
:
Hypoglossal
Exp:
Hypoglossal (c) nucleus is a motor nucleus and develops from the Basal plate
of neural tube.
Neural tube forms the CNS (central nervous system) and is divided transversely
into dorsal (sensory) and ventral (motor) parts by the sulcus limitans.
Neural tube has a dorsal alar plate which gives origin to the sensory nuclei and
a ventral plate separated forming the motor nuclei. The sensory and motor
nuclei are separated from each other by the sulcus limitans.
Cerebellum develops from the rhombic lip of alar plate and hence, Dentate
nucleus (a) is a derivative of alar plate.
- Inferior olivary nucleus (b) is also a derivative of alar plate and is connected
to the cerebellum by the climbing fibres.
-Vestibulo-cochlear nuclei are also derivatives of alar plate and are concerned
with the sensations of hearing & balance.
The tectum of midbrain (superior & inferior colliculi) is derived from
dorsal/sensory alar plate. Superior colliculus is related with visual sensations
and inferior colliculus is for auditory sensations (auditory pathway).
-The tegmentum (midbrain) is derived from the basal/motor plate and includes
the occulomotor and the trochlear nuclei. These nuclei control the motor
movements of eyeball.
-Red nucleus &Substantia nigra (d) are also considered under tegmentum,
derived from basal plate and are involved in motor activity (Extra-pyramidal
motor tracts).
Some authors consider Red nucleus & substantia nigra as a derivative if alar
plate (? Controversial), which have later migrated to the region of basal plate.
-But hypoglossal nucleus is definitely a derivative of basal plate.
Remember, in such questions we may have more than one answer but have to
choose the most appropriate option.
The main motor (basal plate) nuclei in the brainstem:
Occulomotor, Trochlear, Abducent and Hypoglossal (GSE)-for eyeball &
tongue muscles
Edinger Westphal, Lacrimatory, salivatory, Dorsal Vagal (GVE) for
smooth/cardiac muscles & glands
Trigeminal, Facial, Ambigus (SVE)-for pharyngeal arch muscles

(Q.187) Sternocleidomastoid is not supplied by the following artery


Superior thyroid
(a)
Thyrocervical trunk
(b)
Occipital
(c)
Post auricular
(d)
Your Response
:
Correct Answer
B
:
Thyrocervical trunk
Exp:
Thyrocervical trunk (b) seems to be the most appropriate answer, though
suprascapular artery is a branch of thyrocervical trunk and supplies the lower
part of sternocleidomastoid.
Remember, in such questions we may have no appropriate answer but we have
to use the strike out method and choose the less definite/left over option.
Upper part of the muscle is supplied by the occipital (c) and posterior
auricular (d) artery.
-Middle part is supplied by the superior thyroid
(a) artery.
Definitely a/c/d are stroked out and not our answers.
Applied anatomy: A superiorly based flap can be raised on sternocleidomastoid
to reconstruct the lip, floor of mouth and inner cheeks.
-But a conventional myo-cutaneous flap like pectoralis major flap is a better
alternative.
-Microvascular free transfer flaps have superseded all the conventional methods
now a day.

(Q.188) Trendelenburg test is positive due to injury to the nerve


Superior gluteal
(a)
Inferior gluteal
(b)
Obturator
(c)
Tibial
(d)
Your Response
:
Correct Answer
A
:
Superior gluteal
Exp:
Superior gluteal (a) nerve, if injured, paralyses the 3 muscles: gluteus medius,
gluteus minimus and tensor fascia latae and hence lead to Trendelenburg test
positive.
-These 3 muscles, especially the gluteus medius raises the unsupported hip

during walking, which otherwise will be pulled down by the gravity.


In Trendelenburg test this action of gluteus medius (superior pelvic tilt of
contralateral hip) is absent and we actually observe that there is a downward
drop of the unsupported hip -due to unopposed action of gravity.
This leads to Lurching gait in the patient.
Trendelenburg test becomes positive in congenital dislocation of hip/ long
standing fracture of neck of femur.

(Q.189) Mitral cells are present in


Kidney
(a)
Mitral valve
(b)
Olfactory tract
(c)
Optic nerve
(d)
Your Response
:
Correct Answer
C
:
Olfactory tract
Exp:
Mitral cells are present in second order neuron of Olfactory tract. The centre of
olfaction is present at uncus which lies near the hippo campal gyrus.

(Q.190) All are seen in Horners syndrome except


Heterochromia iridis
(a)
Ptosis
(b)
Miosis
(c)
Apparent exophthalmos
(d)
Your Response
:
Correct Answer
D
:
Apparent exophthalmos
Exp:
Horner syndrome presents with apparent enophthalmos (and not exophthalmos
d).
-This is due to paralysis of the orbitalis muscle of the orbit, which normally
keeps the eyeball prominent. Its nerve supply is sympathetic T1 and when
paralysed leads to appearance of sunken eyeball.
Horner syndrome is actually a triad of: Miosis (c) with ptosis (b) and
Anhydrosis.
-Most cases are idiopathic, but it could be due to brainstem lesions, carotid
dissection, or neoplasm compressing upon the sympathetic chain (E.g.,
Pancoasttumour).
Heterochromiairides (a) may be present if the lesion is in a child younger than
2 years. The affected iris may remain blue when the other iris changes to

brown. Iris pigmentation is under sympathetic control during development,


which is completed by age 2 years. Heterochromia is uncommon in patients
with Horner syndrome acquired later in life.
Sympathetic nervous system is concerned with the fright/fight/flight
(Emergency) reaction and leads to
Elevation of upper eyelid by Muller muscle (wide open eyes)
Pupillary dilatation by Dilator pupillae (wide open pupils)
Sweating (sudomotor)
Pallor of the skin (vasomotor)
This activity is under Sympathetic T-1 nerves in the face, and are lesioned in
Horner syndrome leading to:
Drooping of upper eyelid (ptosis) - due to unopposed orbicularis oculi
Pupillary constriction (miosis) due to unopposed sphincter pupillae
Loss of sweating (anhydrosis)
Red flushed face due to loss of vasoconstrictive tone to the skin

(Q.191) A patient came with history of fall and on examination there was tenderness between the
tendons of extensor pollicis longus and brevis. The likely lesion is
Scaphoid #
(a)

(b)
(c)
(d)
Your Response
:
Correct Answer
:
Exp:

1st metacarpal #
Lower end of radius #
Trapezoid #

A
Scaphoid #*
EPL & EPB form the boundaries of the anatomical snuff box. Scaphoid bone
lies at the floor of this box and its fracture leads to tenderness in the box.
Scaphoid fracture is a common injury encountered in family medicine. To avoid
missing this diagnosis, a high index of suspicion and a thorough history and
physical examination are necessary, because early imaging often is unrevealing.
Anatomic snuffbox tenderness is a highly sensitive test for scaphoid fracture,
whereas scaphoid compression pain and tenderness of the scaphoid tubercle
tend to be more specific. Initial radiographs in patients suspected of having a
scaphoid fracture should include anteroposterior, lateral, oblique, and scaphoid
wrist views. Magnetic resonance imaging or bone scintigraphy may be useful if
the diagnosis remains unclear after an initial period of immobilization.
Nondisplaced distal fractures generally heal well with a well-molded short arm
cast. Although inclusion of the thumb is the standard of care, it may not be
necessary. Nondisplaced proximal, medial, and displaced fractures warrant
referral to an orthopedic subspecialist. The scaphoid bone is the most

commonly fractured carpal bone; this injury occurs most often in young men.
Scaphoid fractures are rare in young children and the elderly because of the
relative weakness of the distal radius compared with the scaphoid in these age
groups.1Scaphoid fractures are significant because a delay in diagnosis can lead
to a variety of adverse outcomes that include nonunion, delayed union,
decreased grip strength, decreased range of motion, and osteoarthritis of the
radiocarpal joint. Timely diagnosis, appropriate immobilization, and referral
when indicated can decrease the likelihood of adverse outcomes.

(Q.192) Human communicating nerve is found in which organ?


Brain
(a)
Eye
(b)
Larynx
(c)
Ear
(d)
Your Response
:
Correct Answer
C
:
Larynx
Exp:
The internal branch of the superior laryngeal nerve is joined to the recurrent
laryngeal nerve by a communication called the ramus communicans of Loop
of Galen.
The Galen anastomosis (also called the ramus anastomoticus or Ansa of Galen)
is a connection between the RLN and the internal branch of the SLN (see the
image above). Generally, the posterior branch of the RLN contributes to the
anastomosis; however, the anterior branch can also contribute to the
anastomosis. Traditionally, the Galen anastomosis has been described to
provide purely sensory and autonomic innervation. More recent studies have
shown that the anastomosis may also contain motor fibers.Despite being
initially described as a single nerve, it may exist as a single trunk, several
branches, or a plexus.
The "human communicating nerve" is an anastomosis between the
external branch of the SLN and the distal RLN. Approximately 70% of
human larynges have this anastomosis. The human communicating nerve
may contain both sensory innervation to the larynx and motor innervation
to the thyroarytenoid muscle.

(Q.193) Left spinal lemniscus contains which fibers?


Ipsilateral pain touch temperature
(a)
Ipsilateral position vibration sense
(b)
Contralateral position vibration sense
(c)
Contralateral pain touch temperature
(d)
Your Response
:
Correct Answer
D
:
Contralateral pain touch temperature
Exp:
First order neuron carrying pain touch temperature enter the spinal cord, relay
in same side. Second order neuron cross over to other side and travel up in
anterior and lateral spino thalamic tract and enter the medulla. Same fiber are
now known as spinal lemniscus and they relay at thalamus. So left sided spinal
lemniscus contain pain touch temperature from right side of the body.

(Q.194) A missile hits the body just above the pubic ramus through the ant abdominal wall it will
pierce which of the following structure
Urinary bladder
(a)

(b)
(c)
(d)
Your Response
:
Correct Answer
:
Exp:

Spinal cord
Left renal vein
Abdominal aorta

A
Urinary bladder
Urinary bladder (a) lies just behind the pubis symphysis and gets damaged in
this case.
Spinal cord (b) terminates at the lower border of L-1 vertebra (called as the
transpyloric plane) which is higher than the level of umbilicus (L3-L4).
Left renal vein (c) is at the hilum of the kidney and also lies almost at the level
of trans- pyloric plane. It is higher than the level of projectile and is not pierced.
Abdominal aorta (d) bifurcates at the level of L-4 vertebra and escapes the
injury due to the projectile.

(Q.195) What is false regarding dentate line


Glands of Morgagni open below the line
(a)
Anal glands open at the line
(b)
Dentate line lies 2 cm(s) above the anal verge.
(c)
Transitional epithelium lies above the dentate line
(d)
Your Response
:
Correct Answer
D
:
Transitional epithelium lies above the dentate line
Exp:
-Transitional epithelium is the name of the changing epithelium from the simple
columnar variety to the stratified squamous variety, and is observed at the
pecten (transitional) zone of anal canal below (d) the dentate line.
-This transitional epithelium is not to be confused with the urothelium of renal
tract.
-Dentate line is at the openings of the anal glands (b) and lies about 23 mm (c)
above the anal verge.

(Q.196) Boundaries of ovarian fossa include ?


Posterior obliterated umbilical artery
(a)
Ureter posteriorly
(b)
Uterine tube anteriorly
(c)
Internal iliac artery laterally
(d)
Your Response

:
Correct Answer
B
:
Ureter posteriorly
Exp:
The ovary lies in a shallow depression, named the ovarian fossa, on the lateral
wall of the pelvis.
This fossa has the following boundaries:
Above: by the external iliac vessels
In front: by the obliterated umbilical artery
Behind: by the ureter

(Q.197) Teratomas may arise from the aberrant migration of


Epiblast
(a)
Hypoblast
(b)
Cytotrophoblast
(c)
Syncytiotrophoblast
(d)
Your Response
:
Correct Answer
A
:
Epiblast cells
Exp:
Epiblast cells form the primordial germ cells, which migrate from the region of
primitive streak to the region of future gonads to form the gametes like sperm &
oocyte.
Abnormal remnants of epiblast/primordial germ cells at the primitive streak or
their aberrant migration lead to formation of teratomas.

(Q.198) The following statements are true about Cauda equina syndrome except
Spinal segments affected are L-3 to Co
(a)
Saddle shaped Anaesthesia is observed
(b)
Knee and ankle reflexes are lost
(c)
Bladder/bowel & sexual functions are damaged severely
(d)
Your Response
:
Correct Answer
D
:
Bladder/bowel & sexual functions are damaged severely
Exp:
Severe bladder bowel disturbances are the characteristic feature of conus
medullary syndrome.
Cauda equina syndrome may also present with B&B disturbances, but in later
stage of the disease and the symptoms are not very severe.

Asymmetrical saddle shaped anaesthesia is observed in cauda equina.


Symptoms and Signs of Conus Medullaris and Cauda Equina Syndromes
Conus Medullaris
Cauda Equina Syndrome
Syndrome
Presentation Sudden and bilateral
Gradual and unilateral
Reflexes
Knee jerks preserved but Both ankle and knee jerks affected
ankle jerks affected
Radicular pain Less severe
More severe
Low back
More
Less
pain
Sensory
Numbness tends to be
Numbness tends to be more localized
symptoms and more localized to perianal to saddle area; asymmetrical, may be
signs
area; symmetrical and
unilateral; no sensory dissociation;
bilateral; sensory
loss of sensation in specific
dissociation occurs
dermatomes in lower extremities with
numbness and paresthesia; possible
numbness in pubic area, including
glans penis or clitoris
Motor
Typically symmetric,
Asymmetric areflexic paraplegia that
strength
hyperreflexic distal
is more marked; fasciculations rare;
paresis of lower limbs
atrophy more common
that is less marked;
fasciculations may be
present
Impotence
Frequent
Less frequent; erectile dysfunction
that includes inability to have
erection, inability to maintain
erection, lack of sensation in pubic
area (including glans penis or
clitoris), and inability to ejaculate
Sphincter
Urinary retention and
Urinary retention; tends to present
dysfunction atonic anal sphincter
late in course of disease
cause overflow urinary
incontinence and fecal
incontinence; tend to
present early in course of
disease

(Q.199) Superior rectal vein continues as


Superior mesenteric
(a)
Inferior mesenteric
(b)
Internal iliac
(c)
External iliac
(d)
Your Response
:
Correct Answer
B
:
Inferior mesenteric
Exp:
Superior rectal vein drains into the inferior mesenteric vein (vein of hind gut)
Inferior mesenteric drains into splenic vein, which later joins the superior
mesenteric vein to form portal vein.

Hence superior rectal vein belongs to portal circulation.

(Q.200) In a patient with a tumor in superior mediastinum compressing the superior vena cava, all of
the following veins would serve as alternate pathways for the blood to return to the right
atrium, except
Lateral thoracic vein
(a)

(b)
(c)
(d)
Your Response
:
Correct Answer
:
Exp:

Internal thoracic vein


Hemiazygos vein
Vertebral venous plexus

D
Vertebral venous plexus
-In case of superior vena caval obstruction, the blood returns to heart via the
inferior vena cava using the collateral circulation.
-Blood from the lateral thoracic vein (a) drains into superficial epigastric vein,
thence to great saphenous and progressively into the femoral and iliac(s),
ultimately reaching the inferior vena cava.
-Internal thoracic vein (b) continues as the superior epigastric vein and
connects to inferior epigastric in the rectus sheath. Inferior epigastric vein is a
tributary of external iliac vein and make the venous blood reach the common
iliac and thence to inferior vena cava.
-Hemi-azygous vein (c) drains the lower part of left posterior thoracic wall and
drains the blood into the superior vena cava via the azygous vein. If superior
vena cava is obstructed the blood flows retrograde and reaches the inferior vena
cava, through channels like lumbar azygous vein.
Internal vertebral venous plexus (d) also contributes in the collateral
circulation but is least significant.
.

Das könnte Ihnen auch gefallen